Você está na página 1de 148

2019 - Test 32 -

Exam Title :
Government...
Email :
Contact :

Note: If the exam is multi-lingual i.e. English and Hindi. Hindi solutions will be
after the completion of English solutions.
QUESTION 1. MTU4OTEyK1IuUmFndWwrcm9icmFndWxAZ21haWwuY29tK1FVRVNUSU9OIDA=
Consider the following statements about SPARC scheme?

1. The objective of the scheme is to boost India specific research in defence manufacturing
industries

2. Indian Institute of Science, Bangalore is the National Coordinating Institute to implement the
SPARC programme

3. All Indian Institutions ranked in the overall top 100 or category-wise top 100 in the India
Rankings (NIRF) are eligible to apply

Which of the above statements are correct?

a) 1 and 2 only
b) 2 and 3 only
c) 3 only
d) All of the above
Correct Answer: C
Your Answer: Unanswered
Explanation

Solution (c)

The Union Minister of Human Resource Development Prakash Javadekar launched the ‘SPARC’
scheme.

The objective of the scheme is to boost India specific research in social and pure sciences.

Here ‘SPARC’ stands for - 'Scheme for promotion of Academic and Research Collaboration'.
"SPARC will focus on joint research projects in focus areas, including technology, science and
social sciences, which are of relevance to India.

SPARC scheme aims at improving the research ecosystem of India’s higher educational
institutions by facilitating academic and research collaborations between Indian Institutions
and the best institutions in the world.

Indian Institute of Technology Kharagpur is the National Coordinating Institute to implement


the SPARC programme.

All Indian Institutions ranked in the overall top100 or category-wise top-100 in the India
Rankings (NIRF) are eligible to apply.

Only such private institutions which are falling in the above category, and also recognised
under Section 12(8) of UGC Act are eligible.

QUESTION 2. MTU4OTEyK1IuUmFndWwrcm9icmFndWxAZ21haWwuY29tK1FVRVNUSU9OIDE=
Recently the government has launched IMPRESS scheme with an aim to promote policy
research in social science. What does the term IMPRESS stands for?

IASbaba
Web: http://ilp.iasbaba.com/ Score:
Email: ilp@iasbaba.com 0.00 / 198
Page 1
2019 - Test 32 -
Exam Title :
Government...
Email :
Contact :

a) Impactful Policy Research in Social Sciences


b) Impactful Policy Research in Social Sector
c) Impressive Policy Research in Social Sector
d) Impulsive Policy Research in Social Sciences
Correct Answer: A
Your Answer: Unanswered
Explanation

Solution (a)

With an aim to promote policy research in social science areas Union government has launched
‘IMPRESS’ scheme in the country.

‘IMPRESS’ stands for - Impactful Policy Research in Social Sciences

The scheme will provide an opportunity for social science researchers in any institution in the
country which includes all universities (central and state).

The scheme will be implemented by the Indian Council of Social Science and Research (ICSSR).

Under IMPRESS, 1,500 research projects will be awarded for two years to support social
science research in the higher educational institutions.

QUESTION 3. MTU4OTEyK1IuUmFndWwrcm9icmFndWxAZ21haWwuY29tK1FVRVNUSU9OIDI=
Consider the following statements about Digi -Yatra

1. It was launched by the Union Ministry of Civil Aviation

2. With this initiative, ticket booking, airport entry and boarding pass security check-in will be
made digital

3. The scheme also provides tourists with free guest houses who are travelling to India for the
first time

Which of the above statements are correct?

a) 1 and 2 only
b) 2 and 3 only
c) 1 and 3 only
d) All of the above
Correct Answer: A
Your Answer: Unanswered
Explanation

Solution (a)

Union Ministry of Civil Aviation has released policy on biometric based digital processing of
passengers at airports called Digi Yatra.

The initiative seeks to promote paperless and hassle-free air travel.

IASbaba
Web: http://ilp.iasbaba.com/ Score:
Email: ilp@iasbaba.com 0.00 / 198
Page 2
2019 - Test 32 -
Exam Title :
Government...
Email :
Contact :

It will be operational by end of February, 2019 at Bengaluru and Hyderabad airports.

In later phase, Airports Authority of India (AAI) will roll out this initiative at Kolkata, Varanasi,
Pune and Vijayawada airports by April 2019.

Key points

1) Under Digi Yatra, there will be one-time verification at departure airport while travelling for
first time using ID.

2) After successful verification, facial recognition biometric will be captured and stored in Digi
Yatra ID.

3) With this initiative, ticket booking, airport entry and boarding pass security check-in will be
made digital.

4) For this system, passengers will be registered through centralized system and will be given
Digi Travel ID.

QUESTION 4. MTU4OTEyK1IuUmFndWwrcm9icmFndWxAZ21haWwuY29tK1FVRVNUSU9OIDM=
Which of the following statements about scheme Udyam Abhilasha are incorrect?

1. The scheme was launched by union ministry of Labour and employment

2. It was launched in all the 115 Aspirational Districts identified by NITI Aayog in 28 States

3. It aims to inspire rural youth in aspirational districts to be entrepreneurs by assisting them to


set up their own enterprise.

Choose the incorrect code

a) 1 and 2 only
b) 1 only
c) 2 only
d) None of the above
Correct Answer: B
Your Answer: Unanswered
Explanation

Solution (b)

Small Industries Development Bank of India (SIDBI) has launched Udyam Abhilasha a national
level entrepreneurship awareness campaign.

It was launched in 115 Aspirational Districts identified by NITI Aayog in 28 States

Objectives of Udyam Abhilasha

1) To inspire rural youth in aspirational districts to be entrepreneurs by assisting them to set up


their own enterprise.

2) To impart trainings through digital medium across the country,

IASbaba
Web: http://ilp.iasbaba.com/ Score:
Email: ilp@iasbaba.com 0.00 / 198
Page 3
2019 - Test 32 -
Exam Title :
Government...
Email :
Contact :

3) To create business opportunities for CSC-VLEs.

4) To focus on women aspirants in aspirational districts to encourage women entrepreneurship.

5) To assist participants to become bankable and avail credit facility from banks to set up their
own enterprise

5)

QUESTION 5. MTU4OTEyK1IuUmFndWwrcm9icmFndWxAZ21haWwuY29tK1FVRVNUSU9OIDQ=
Which of the following states have launched 'Nirman Kusuma' programme for providing
financial assistance to the children of the construction workers for their technical education in
the state ?

a) Madhya Pradesh
b) Bihar
c) Odisha
d) West Bengal
Correct Answer: C
Your Answer: Unanswered
Explanation

Solution (c)

Odisha government has launched 'Nirman Kusuma' scheme for providing financial assistance to
the children of the construction workers for their technical education in the state.

QUESTION 6. MTU4OTEyK1IuUmFndWwrcm9icmFndWxAZ21haWwuY29tK1FVRVNUSU9OIDU=
Consider the following statements about Atal Bimit Vyakti Kalyan Yojna

1. It was launched by the ministry of Human resources and Development

2. It is dedicated for the Insured persons covered under the Employees’ State Insurance Act

3. This scheme is a relief payable in cash directly to their Bank Account in case of
unemployment and while they search for new engagement

Which of the above statements are correct?

a) 1 and 2 only
b) 2 and 3 only
c) 1 and 3 only
d) All of the above
Correct Answer: B
Your Answer: Unanswered
Explanation

IASbaba
Web: http://ilp.iasbaba.com/ Score:
Email: ilp@iasbaba.com 0.00 / 198
Page 4
2019 - Test 32 -
Exam Title :
Government...
Email :
Contact :

Solution (b)

The Employees’ State Insurance Corporation (ESIC) has approved a new scheme- Atal Bimit
Vyakti Kalyan Yojna for Insured persons covered under the Employees’ State Insurance Act.

Key points

1) This scheme is a relief payable in cash directly to their Bank Account in case of
unemployment and while they search for new engagement.

2) The eligibility for availing Super Specialty treatment for dependents of Insured Person has
now been relaxed to insurable employment of one year with 156 days of contributions.

3) The ESIC also approved the proposal for increasing the funeral expenses from existing 10
thousand rupees to 15 thousand rupees being paid on the death of Insured Person.

QUESTION 7. MTU4OTEyK1IuUmFndWwrcm9icmFndWxAZ21haWwuY29tK1FVRVNUSU9OIDY=
Consider the following statements about Neelakurinji plant?

1) This plant flowers only once in 14 years

2) It is a tropical plant species endemic to Shola forests in Western Ghats.

Which of the above statements are correct?

a) 1 only
b) 2 only
c) Both 1 and 2
d) None of the above
Correct Answer: D
Your Answer: Unanswered
Explanation

Solution (d)

Tamil Nadu government has announced Scheme for protection of exotic Neela kurinji
(Strobilanthus kunthianus) plants that flower only once in 12 years.

This scheme comes following complaints that these rare and ecologically unique flowers are
being packaged and sold on the commercial basis.

Neelakurinji plant

1) Neelakurinji is a tropical plant species. It is native to Shola forests in Western Ghats.

2) It is also seen in Shevroys Hills in Eastern Ghats, Anamalai hills and Agali hills in Kerala and
Sanduru hills in Karnataka.

QUESTION 8. MTU4OTEyK1IuUmFndWwrcm9icmFndWxAZ21haWwuY29tK1FVRVNUSU9OIDc=

IASbaba
Web: http://ilp.iasbaba.com/ Score:
Email: ilp@iasbaba.com 0.00 / 198
Page 5
2019 - Test 32 -
Exam Title :
Government...
Email :
Contact :

Which of the following statements about Pradhan Mantri Annadata Aay Sanrakshan Abhiyan
(PM-AASHA) are correct?

1. It was an umbrella programme announced in the Union Budget for 2018

2. The scheme aims at ensuring remunerative prices to the farmers for their produce

3. It allows the states to choose from three schemes

Choose the correct code

a) 1 and 2 only
b) 2 and 3 only
c) 1 and 3 only
d) All of the above
Correct Answer: D
Your Answer: Unanswered
Explanation

Solution (d)

The Union Cabinet chaired by Prime Minister has approved a new Umbrella Scheme “Pradhan
Mantri Annadata Aay SanraksHan Abhiyan’ (PM-AASHA). The Scheme is aimed at ensuring
remunerative prices to the farmers for their produce as announced in the Union Budget for
2018.

Aim – The scheme aimed at ensuring remunerative prices to the farmers for their produce.

Its objective is to empower farmers and strengthen the Agriculture sector.

Allocation - The Cabinet has sanctioned Rs 15,053 crore to implement the PM-AASHA in the
next two financial years, of which Rs 6,250 crore will be spent this year.

Components of PM-AASHA: Under the PM-AASHA, states would be allowed to choose from
three schemes –

1) Price Support Scheme (PSS),

2) Price Deficiency Payment Scheme (PDPS)

3) Pilot of Private Procurement & Stockist Scheme (PPPS).

QUESTION 9. MTU4OTEyK1IuUmFndWwrcm9icmFndWxAZ21haWwuY29tK1FVRVNUSU9OIDg=
Which of the following statements about Atal Pension Yojana are correct?

1. It had replaced earlier government backed pension Swavalamban scheme

2. The age limit is from 18 to 65 years

3. It covers an overdraft facility of 10,000 Rs and accidental insurance of 2 lakh Rs

Choose the correct code

IASbaba
Web: http://ilp.iasbaba.com/ Score:
Email: ilp@iasbaba.com 0.00 / 198
Page 6
2019 - Test 32 -
Exam Title :
Government...
Email :
Contact :

a) 1 and 2 only
b) 2 and 3 only
c) 1 and 3 only
d) All of the above
Correct Answer: A
Your Answer: Unanswered
Explanation

Solution (a)

Union Cabinet decided to indefinitely extended Atal Pension Scheme, which had lapsed in
August 2018 to further incentivize people’s participation in the scheme.

Atal Pension Yojana (APY) –

It is a social security scheme launched by the government in 2015 to provide a defined pension
between Rs 1,000 to Rs 5,000.

It had replaced earlier government-backed pension Swavalamban scheme.

Key features of the schemes:

1) Overdraft facility doubled - The Finance Minister Arun Jaitley stated that the overdraft
facility has been doubled from Rs 5,000 to Rs 10,000 under the Yojana.

2) Age limit increased - The age limit has been revised to 18 to 65 years from the earlier 18 to
60 years.

3) Accidental insurance cover raised - The accidental insurance cover has been raised from Rs 1
lakh to Rs 2 lakh.

3)

QUESTION 10. MTU4OTEyK1IuUmFndWwrcm9icmFndWxAZ21haWwuY29tK1FVRVNUSU9OIDk=


Consider the following statements about Swadesh Darshan scheme

1. It was launched by ministry of Culture

2. It focuses on development and beautification of the identified pilgrimage destinations

3. The first project under Swadesh Darshan Scheme ‘North East Circuit: Imphal and Khongjom’
was inaugurated in Manipur

Which of the above statements are correct?

a) 1 and 2 only
b) 2 and 3 only
c) 3 only
d) All of the above
Correct Answer: C
Your Answer: Unanswered

IASbaba
Web: http://ilp.iasbaba.com/ Score:
Email: ilp@iasbaba.com 0.00 / 198
Page 7
2019 - Test 32 -
Exam Title :
Government...
Email :
Contact :
Explanation

Solution (c)

Under the plan scheme ‘Swadesh Darshan’ and ‘PRASAD’ the Ministry of Tourism provides
Central Financial Assistance (CFA) to State Governments/Union Territory Administrations, for
various tourism projects subject to availability of funds, liquidation of pending utilization
certificates and adherence to the scheme guidelines.

Under the Swadesh Darshan scheme, thirteen thematic circuits have been identified, for
development namely: North-East India Circuit, Buddhist Circuit, Himalayan Circuit, Coastal
Circuit, Krishna Circuit, Desert Circuit, Tribal Circuit, Eco Circuit, Wildlife Circuit, Rural
Circuit, Spiritual Circuit, Ramayana Circuit and Heritage Circuit.

Under the PRASAD scheme thirteen sites have been identified for development, namely:
Amritsar, Ajmer, Dwaraka, Mathura, Varanasi, Gaya, Puri, Amaravati, Kanchipuram,
Vellankanni, Kedarnath, Kamakhya and Patna.

Under the ‘PRASAD’ scheme the focus is on development and beautification of the identified
pilgrimage destinations. Whereas, in the ‘Spiritual Circuit’ identified under the Swadesh
Darshan scheme, the thrust is on development of particular thematic circuit consisting of
various religious/spiritual destinations in a State and Union Territory.

The first project under Swadesh Darshan Scheme ‘North East Circuit: Imphal and Khongjom’
was inaugurated in Manipur.

The project covers two sites i.e. Kangla Fort and Khongjom in Manipur.

QUESTION 11. MTU4OTEyK1IuUmFndWwrcm9icmFndWxAZ21haWwuY29tK1FVRVNUSU9OIDEw


Which of the following ministries have launched O-SMART scheme?

a) Ministry of Earth Sciences


b) Ministry of Science and Technology
c) Ministry of Electronics and Information Technology
d) Ministry of Power
Correct Answer: A
Your Answer: Unanswered
Explanation

Solution (a)

O-SMART scheme is launched by the Ministry of Earth Sciences

The scheme encompasses a total of 16 sub-projects addressing ocean development activities


such as Services, Technology, Resources, Observations and Science.

The services rendered under the O-SMART will provide economic benefits to a number of user
communities in the coastal and ocean sectors, namely, fisheries, offshore industry, coastal
states, Defence, Shipping, Ports etc. Currently, five lakhs fishermen community are receiving
this information daily through mobile which includes allocation of fish potential and local
weather conditions in the coastal waters. This will help in reducing the search time for
fishermen resulting savings in the fuel cost.

IASbaba
Web: http://ilp.iasbaba.com/ Score:
Email: ilp@iasbaba.com 0.00 / 198
Page 8
2019 - Test 32 -
Exam Title :
Government...
Email :
Contact :

Implementation of O-SMART will help in addressing issues relating to Sustainable Development


Goal-14, which aims to conserve use of oceans, marine resources for sustainable development.
This scheme (O-SMART) also provides necessary scientific and technological background
required for implementation of various aspects of Blue Economy.

The State of Art Early Warning Systems established under the O-SMART Scheme will help in
effectively dealing with ocean disasters like Tsunami, storm surges.

The technologies being developed under this Scheme will help in harnessing the vast ocean
resources of both living and non-living resources from the seas around India

QUESTION 12. MTU4OTEyK1IuUmFndWwrcm9icmFndWxAZ21haWwuY29tK1FVRVNUSU9OIDEx


Consider the following statements about Smart India Hackathon 2019

1. It is an initiative by ministry of Human resources and Development

2. It is the World's biggest Software and Hardware hackathon

3. It provides platform to solve some of pressing problems we face in our daily lives, and thus
inculcating a culture of product innovation and a mindset of problem solving.

Which of the above statements are correct?

a) 1 and 2 only
b) 2 and 3 only
c) 1 and 3 only
d) All of the above
Correct Answer: D
Your Answer: Unanswered
Explanation

Solution (d)

Smart India Hackathon 2019 is a nationwide initiative to provide students a platform to solve
some of pressing problems we face in our daily lives, and thus inculcate a culture of product
innovation and a mindset of problem solving.

The last edition of the hackathon saw over 5 million+ students from various engineering
colleges compete for the top prize at 35+ locations.

In SIH 2019, the students would also have the opportunity to work on challenges faced within
the private sector organisations and create world class solutions for some of the top companies
in the world, thus helping the Private sector hire the best minds from across the nation.

What is SIH 2019?

1) An initiative by Ministry of HRD, AICTE, Persistent Systems, i4c and Rambhau Mhalgi
Prabodhini

2) Involves 1 Lakh+ technical students, 3000+ technical institutions, 200+ organizations from
across India

IASbaba
Web: http://ilp.iasbaba.com/ Score:
Email: ilp@iasbaba.com 0.00 / 198
Page 9
2019 - Test 32 -
Exam Title :
Government...
Email :
Contact :

3) World's biggest Software and Hardware hackathon

4) 3rd edition of highly successful Smart India Hackathon initiative

5) Technology Students across India compete to creatively solve problems and offer technical
solutions

6) Harness expertise of students from IISc, IITs, NITs and AICTE/UGC approved institutions

Why join SIH 2019?

1) Get innovative solutions to your problems in cost effective ways

2) Opportunity to brand your organization nationally

3) Recognition and visibility for your organization across all technical institutions in India

4) Young techies from all over the country offer out-of-the-box solutions to your problems

5) Be part of World's biggest Open Innovation Movement

6) Opportunity to work with some of the best talent in the country

QUESTION 13. MTU4OTEyK1IuUmFndWwrcm9icmFndWxAZ21haWwuY29tK1FVRVNUSU9OIDEy


Which of the following statements about IMPRINT 2 are correct?

1. It is launched by the ministry of Human Resource Development

2. SERB (Science and Engineering Research Board) in the department of Science & Technology
(DST) is proposed to be made the nodal agency for implementing the IMPRINT-2 initiative

3. Under it only IITs and IISC, are asked to identify major areas where India is facing
engineering and technology challenges

Choose the correct code

a) 1 and 2 only
b) 2 and 3 only
c) 1 and 3 only
d) All of the above
Correct Answer: A
Your Answer: Unanswered
Explanation

Solution (a)

Union Ministry of Hume Resource Development (MHRD) has approved 122 new research
project proposals worth Rs 112 crore for funding under its IMPRINT-II (Impacting Research
Innovation and Technology) scheme.

IASbaba
Web: http://ilp.iasbaba.com/ Score:
Email: ilp@iasbaba.com 0.00 / 198
Page 10
2019 - Test 32 -
Exam Title :
Government...
Email :
Contact :

IMPRINT is first of its kind MHRD supported scheme to address major science and engineering
challenges and boost original scientific and technological research in 10 technology identified
domains in the country.

It was launched in November 2015 with an aim to develop new engineering education policy
and create road map to pursue engineering challenges

SERB (Science and Engineering Research Board) in the Department of Science & Technology
(DST) is proposed to be made the nodal agency for implementing the IMPRINT-2 initiative
working along with the National Coordinator . The SERB will create a separate vertical under
its overall set up with allocated resources and a time bound action plan for selection, funding
and management of research projects under IMPRINT. Secretary (HE) and Secretary (DST)
would be co-chairpersons of this vertical. The Expert Pool of Indian National Academy of
Engineering (INAE) will also be utilised for project review and knowledge management.

IMPRINT provides overarching vision that guides research into areas that are predominantly
socially relevant. Initially under IMPRINT-I, IITs and IISC were asked to identify major areas
where India is facing engineering and technology challenges. Now its scope also has been
expanded private institutions along with IITs and IISc under IMPRINT-II and jointly funded and
steered by MHRD and Department of Science and Technology (DST).

QUESTION 14. MTU4OTEyK1IuUmFndWwrcm9icmFndWxAZ21haWwuY29tK1FVRVNUSU9OIDEz


Consider the following statements about Pradhan Mantri Ujjwala Yojana

1. It was launched in 2016 with a theme of ‘Swachh Indhan, Behtar Jeevan.’

2. West Bengal has got the maximum number of LPG connections under the scheme

3. It is implemented by Ministry of New and Renewable energy

Which of the above statements are correct?

a) 1 and 2 only
b) 1 only
c) 1 and 3 only
d) All of the above
Correct Answer: B
Your Answer: Unanswered
Explanation

Solution (b)

Union Government has announced that it has reached target of providing 5 crore free LPG
connections under Pradhan Mantri Ujjwala Yojana (PMUY) almost eight months ahead of
schedule (i.e. in 27 months instead of 35 months).

World Health Organisation (WHO) has recognised PMUY as one of decisive intervention by
Government to address indoor air pollution which accounts for nearly 10 lakh deaths in year in
the country.

Key points:

IASbaba
Web: http://ilp.iasbaba.com/ Score:
Email: ilp@iasbaba.com 0.00 / 198
Page 11
2019 - Test 32 -
Exam Title :
Government...
Email :
Contact :

1) Union Government launched PMUY in May, 2016 with tagline of ‘Swachh Indhan, Behtar
Jeevan.’

2) It aims at providing clean-cooking fuel to poor households, who are otherwise vulnerable
tovarious health hazards associated with indoor air pollution and bringing in qualitative charges
in living standards.

3) PMUY is implemented by Ministry of Petroleum and Natural Gas.

4) Considering its huge success, government had revised target to 8 crores with budgetary
allocation of Rs. 12,800 crore.

5) Uttar Pradesh has got the maximum number of LPG connections (87 lakh) followed by West
Bengal (67 lakh) and Bihar (61 lakh)

QUESTION 15. MTU4OTEyK1IuUmFndWwrcm9icmFndWxAZ21haWwuY29tK1FVRVNUSU9OIDE0


Which of the following statements about Sukanya Samriddhi Scheme (SSY) are correct?

1. The minimum initial deposit to open the account under this scheme has been fixed at Rs 250

2. It is meant for girl children below the age of 10 years

3. A girl child is eligible for an SSY account only if she is a resident Indian citizen when the
account is opened

Choose the correct code

a) 1 and 2 only
b) 2 and 3 only
c) 1 and 3 only
d) All of the above
Correct Answer: D
Your Answer: Unanswered
Explanation

Solution (d)

The Union Government has reduced the minimum annual deposit requirement for accounts
under Sukanya Samriddhi Yojana from Rs 1000 to Rs 250.

The minimum initial deposit to open the account has also been reduced to Rs 250.

The move is aimed at enabling more people to take advantage of the girl child savings scheme.

Key points :

Aim - The scheme aims to provide financial security to a girl till the time that she gets married.

Age limit - The Sukanya Samriddhi Account scheme is meant for girl children below the age of
10 years.

IASbaba
Web: http://ilp.iasbaba.com/ Score:
Email: ilp@iasbaba.com 0.00 / 198
Page 12
2019 - Test 32 -
Exam Title :
Government...
Email :
Contact :

The account matures in 21 years before which it is in a lock-in period where funds cannot be
withdrawn.

Also, the interest rate for the July-September quarter is 8.1%.

Flexible deposit amounts: The account can be opened with a minimum deposit of Rs.250 and in
multiples of Rs.100 thereafter.

A maximum of Rs 1.5 lakhs can be deposited per account every year

A girl child is eligible for an SSY account only if she is a resident Indian citizen when the
account is opened, and remains so until the maturity or the closure of account.

Non-resident Indians can no longer open an SSY account. In fact, if you or your child's
residential status changes to non-resident or she takes up another country's citizenship during
the term of the scheme, no interest shall be paid from the date of citizenship or residential
status changes and the account will be considered closed.

QUESTION 16. MTU4OTEyK1IuUmFndWwrcm9icmFndWxAZ21haWwuY29tK1FVRVNUSU9OIDE1


Which of the following have conducted Swachh Survekshan Grameen 2018?

a) Ministry of Drinking Water & Sanitation


b) Ministry of Rural development and Panchayat Raj institutions
c) Ministry of Health and Family welfare
d) NITI Aayog
Correct Answer: A
Your Answer: Unanswered
Explanation

Solution (a)

The Ministry of Drinking Water and Sanitation had commissioned “Swachh Survekshan
Grameen-2018” (SSG 2018) through an independent survey agency to develop ranking of all
districts of India on the basis of quantitative and qualitative sanitation (Swachhata) parameters.
This ranking was done based on a comprehensive set of parameters including surveys of public
places like schools, Anganwadis, PHCs, Haat/ Bazaars, Panchayat and citizen’s perception of
Swachhata and their recommendations for improvement of the program and data from the
SBM-G IMIS.

Haryana was ranked as the best State while Satara District of Maharashtra was ranked as the
best district as per the ranking undertaken by Swachh SurvekshanGrameen 2018. Uttar
Pradesh was rewarded for maximum citizens’ participation.Later, Ms Uma Bharti, Union
Minister for Drinking Water and Sanitation gave awards at PravasiBhartiya Kendra, New Delhi
to top three States and Districts from all the zones of the country.

QUESTION 17. MTU4OTEyK1IuUmFndWwrcm9icmFndWxAZ21haWwuY29tK1FVRVNUSU9OIDE2


Consider the following statements about Solar Charkha Mission

IASbaba
Web: http://ilp.iasbaba.com/ Score:
Email: ilp@iasbaba.com 0.00 / 198
Page 13
2019 - Test 32 -
Exam Title :
Government...
Email :
Contact :

1. It was launched by ministry of New and renewable energy

2. The Khadi and Village Industries Commission (KVIC) is the implementing authority for this
mission

3. One of its objective is to boost rural economy and help in arresting migration from rural to
urban areas

Which of the above statements are correct?

a) 1 and 2 only
b) 2 and 3 only
c) 1 and 3 only
d) All of the above
Correct Answer: B
Your Answer: Unanswered
Explanation

Solution (b)

The Solar Charkha Mission is a Ministry of Micro Small & Medium Enterprises (MSME)
initiative launched during June 2018. The Khadi and Village Industries Commission (KVIC)
would implement the programme.

It was launched by President Ram Nath Kovind under which Government will disburse subsidy
of Rs 550 crore to thousands of artisans, generating employment in rural areas.

It was launched during the event of Udyam Sangam (National MSME Conclave) on the occasion
of World MSME Day (observed on 27 June).

The objectives of the Scheme are as follows:

1) To ensure inclusive growth by generation of employment, especially for women and youth,
and sustainable development through solar charkha clusters in rural areas.

2) To boost rural economy and help in arresting migration from rural to urban areas.

3) To leverage low-cost, innovative technologies and processes for sustenance

QUESTION 18. MTU4OTEyK1IuUmFndWwrcm9icmFndWxAZ21haWwuY29tK1FVRVNUSU9OIDE3


Consider the following statements about Seva Bhoj Yojna

1. It was launched by union Ministry of Culture

2. It provides financial assistance on purchase of specific food items by Charitable Religious


Institutions (CRIs) for free distribution among people

3. Under this scheme, entire Goods and Services Tax (CGST) charged on the raw food materials
purchased by the religious institutions will be refunded.

Which of the above statements are correct?

IASbaba
Web: http://ilp.iasbaba.com/ Score:
Email: ilp@iasbaba.com 0.00 / 198
Page 14
2019 - Test 32 -
Exam Title :
Government...
Email :
Contact :

a) 1 and 2 only
b) 2 and 3 only
c) 1 and 3 only
d) All of the above
Correct Answer: D
Your Answer: Unanswered
Explanation

Solution (d)

Union Ministry of Culture has launched a new scheme Seva Bhoj Yojna, to provide financial
assistance on purchase of specific food items by Charitable Religious Institutions (CRIs) for free
distribution among people.

The Union Ministry of Culture has launched Seva Bhoj Yojna, a new scheme to provide financial
assistance on purchase of specific food items by Charitable Religious Institutions (CRIs) for free
distribution among people.

Under this scheme, Centre’s share of Central Goods and Services Tax (CGST) charged on the
raw food materials purchased by the religious institutions will be refunded.

The scheme has been launched for financial years 2018-19 and 2019-20 with a total outlay of Rs
325.00 crores

QUESTION 19. MTU4OTEyK1IuUmFndWwrcm9icmFndWxAZ21haWwuY29tK1FVRVNUSU9OIDE4


Consider the following statements about Saubhagya scheme

1. It aims to provide energy access to all by last mile connectivity and electricity connections to
all remaining un-electrified households

2. Ministry of Power is the implementing authority of the scheme

3. The scheme covers rural households only

Which of the above statements are correct?

a) 1 and 2 only
b) 2 and 3 only
c) 1 and 3 only
d) All of the above
Correct Answer: A
Your Answer: Unanswered
Explanation

Solution (a)

Pradhan Mantri Sahaj Bijli Har Ghar Yojana - Saubhagya is to provide energy access to all by
last mile connectivity and electricity connections to all remaining un-electrified households in
rural as well as urban areas to achieve universal household electrification in the country.

IASbaba
Web: http://ilp.iasbaba.com/ Score:
Email: ilp@iasbaba.com 0.00 / 198
Page 15
2019 - Test 32 -
Exam Title :
Government...
Email :
Contact :

The electricity connection to households include release of electricity connections by drawing a


service cable from the nearest pole to the household premise, installation of energy meter,
wiring for a single light point with LED bulb and a mobile charging point. In case the electricity
pole is not available nearby from household for drawing service cable, the erection of additional
pole along with conductor and associated accessories shall also be covered under the scheme.

Highlights

1) The scheme is launched on the occasion of the birth centenary celebration of Pandit Deen
Dayal Upadhyaya on 25th September.

2) Scheme aims to achieve 24x7 power for all by 2019 by providing electricity connection to
each household across the country. 158912

3) Scheme will provide subsidy on equipment such as transformers, wires and meters.

4) Ministry of Power would be the implementing authority of the scheme.

5) Power connection will be provided in both rural and urban areas of the country.

QUESTION 20. MTU4OTEyK1IuUmFndWwrcm9icmFndWxAZ21haWwuY29tK1FVRVNUSU9OIDE5


Recently government announced that the last unconnected Indian village of Leisang had been
connected to the grid. Leisang is located in which of the following states?

a) Assam
b) Manipur
c) Mizoram
d) Nagaland
Correct Answer: B
Your Answer: Unanswered
Explanation

Solution (b)

Leisang is located in Manipur

QUESTION 21. MTU4OTEyK1IuUmFndWwrcm9icmFndWxAZ21haWwuY29tK1FVRVNUSU9OIDIw


Which the following statements about PRAAPTI App is correct?

a) It is launched by ministry of Power to bring transparency in power purchase transactions


between Generators and Discoms
b) It is launched by ministry of Power to bring transparency in power purchase by the
households
c) It is launched by ministry of Coal to bring transparency in coal allocations
d) It is launched by ministry of Power to address the issue of power theft
Correct Answer: A
Your Answer: Unanswered

IASbaba
Web: http://ilp.iasbaba.com/ Score:
Email: ilp@iasbaba.com 0.00 / 198
Page 16
2019 - Test 32 -
Exam Title :
Government...
Email :
Contact :
Explanation

Solution (a)

A Web portal and an App namely PRAAPTI (Payment Ratification and Analysis in Power
procurement for bringing Transparency in Invoicing of generators), www.praapti.in was
launched to bring transparency in power purchase transactions between Generators and
Discoms.

The App and Web Portal will capture the Invoicing and payment data for various long term PPAs
from the Generators. This will help the stakeholders in getting month-wise and legacy data on
outstanding amounts of Discoms against power purchase.

The app will also allow users to know the details related to the payments made by the Discoms
to the power generation company and when they were made.

PRAAPTI will also enable the consumers to evaluate financial performance of their Discoms in
terms of payments being made to the generation companies.

The Portal would also help DISCOMs and GENCOs to reconcile their outstanding payments

QUESTION 22. MTU4OTEyK1IuUmFndWwrcm9icmFndWxAZ21haWwuY29tK1FVRVNUSU9OIDIx


Which of the following statements about IP Nani correct?

1. It is a tech-savvy grandmother who helps the government and enforcement agencies in


combating human trafficking

2. It was launched in the conference on National Intellectual Property Rights Policy in New
Delhi

Choose the correct code

a) 1 only
b) 2 only
c) Both 1 and 2
d) None of the above
Correct Answer: B
Your Answer: Unanswered
Explanation

Solution (b)

The union minister Shri Suresh Prabhu launched the Intellectual Property (IP) mascot IP Nani
at the conference in New Delhi.

Mascot IP Nani is a tech-savvy grandmother who helps the government and enforcement
agencies in combating IP crimes with the help of her grandson “Chhotu” aka Aditya.

The IP mascot will spread awareness about the importance of Intellectual Property Rights
(IPRs) among people, especially children, in an interesting manner

IASbaba
Web: http://ilp.iasbaba.com/ Score:
Email: ilp@iasbaba.com 0.00 / 198
Page 17
2019 - Test 32 -
Exam Title :
Government...
Email :
Contact :

QUESTION 23. MTU4OTEyK1IuUmFndWwrcm9icmFndWxAZ21haWwuY29tK1FVRVNUSU9OIDIy


Which of the following statements about SWAYAM are correct?

1. It was launched by ministry of ministry of Human Resource Development

2. It uses the Massive Open Online Courses platform to bring online the educational content
taught in classrooms from 8th class till graduation to be accessed by anyone, anywhere at any
time

3. SWAYAM platform is indigenously developed by Ministry of Human Resource Development


(MHRD) and All India Council for Technical Education (AICTE) with the help of Microsoft

Choose the correct code

a) 1 and 2 only
b) 2 and 3 only
c) 1 and 3 only
d) All of the above
Correct Answer: C
Your Answer: Unanswered
Explanation

Solution (c)

Human Resource Development Ministry has launched an initiative ‘SWAYAM’ to train teachers
using digital platform.

The government will use the Massive Open Online Courses platform SWAYAM aims to update
the teaching technique of 1.5 million higher education faculty with focus on new and emerging
trends.

To achieve this, in the first phase, 75 discipline specific National Resource Centres (NRCs) have
been identified to prepare online training material and keep the teachers well-informed of latest
developments in their disciplines through the online refresher course.

Several institutions such as Central, State and Open Universities, National Institutes for
Technical Teachers Training, IITs, IIITs, NITs and other have been notified as NRCs in this
regard.

SWAYAM platform is indigenously developed by Ministry of Human Resource Development


(MHRD) and All India Council for Technical Education (AICTE) with the help of Microsoft and
would be ultimately capable of hosting 2000 courses and 80000 hours of learning: covering
school, under-graduate, post-graduate, engineering, law and other professional courses.

All the courses are interactive, prepared by the best teachers in the country and are available,
free of cost to the residents in India. More than 1,000 specially chosen faculty and teachers
from across the Country have participated in preparing these courses.

QUESTION 24. MTU4OTEyK1IuUmFndWwrcm9icmFndWxAZ21haWwuY29tK1FVRVNUSU9OIDIz

IASbaba
Web: http://ilp.iasbaba.com/ Score:
Email: ilp@iasbaba.com 0.00 / 198
Page 18
2019 - Test 32 -
Exam Title :
Government...
Email :
Contact :

Consider the following statements about Atal New India Challenge

1. It launched under Atal Innovation Mission (AIM) with an aim to bring-in innovations and
technologies relevant to the people

2. Under the Atal Innovation Mission grants of up to Rs 1 crore will be provided to the
applicants showing capability, intent, and potential to productize technologies

3. It was launched by NITI Aayog

Which of the above statements are correct?

a) 1 and 2 only
b) 2 and 3 only
c) 1 and 3 only
d) All of the above
Correct Answer: D
Your Answer: Unanswered
Explanation

Solution (d)

Atal New India Challenge - On 26th April 2018, NITI Aayog launched Atal New India Challenge
under Atal Innovation Mission (AIM) with an aim to bring-in innovations and technologies
relevant to the people.

Under the Atal Innovation Mission grants of up to Rs 1 crore will be provided to the applicants
showing capability, intent, and potential to productize technologies.

The objective of this initiative is to solve problems in 17 different areas, which would have
direct impact on improving lives of citizens and also generate employment.

Under this new initiative, Atal Innovation Mission has partnered with the Ministries of Road
Transport and Highways, Housing and Urban Affairs, Agriculture and Family Welfare, Drinking
Water & Sanitation and the Railway Board.

QUESTION 25. MTU4OTEyK1IuUmFndWwrcm9icmFndWxAZ21haWwuY29tK1FVRVNUSU9OIDI0


Consider the following statements about Unnat Bhrat Abhiyan

1. It was launched by ministry of Rural Development

2. It envisions to bring transformational change in rural development processes by leveraging


knowledge institutions to help build the architecture of an Inclusive India

3. Under this each MLA will adopt the nearby villages and visit them to get familiar with the life
of the village people and the problems faced by them in the day to day life

Which of the above statements are correct?

a) 1 and 2 only
b) 2 only

IASbaba
Web: http://ilp.iasbaba.com/ Score:
Email: ilp@iasbaba.com 0.00 / 198
Page 19
2019 - Test 32 -
Exam Title :
Government...
Email :
Contact :

c) 1 and 3 only
d) All of the above
Correct Answer: B
Your Answer: Unanswered
Explanation

Solution (b)

The Ministry of Human Resource Development (HRD) launched the second edition of the Unnat
Bharat Abhiyan in New Delhi.

Unnat Bharat Abhiyan is inspired by the vision of transformational change in rural development
processes by leveraging knowledge institutions to help build the architecture of an Inclusive
India.

The objective of this mission is to enable students from 750 colleges and universities across the
country will adopt the nearby villages and visit them to get familiar with the life of the village
people and the problems faced by them in the day to day life

The Mission of Unnat Bharat Abhiyan is to enable higher educational institutions to work with
the people of rural India in identifying development challenges and evolving appropriate
solutions for accelerating sustainable growth. It also aims to create a virtuous cycle between
society and an inclusive academic system by providing knowledge and practices for emerging
professions and to upgrade the capabilities of both the public and the private sectors in
responding to the development needs of rural India.

QUESTION 26. MTU4OTEyK1IuUmFndWwrcm9icmFndWxAZ21haWwuY29tK1FVRVNUSU9OIDI1


Consider the following statements about Revitalising Infrastructure and Systems in Education
(RISE) scheme

1. It was announced in Union Budget 2017-18 which aims to lend low-cost funds to government
higher educational institutions

2. Under it, only IITs, IIMs, NITs and IISERs can borrow from a Rs 1,00,000 crore corpus over
next 4 years to expand and build new infrastructure

3. Higher Education Financing Agency (HEFA), a non-banking financial company acts as a


financing agency for the scheme

Which of the above statements are correct?

a) 1 and 2 only
b) 2 and 3 only
c) 1 and 3 only
d) All of the above
Correct Answer: C
Your Answer: Unanswered
Explanation

Solution (c)

IASbaba
Web: http://ilp.iasbaba.com/ Score:
Email: ilp@iasbaba.com 0.00 / 198
Page 20
2019 - Test 32 -
Exam Title :
Government...
Email :
Contact :

RISE scheme was announced in Union Budget 2017-18. It aims to lend low-cost funds to
government higher educational institutions. Under it, all centrally-funded institutes (CFIs),
including central universities, IITs, IIMs, NITs and IISERs can borrow from a Rs 1,00,000 crore
corpus over next 4 years to expand and build new infrastructure. It will be financed via
restructured Higher Education Financing Agency (HEFA), a non-banking financial company.

Key facts

· With introduction of RISE, all financing for infrastructure development at CFIs in higher
education will be done through HEFA, which was set up by government as a Section 8 company
(a company with charitable objectives) in 2017 to mobilise funds from the market and offer 10-
year loans to centrally-run institutes.

· Equity Share: In order to mobilise funds Rs. 1 lakh crore corpus under RISE, HEFA will need
equity of Rs 10,000 crore, of which Rs 8,500 crore will be provided government and remaining
by Canara Bank, which partnered with government to set up HEFA, and other corporations.

· Target: All infrastructure and research projects sanctioned by HEFA are to be completed by
December 2022.

· Fund Raising: HEFA will release money directly to vendors or contractors on certification by
executing agency and educational institutes. Loans taken from HEFA, under the RISE
programme, will be paid back over 10 years. There will be different modes of loan repayment
for different institutes, based on their internal revenue.

QUESTION 27. MTU4OTEyK1IuUmFndWwrcm9icmFndWxAZ21haWwuY29tK1FVRVNUSU9OIDI2


Which of the following statements about KUSUM Yojana are correct?

1. It is formulated by the ministry of New and Renewable Energy

2. One of the primary aims of this program is to provide interested farmers with solar pumps

3. It proposes for the installation of grid-connected solar power plants each of Capacity up to 2
MW in the rural areas

Choose the correct code

a) 1 and 2 only
b) 2 and 3 only
c) 1 and 3 only
d) All of the above
Correct Answer: B
Your Answer: Unanswered
Explanation

Solution (b)

The Ministry of New and Renewable Energy has formulated a Scheme ‘Kisan Urja Suraksha
evam Utthaan Mahabhiyan (KUSUM)’.

The proposal on KUSUM Scheme provides for :-

IASbaba
Web: http://ilp.iasbaba.com/ Score:
Email: ilp@iasbaba.com 0.00 / 198
Page 21
2019 - Test 32 -
Exam Title :
Government...
Email :
Contact :

1) installation of grid-connected solar power plants each of Capacity up to 2 MW in the rural


areas;

2) installation of standalone off-grid solar water pumps to fulfil irrigation needs of farmers not
connected to grid; and

3) solarisation of existing grid-connected agriculture pumps to make farmers independent of


grid supply and also enable them to sell surplus solar power generated to DISCOM and get
extra income.

One of the primary aims of this program is to provide interested farmers with solar pumps. The
government states that 17.5 lakh solar powered pumps will be provided to agricultural labours.

The Govt. has also announced that it will take initiative to construct plants, which will generate
solar power. As per the draft, these plants will only be erected on infertile areas, capable of
generation a total of 28000 MW Power.

Apart from the solar power plants, government will work towards the installation of new solar
pumps in farms, which have Diesel pumps. The capacity of these pumps will be 720 MW. Apart
from distribution, the scheme also provides all farmers with the chance to earn more money by
installing the solar pumps. The excess amount of energy that the farmers generate can be sold
to the grid.

QUESTION 28. MTU4OTEyK1IuUmFndWwrcm9icmFndWxAZ21haWwuY29tK1FVRVNUSU9OIDI3


Which of the following statements about Operation Green are incorrect?

1. It is a price fixation scheme that aims to ensure farmers are given the right price for their
produce

2. It aims to aid farmers and help control and limit erratic fluctuations in the prices of tomatoes,
onions and potatoes only

Choose the correct code

a) 1 only
b) 2 only
c) Both 1 and 2
d) None of the above
Correct Answer: D
Your Answer: Unanswered
Explanation

Solution (d)

Note: Both the statements are correct. Incorrect options have been asked.

It will be launched with an allocation of Rs 500 crore on the lines of Operation Flood. It aims to
promote farmer producers organisations, processing facilities, agri-logistics and professional
management.

IASbaba
Web: http://ilp.iasbaba.com/ Score:
Email: ilp@iasbaba.com 0.00 / 198
Page 22
2019 - Test 32 -
Exam Title :
Government...
Email :
Contact :

It also aims to aid farmers and help control and limit erratic fluctuations in the prices of
tomatoes, onions and potatoes (TOP). It is essentially price fixation scheme that aims to ensure
farmers are given the right price for their produce. The idea behind it is to double the income of
farmers by the end of 2022.

QUESTION 29. MTU4OTEyK1IuUmFndWwrcm9icmFndWxAZ21haWwuY29tK1FVRVNUSU9OIDI4


Consider the following statements about Ekalavya Model Residential Schools (EMRS)

1. The scheme is being implemented by the ministry of Tribal Affairs

2. They are set up in States/UTs with grants under Article 275(1) of the Constitution of India

3. As per the budget 2018-19, every block with more than 50% ST population and at least
20,000 tribal persons, will have an Ekalavya Model Residential School by the year 2022.

Which of the above statements are correct?

a) 1 and 2 only
b) 2 and 3 only
c) 1 and 3 only
d) All of the above
Correct Answer: D
Your Answer: Unanswered
Explanation

Solution (d)

In the context of the trend of establishing quality residential schools for the promotion of
education in all areas and habitations in the country, the Ekalavya Model Residential Schools
(EMRS) for ST students take their place among the Jawahar Navodaya Vidyalayas, the Kasturba
Gandhi Balika Vidyalayas and the Kendriya Vidyalayas. Eklavya Model Residential Schools
(EMRS) are set up in States/UTs with grants under Article 275(1) of the Constitution of India.

The scheme is being implemented by the Ministry of Tribal Affairs, Government of India.

The objective of EMRS is to provide quality middle and high level education to Scheduled Tribe
(ST) students in remote areas, not only to enable them to avail of reservation in high and
professional educational courses and as jobs in government and public and private sectors but
also to have access to the best opportunities in education at par with the non ST population.

As per the budget 2018-19, every block with more than 50% ST population and at least 20,000
tribal persons, will have an Ekalavya Model Residential School by the year 2022.

QUESTION 30. MTU4OTEyK1IuUmFndWwrcm9icmFndWxAZ21haWwuY29tK1FVRVNUSU9OIDI5


Budget 2018 extended the Kisan Credit Card to which of the following?

1. Fishermen

2. Cattle owners

IASbaba
Web: http://ilp.iasbaba.com/ Score:
Email: ilp@iasbaba.com 0.00 / 198
Page 23
2019 - Test 32 -
Exam Title :
Government...
Email :
Contact :

3. Plantation workers

4. Mining workers

Choose the correct code

a) 1, 2 and 3 only
b) 1 and 2 only
c) 2, 3 and 4 only
d) 1 and 3 only
Correct Answer: B
Your Answer: Unanswered
Explanation

Solution (b)

Kisan Credit Card was extended to fishermen and cattle owners. It will enable them to avail the
easier loans. It will help people associated with milk production business in rural areas by
providing financial assistance and also to fisheries

QUESTION 31. MTU4OTEyK1IuUmFndWwrcm9icmFndWxAZ21haWwuY29tK1FVRVNUSU9OIDMw


Consider the following statements about Affordable housing fund

1. It is a fund of funds set up under SIDBI

2. It was announced in Economic Survey 2018-19

3. It is funded from priority sector lending shortfall and fully serviced bonds authorised by the
Government of India

Which of the above statements are correct?

a) 1 and 2 only
b) 2 and 3 only
c) 3 only
d) All of the above
Correct Answer: C
Your Answer: Unanswered
Explanation

Solution (c)

In Union Budget 2018, the Finance Minister Arun Jaitely has announced the new scheme to
provide homes at low cost.

Its objective is to boost demand and supply of low-cost homes.

Government will establish a dedicated Affordable Housing Fund (AHF) in National Housing
Bank, funded from priority sector lending shortfall and fully serviced bonds authorized by the
Government of India.

IASbaba
Web: http://ilp.iasbaba.com/ Score:
Email: ilp@iasbaba.com 0.00 / 198
Page 24
2019 - Test 32 -
Exam Title :
Government...
Email :
Contact :

QUESTION 32. MTU4OTEyK1IuUmFndWwrcm9icmFndWxAZ21haWwuY29tK1FVRVNUSU9OIDMx


Which of the following have initiated a programme, Swasth Bachche, Swasth Bharat?

a) Kendriya Vidyalaya Sangathan


b) Ministry of Human resource and development
c) NITI Aayog
d) Ministry of Health anf Family Welfare
Correct Answer: A
Your Answer: Unanswered
Explanation

Solution (a)

Swasth Bachche, Swasth Bharat’ was launched by HRD Minister Shri Prakash Javadekar in
Kochi, Kerala.

The programme is an initiative of Kendriya Vidyalaya Sangathan to prepare a physical Health


and Fitness Profile Card for more than 12 lakhs of Kendriya Vidyalaya students

QUESTION 33. MTU4OTEyK1IuUmFndWwrcm9icmFndWxAZ21haWwuY29tK1FVRVNUSU9OIDMy


Consider the following statements about Varishtha Pension Bima Yojana

1. The Life Insurance Corporation of India (LIC) would be the implementing body of the scheme

2. It aims to provide social security to elderly persons aged 60 years during their old age

3. Under the scheme, an assured pension with guaranteed rate of return of 8% per annum
would be provided for a period of ten years

Which of the above statements are correct?

a) 1 and 2 only
b) 2 and 3 only
c) 3 only
d) All of the above
Correct Answer: D
Your Answer: Unanswered
Explanation

Solution (d)

As a part of government’s commitment for financial inclusion and social security the Union
Cabinet has given its post-facto approval to launch Varishtha Pension Bima Yojana.

The Life Insurance Corporation of India (LIC) would be the implementing body of the scheme
during the current financial year.

IASbaba
Web: http://ilp.iasbaba.com/ Score:
Email: ilp@iasbaba.com 0.00 / 198
Page 25
2019 - Test 32 -
Exam Title :
Government...
Email :
Contact :

The scheme aims to provide social security to elderly persons aged 60 years during their old
age.

Under the scheme, an assured pension with guaranteed rate of return of 8% per annum would
be provided for a period of ten years. Besides, the nominee would have an option for pension on
a monthly / quarterly / half yearly and annual basis.

QUESTION 34. MTU4OTEyK1IuUmFndWwrcm9icmFndWxAZ21haWwuY29tK1FVRVNUSU9OIDMz


Which of the following statements about Modified Special Incentive Package Scheme (M-SIPS)

1. It promotes large-scale manufacturing in the Electronic System Design and Manufacturing


(ESDM) sector to achieve net zero imports by 2020

2. Under M-SIPS, the Government will provide subsidy of 25 percent on capital investments in
special economic zones (SEZs)

Choose the correct code

a) 1 only
b) 2 only
c) Both 1 and 2
d) None of the above
Correct Answer: A
Your Answer: Unanswered
Explanation

Solution (a)

The Government has approved Special incentive package to promote large-scale


manufacturing in the Electronic System Design and Manufacturing (ESDM) sector. The scheme
is called the Modified Special Incentive Package Scheme (M-SIPS).

Under M-SIPS, the Government will provide subsidy of 20 percent on capital investments in
special economic zones (SEZs) and 25 percent on capital investments in non-SEZs for individual
companies. It also provides for re-imbursement of CVD/excise for capital equipment for the non-
SEZ units. For high technology and high capital investment units, like fabs, re-imbursement of
central taxes and duties is also provided. The incentives are available for investments made in a
project within a period of 10 years from the date of approval. The Modified Special Incentive
Package Scheme (M-SIPS) encourages investments in the Electronics System Design and
Manufacturing sector in India

The incentives would be available for 29 category of ESDM products including telecom, IT
hardware, consumer electronics, medical electronics, automotive electronics, solar
photovoltaic, LEDs, LCDs, strategic electronics, avionics, industrial electronics, nano-
electronics, semiconductor chips and chip components, other electronic components. Units
across the value chain starting from raw materials including assembly, testing, packaging and
accessories of these categories of products are included.

IASbaba
Web: http://ilp.iasbaba.com/ Score:
Email: ilp@iasbaba.com 0.00 / 198
Page 26
2019 - Test 32 -
Exam Title :
Government...
Email :
Contact :
QUESTION 35. MTU4OTEyK1IuUmFndWwrcm9icmFndWxAZ21haWwuY29tK1FVRVNUSU9OIDM0
Consider the following statements about Pradhan Mantri Shram Yogi Mandhan Yojana

1. It will provide the pension benefits to unrecognized sector workers who monthly income is up
to Rs 15000.

2. The workers whose age is 18 to 40 years are eligible for enrolment

3. LIC will be the Pension Fund Manager and responsible for Pension pay out under the scheme

Which of the above statements are correct?

a) 1 and 2 only
b) 2 and 3 only
c) 1 and 3 only
d) All of the above
Correct Answer: D
Your Answer: Unanswered
Explanation

Solution (d)

Pradhan Mantri Shram Yogi Mandhan Yojana was announced by the Finance Minister Mr.
Piyush Goyal in Budget 2019. Under the PM Shram Yogi Mandhan Yojana the workers of
unrecognized sector whose monthly income is less than Rs 15000 may avail the benefits. Only
by contributing Rs. 100 per month the beneficiary will get Rs. 3000 per month after 60 year
age. Pradhan Mantri Shram Yogi Mandhan Yojana is also known social security pension scheme
for Economic Weaker Section.

PM-SYM Scheme is a Central government Scheme administered by the Ministry of Labour and
Employment

Shram Yogi Mandhan Yojana is being implemented through Life Insurance Corporation of India
and CSC eGovernance Services India Limited (CSC SPV).

LIC will be the Pension Fund Manager and responsible for Pension pay out. Application Form
PMSYM Yojana

PM-SYM pension scheme shall be invested as per the investment pattern specified by
Government of India.

In case subscriber exits the scheme within a period of less than 10 years, the beneficiary’s
share of contribution only will be returned to him with savings bank interest rate.

QUESTION 36. MTU4OTEyK1IuUmFndWwrcm9icmFndWxAZ21haWwuY29tK1FVRVNUSU9OIDM1


India has partnered with which of the following to implement National Rural Economic
Transformation Project (NRETP)?

a) World Bank
b) Asian Development Bank
c) European Union

IASbaba
Web: http://ilp.iasbaba.com/ Score:
Email: ilp@iasbaba.com 0.00 / 198
Page 27
2019 - Test 32 -
Exam Title :
Government...
Email :
Contact :

d) Japan
Correct Answer: A
Your Answer: Unanswered
Explanation

Solution (a)

The Union Cabinet has approved the Implementation of an Externally Aided Project namely
“National Rural Economic Transformation Project (NRETP) under the Deendayal Antyodaya
Yojana – National Rural Livelihoods Mission (DAY-NRLM) through loan assistance (IBRD Credit)
from World Bank.

The technical assistance provided by NRETP and the higher level interventions facilitated by
the project will enhance the livelihoods promotion and access to finance and scale-up initiatives
on digital finance and livelihood interventions.

DAY-NRLM lays special emphasis on targeting the poorest of the poor and the most vulnerable
communities and their financial inclusion. Innovative projects will be undertaken under NRETP
to pilot alternate channels of financial inclusion, creating value chains around rural products,
introduce innovative models in livelihoods promotion and access to finance and scale-up
initiatives on digital finance and livelihoods interventions. DAY-NRLM provides for mutually
beneficial working relationship and formal platforms for consultations between Panchayati Raj
Institutions (PRIs) and Community Based Organizations(CBOs). NRLM has also developed
activity map to facilitate convergence in different areas of interventions where NRLM
institutions and PRIs could work together which has been disseminated to all state Rural
Livelihood Missions.

QUESTION 37. MTU4OTEyK1IuUmFndWwrcm9icmFndWxAZ21haWwuY29tK1FVRVNUSU9OIDM2


Which of the following statements about Know India programme are correct?

1. The Ministry of External Affairs implements this programme in association with the chosen
partner state

2. The programme is open to youth of Indian origin including non-resident Indians with
preference to those from Girmitiya countries

3. It mandates a minimum qualification of graduation or being enrolled for graduation to be


part of the scheme

Which of the above statements are correct?

a) 1 and 2 only
b) 2 and 3 only
c) 1 and 3 only
d) All of the above
Correct Answer: C
Your Answer: Unanswered
Explanation

Solution (c)

IASbaba
Web: http://ilp.iasbaba.com/ Score:
Email: ilp@iasbaba.com 0.00 / 198
Page 28
2019 - Test 32 -
Exam Title :
Government...
Email :
Contact :

Know India Programme (KIP) is a flagship initiative for Diaspora engagement. The programme
aims to familiarize Indian-origin youth (18-30 years) with their Indian roots and contemporary
India.

Know India Programme is a three-week orientation programme for diaspora youth. The
programme aims to promote awareness on different facets of life in India and the progress
made by the country in various fields e.g. economic, industrial, education, science & technology,
communication & information Technology and culture.

The programme is open to youth of Indian origin (excluding non-resident Indians) with
preference to those from Girmitiya countries (Mauritius, Fiji, Suriname, Guyana, T&T, Jamaica
etc). The programme mandates a minimum qualification of graduation or being enrolled for
graduation to be part of the scheme

The Ministry of External Affairs implements the programme in association with the chosen
partner state.

QUESTION 38. MTU4OTEyK1IuUmFndWwrcm9icmFndWxAZ21haWwuY29tK1FVRVNUSU9OIDM3


Which of the following are correctly matched?

Scheme State

1. Rythu Bandhu A) Andhra Pradesh

2. Swayangsiddha B) West Bengal

3. Khushi C) Madhya Pradesh

Choose the correct code

a) 1 and 2 only
b) 2 and 3 only
c) 2 only
d) 1 and 3 only
Correct Answer: C
Your Answer: Unanswered
Explanation

Solution (c)

1) Odisha government has launched the KHUSHI scheme.

Its aim is to provide free sanitary napkins to school girls across the state.

2) Rythu bandhu scheme is launched by Telangana Government.

Rythu bandhu means friend of farmers.

The Rythu Bandhu (Agriculture Investment Support Scheme) takes care of initial investment
needs of every farmer.

IASbaba
Web: http://ilp.iasbaba.com/ Score:
Email: ilp@iasbaba.com 0.00 / 198
Page 29
2019 - Test 32 -
Exam Title :
Government...
Email :
Contact :

It provides a grant of Rs 4000 per acre per farmer each season for the purchase of inputs like
seeds, fertilizers, pesticides, labour and other investments in the field operations of farmers
choice form crop season.

3) Swayangsiddha scheme was launched by West Bengal government.

The scheme aims to empower young boys and girls to make informed choices so that they are
less vulnerable to trafficking and child marriage.

Swayangsiddha means self- reliance and it will be executed by West Bengal police.

QUESTION 39. MTU4OTEyK1IuUmFndWwrcm9icmFndWxAZ21haWwuY29tK1FVRVNUSU9OIDM4


Who will benefit from the Pradhan Mantri Kisan Samman Nidhi Yojana?

a) Small and marginal farmer who own cultivable land up to 2 hectares as per land records of
the concerned State/UT
b) Small and marginal farmer who own cultivable land up to 4 hectares as per land records of
the concerned State/UT
c) Any farmer who owns land up to 5 hectares as per land records of the concerned State/UT
d) Any farmer who owns non-agricultural land up to 6 hectares as per land records of the
concerned State/UT
Correct Answer: A
Your Answer: Unanswered
Explanation

Solution (a)

The interim budget 2019 has announced a farm income support scheme PM Kisan Samman
Nidhi (PM-KISAN). The feature of the PM Kisan Samman Nidhi scheme includes transferring Rs
6,000 every year to small and marginal farmers.

The Rs 6000 help will be provided in three instalments of Rs 2000 each in the bank accounts of
the eligible farmers. Under the scheme, Rs 6,000 will be given per year to small and marginal
farmer families having combined land holding/ownership of up to two hectares. The amount will
be given in three instalments of Rs 2,000 each and will be transferred directly to the bank
accounts of beneficiaries through the direct benefit transfer (DBT) scheme. The Direct Benefit
Transfer will ensure transparency in the process and save time for farmers.

The scheme aims to augment the income of the Small and Marginal Farmers (SMFs). It will
supplement the financial needs of the SMFs in procuring various inputs to ensure proper crop
health and appropriate yields, commensurate with the anticipated farm income at the end of
each crop cycle. The scheme will also help protect farmers from falling in the clutches of
moneylenders for meeting such expenses and ensure their continuance in the farming activities.

The SMFs landholder farmer family is defined as “a family comprising of husband, wife and
minor children who collectively own cultivable land up to 2 hectares as per land records of the
concerned State/UT

IASbaba
Web: http://ilp.iasbaba.com/ Score:
Email: ilp@iasbaba.com 0.00 / 198
Page 30
2019 - Test 32 -
Exam Title :
Government...
Email :
Contact :
QUESTION 40. MTU4OTEyK1IuUmFndWwrcm9icmFndWxAZ21haWwuY29tK1FVRVNUSU9OIDM5
Consider the following statements about Mission Raksha Gyan Shakti

1. It is instituted by the Department of Defence Production

2. It aims to provide a boost to the IPR culture in indigenous defence industry

3. The Directorate General of Quality Assurance (DGQA) has been entrusted with the
responsibility of coordinating and implementing the programme

Choose the correct code

a) 1 and 2 only
b) 2 and 3 only
c) 1 and 3 only
d) All of the above
Correct Answer: D
Your Answer: Unanswered
Explanation

Solution (d)

The Union Defence Minister, Nirmala Sitharaman on November 27, 2018 formally launched
‘Mission Raksha Gyan Shakti’ in New Delhi.

Mission Raksha Gyan Shakti has been instituted by the Department of Defence Production as a
part of the ongoing initiatives to enhance self-reliance in defence.

The mission aims to provide a boost to the IPR culture in indigenous defence industry.

The Directorate General of Quality Assurance (DGQA) has been entrusted with the
responsibility of coordinating and implementing the programme.

The end objective of the programme is to inculcate IP culture in Indian defence manufacturing
ecosystem.

QUESTION 41. MTU4OTEyK1IuUmFndWwrcm9icmFndWxAZ21haWwuY29tK1FVRVNUSU9OIDQw


Ministry of Human Resource Development have launched a scheme called ‘Samagra Shiksha’.
Which of the following statements regarding this are correct?

1. It subsumes the three Schemes of Sarva Shiksha Abhiyan (SSA), Rashtriya Madhyamik
Shiksha Abhiyan (RMSA) and Teacher Education (TE).

2. It treats school education holistically as a continuum from Pre-school to Class 12

3. Emphasis will be on improvement of learning outcome.

Select the code from following:

a) 1 and 2
b) 2 and 3

IASbaba
Web: http://ilp.iasbaba.com/ Score:
Email: ilp@iasbaba.com 0.00 / 198
Page 31
2019 - Test 32 -
Exam Title :
Government...
Email :
Contact :

c) 1 and 3
d) All of the above
Correct Answer: D
Your Answer: Unanswered
Explanation

Solution (d)

Samagra Shiksha

The Union Budget, 2018-19, has proposed to treat school education holistically without
segmentation from pre-nursery to Class 12. Samagra Shiksha - an overarching programme for
the school education sector extending from pre-school to class 12 has been, therefore, prepared
with the broader goal of improving school effectiveness measured in terms of equal
opportunities for schooling and equitable learning outcomes. It subsumes the three Schemes of
Sarva Shiksha Abhiyan (SSA), Rashtriya Madhyamik Shiksha Abhiyan (RMSA) and Teacher
Education (TE).

Holistic approach to education

• Single Scheme for the School Education Sector from Classes I to XII- extension of
interventions to senior secondary stage.
• Treat school education holistically as a continuum from Pre-school to Class 12
• Supporting States to initiate pre-primary education
• Inclusion of senior secondary levels and pre-school levels in support for School education
for the first time

Administrative reform

• Single and unified administrative structure leading to harmonized implementation


• Flexibility to States to prioritise their interventions under the Scheme
• An integrated administration looking at ‘school’ as a continuum

Enhanced Funding for Education

• The budget has been enhanced.


• Learning outcomes and steps taken for quality improvement will be the basis for
allocation of grants under the Scheme.

Focus on Quality of Education

• Emphasis on improvement of Learning Outcomes


• Enhanced Capacity Building of Teachers
• Focus on strengthening Teacher Education Institutions like SCERTs and DIETs to improve
the quality of prospective teachers in the system
• SCERT to be the nodal institution for in-service and pre-service teacher training – will
make training dynamic and need-based.
• Key focus on quality education emphasizing capacity building of teachers in online and
offline mode as well as strengthening of Teacher Education Institutions SCERT/DIET/BRC/
CRC/CTEs/IASEs.
• Annual Grant per school for strengthening of Libraries
• Almost 1 million schools to be given library grant .
• Enhanced focus on improving quality of education by focus on the two T’s – Teachers and
Technology
• Outcome oriented allocation of resources

IASbaba
Web: http://ilp.iasbaba.com/ Score:
Email: ilp@iasbaba.com 0.00 / 198
Page 32
2019 - Test 32 -
Exam Title :
Government...
Email :
Contact :

Focus on Digital Education

• Support ‘Operation Digital Board’ in all secondary schools over a period of 5 years, which
will revolutionize education- easy to understand, technology based learning classrooms
will become flipped classrooms.
• Enhanced use of digital technology in education through smart classrooms, digital boards
and DTH channels
• Digital initiatives like Shala Kosh, Shagun, Shaala Saarthi to be strengthened
• Strengthening of ICT infrastructure in schools from upper primary to higher secondary
level.
• “DIKSHA”, digital portal for teachers to be used extensively for upgrading skills of
teachers
• Enhanced Use of Technology to improve access and provision of quality education –
‘Sabko Shiksha Achhi Shiksha’

Strengthening of Schools

• Emphasis on consolidation of schools for improvement of quality


• Enhanced Transport facility to children across all classes from I to VIII for universal
access to school
• Increased allocation for infrastructure strengthening in schools
• Composite school grant increased and to be allocated on the basis of school enrolment.
• Specific provision for Swachhta activities – support ‘Swachh Vidyalaya’
• Improve the Quality of Infrastructure in Government Schools

Focus on Girl Education

• Empowerment of girls
• Upgradation of KGBVs from Class 6-8 to Class 6-12 .
• Self-defence training for girls from upper primary to higher secondary stage
• Stipend for CWSN girls to be provided from Classes I to XII. – earlier only IX to XII.
• Enhanced Commitment to ‘Beti Bachao Beti Padhao’

Focus on Inclusion

• Allocation for uniforms under RTE Act enhanced per child per annum.
• Allocation for textbooks under the RTE Act, enhanced per child per annum. Energized
textbooks to be introduced.
• Allocation for Children with Special Needs (CwSN) increased from Rs. 3000 to Rs. 3500
per child per annum. Stipend of Rs. 200 per month for Girls with Special Needs from
Classes 1 to 12.
• Commitment to ‘Sabko Shiksha Achhi Shiksha’

Focus on Skill Development

• Exposure to Vocational Skills at Upper Primary Level would be extended.


• Strengthening of vocational education at secondary level as an integral part of curriculum
• Vocational education which was limited to Class 9-12, to be started from class 6 as
integrated with the curriculum and to be made more practical and industry oriented.
• Reinforce emphasis on ‘Kaushal Vikas’

Focus on Sports and Physical Education

• Sports equipment will be provided to all schools under this component.


• Sports Education to be an integral part of curriculum

IASbaba
Web: http://ilp.iasbaba.com/ Score:
Email: ilp@iasbaba.com 0.00 / 198
Page 33
2019 - Test 32 -
Exam Title :
Government...
Email :
Contact :
• Every school will receive sports equipments under the scheme to inculcate and emphasize
relevance of sports in the school curriculum
• Support ‘Khelo India’

Focus on Regional Balance

• Promote Balanced Educational Development


• Preference to Educationally Backward Blocks (EBBs), LWEs, Special Focus Districts
(SFDs), Border areas and the 115 aspirational districts identified by Niti Aayog
• ‘Sabka Saath Sabka Vikas’ and Sabko Shiksha Achhi Shiksha

QUESTION 42. MTU4OTEyK1IuUmFndWwrcm9icmFndWxAZ21haWwuY29tK1FVRVNUSU9OIDQx


Consider the following statements regarding Scheme to Provide Quality Education in Madrasas
(SPQEM):

1. It has been launched by Ministry of Minority Affairs

2. To strengthen capacities in Madrasas for teaching of the formal curriculum subjects like
Science, Mathematics, Language, Social Studies etc. through enhanced payment of teacher
honorarium.

3. Strengthening of libraries/book banks and providing teaching learning materials at all levels
of madrasas.

Which of the above statements are correct regarding the scheme?

a) 1 and 2
b) 2 and 3
c) 1 and 3
d) All of the above
Correct Answer: B
Your Answer: Unanswered
Explanation

Solution (b)

Scheme to Provide Quality Education in Madrasas


(SPQEM)
SPQEM seeks to bring about qualitative improvement in Madrasas to enable Muslim children
attain standards of the national education system in formal education subjects. The salient
features of SPQEM scheme are :

• To strengthen capacities in Madrasas for teaching of the formal curriculum subjects like
Science, Mathematics, Language, Social Studies etc. through enhanced payment of
teacher honorarium.
• Training of such teachers every two years in new pedagogical practices.
• Providing Science labs, Computer labs with annual maintenance costs in the secondary
and higher secondary stage madrasas.
• Provision of Science/Mathematics kits in primary/upper primary level madrassas.

IASbaba
Web: http://ilp.iasbaba.com/ Score:
Email: ilp@iasbaba.com 0.00 / 198
Page 34
2019 - Test 32 -
Exam Title :
Government...
Email :
Contact :
• Strengthening of libraries/book banks and providing teaching learning materials at all
levels of madrasas.
• The unique feature of this modified scheme is that it encourages linkage of Madrasas with
National Institute for Open Schooling (NIOS), as accredited centres for providing formal
education, which will enable children studying in such Madrasas to get certification for
class 5, 8, 10 and 12. This will enable them to transit to higher studies and also ensure
that quality standards akin to the national education system. Registration & examination
fees to the NIOS will be covered under this scheme as also the teaching learning
materials to be used.
• The NIOS linkage will be extended under this scheme for Vocational Education at the
secondary and higher secondary stage of Madrasas.
• For the monitoring and popularization of the scheme it will fund State Madrasa Boards.
GoI will itself run periodic evaluations, the first within two years.

Note: The scheme has been launched by MHRD and not by Minority Affairs

QUESTION 43. MTU4OTEyK1IuUmFndWwrcm9icmFndWxAZ21haWwuY29tK1FVRVNUSU9OIDQy


Which of the following statements are correct regarding ‘Sakshar Bharat’?

1. The objective of the program is to achieve 80% literacy level at national level

2. The principal target of the programme is to impart functional literacy to 70 million non-
literate adults in the age group of 15 years and beyond.

3. Programme provides for coverage of only rural areas in the eligible districts

Select the code from following:

a) 1 and 2
b) 2 and 3
c) 1 and 3
d) All of the above
Correct Answer: D
Your Answer: Unanswered
Explanation

Solution (d)

Saakshar Bharat

Saakshar Bharat Programme goes beyond ‘3’ R’s (i.e. Reading, Writing & Arithmetic) ; for it
also seeks to create awareness of social disparities and a person’s deprivation on the means for
its amelioration and general well being. This programme was formulated in 2009 with the
objective of achieving 80% literacy level at national level, by focusing on adult women literacy
seeking – to reduce the gap between male and female literacy to not more than 10 percentage
points .it has four broader objectives, namely imparting functional literacy and numeracy to
non-literates; acquiring equivalency to formal educational system; imparting relevant skill
development programme; and promote a leaning society by providing opportunities for
continuing education. The principal target of the programme is to impart functional literacy to
70 million non-literate adults in the age group of 15 years and beyond. This Includes coverage
of 14 million Scheduled Castes(SCs), 8 million Scheduled Tribes(STs), 12 million minorities &

IASbaba
Web: http://ilp.iasbaba.com/ Score:
Email: ilp@iasbaba.com 0.00 / 198
Page 35
2019 - Test 32 -
Exam Title :
Government...
Email :
Contact :
36 million others. The overall coverage of women is aimed at 60 million. 410 districts belonging
to 27 States/UTs of the country were identified to be covered under Saakshar Bharat.

Eligibility criteria for coverage under Saakshar Bharat - A district, including a new district
carved out of an erstwhile district that had adult female literacy rate of 50 per cent or below, as
per 2001 census, were considered eligible for coverage under the Saakshar Bharat programme.
In addition, all left wing extremism-affected districts, irrespective of their literacy rate, were
also eligible for coverage under the programme. There were 365 districts in the country that
had adult female literacy rate of 50 per cent or below. Home Ministry had declared 35 districts
as left wing extremism affected districts. However, 30 left wing extremism affected districts
also had adult female literacy of 50 per cent or below. Initially 370 having the adult female
literacy of 50% or below as per 2001 census that qualified for coverage under the programme.
Since 2001, several eligible districts have been bifurcated or trifurcated. This has raised the
total number of eligible districts including 35 which are left wing extremism affected districts.
Programme provides for coverage of only rural areas in the eligible districts

QUESTION 44. MTU4OTEyK1IuUmFndWwrcm9icmFndWxAZ21haWwuY29tK1FVRVNUSU9OIDQz


The State Resource Centre(s) (SRCs) are mandated to provide academic and technical resource
support to adult and continuing education through development and production of material and
training modules. Which of the following comes under its scope of work?

1. Training literacy functionaries

2. Evaluation and monitoring of literacy projects

3. Providing infrastructure for high end technological research

Select the code from following:

a) 1 and 2
b) 2 and 3
c) 1 and 3
d) All of the above
Correct Answer: A
Your Answer: Unanswered
Explanation

Solution (a)

The State Resource Centre(s) (SRCs) are mandated to provide academic and technical resource
support to adult and continuing education through development and production of material and
training modules. In addition SRC would be required to conduct motivational and
environmental building, action research and evaluation and monitoring.

Scope of Work
• Preparation of teaching learning and training material for Adult Education programme
• Training literacy Functionaries
• Action Research
• Evaluation and monitoring of literacy projects
• Undertaking innovative projects to identify future needs Adult Education programme

IASbaba
Web: http://ilp.iasbaba.com/ Score:
Email: ilp@iasbaba.com 0.00 / 198
Page 36
2019 - Test 32 -
Exam Title :
Government...
Email :
Contact :
• Any other function incidental to accomplishment of the overall objectives of the NLM or
specifically assigned by the Ministry of Human Resource Development, Government of
India.

QUESTION 45. MTU4OTEyK1IuUmFndWwrcm9icmFndWxAZ21haWwuY29tK1FVRVNUSU9OIDQ0


Ministry of Social Justice and Empowerment has launched Deendayal Disabled Rehabilitation
Scheme (DDRS). Which of the following statements regarding the scheme are correct?

1. Scheme will be implemented in all the States and Union Territories of India.

2. Under the scheme NGOs are provided with financial assistance for running their projects for
the rehabilitation of persons with disability such as Special school, pre-school and early
intervention, Halfway Homes and Community Based Rehabilitation etc.

Select the code from following:

a) 1 only
b) 2 only
c) Both 1 and 2
d) Neither 1 nor 2
Correct Answer: B
Your Answer: Unanswered
Explanation

Solution (b)

Deendayal Disabled Rehabilitation Scheme (DDRS)

DDRS is being organized by the Department of Empowerment of Persons with Disabilities


(DEPwD), Ministry of Social Justice and Empowerment . It will cover 13 States of Eastern
& North Eastern region namely West Bengal, Jharkhand, Odisha, Chhattisgarh, Bihar, Assam,
Arunachal Pradesh, Manipur, Meghalaya, Mizoram, Nagaland, Sikkim & Tripura. The
stakeholders i.e. the NGOs receiving grant under DDRS, State Government Officers & District
level officers will be participating.

Provisions of the scheme

• Every year more than 600 NGOs are provided with financial assistance for running their
projects for the rehabilitation of persons with disability such as Special school, pre-school
and early intervention, Halfway Homes and Community Based Rehabilitation etc.
• The NGOs being funded are catering the rehabilitative services to more than 35000 to
40000 beneficiaries every year.

Facts and figures

• According to Census 2011, there were about 2.68 crore persons with disabilities in India,
constituting 2.21% of the total population in the country .
• The mandate of the Constitution of India is to ensure equality, freedom, justice and dignity
of all individuals, which implies an inclusive society for all, especially the disadvantaged.
• Article 41 of Part IV (Directive Principles of State Policy – which is particularly relevant
with regard to persons with disabilities.

IASbaba
Web: http://ilp.iasbaba.com/ Score:
Email: ilp@iasbaba.com 0.00 / 198
Page 37
2019 - Test 32 -
Exam Title :
Government...
Email :
Contact :
• In 2015, the Government launched the National Action Plan (NAP) for skill
development of persons with disabilities. The NAP envisages skilling of the persons with
disabilities on various trades so as to enhance their employability both in the government
and private sector and also to enhance the scope of their self-employment

QUESTION 46. MTU4OTEyK1IuUmFndWwrcm9icmFndWxAZ21haWwuY29tK1FVRVNUSU9OIDQ1


In order to provide safety for women in public spaces, the Government has identified eight
cities for implementation of Safe City project. Which of the following are the features of Safe
City Project?

1. Identification of sensitive hot spots in each city

2. Automated number plate reading machines to be deployed in extremely sensitive areas.

3. Improving street lighting and public toilet facilities for women.

Select the code from following:

a) 1 and 2
b) 2 and 3
c) 1 and 3
d) All of the above
Correct Answer: D
Your Answer: Unanswered
Explanation

Solution (d)

SCIM portal under Safe City Project: In order to provide safety for women in public spaces,
the Government has identified eight cities for implementation of Safe City project. The 8 major
cities which have been chosen are Ahmedabad, Bengaluru, Chennai, Delhi, Hyderabad, Kolkata,
Lucknow and Mumbai. The projects include creation on ground assets & resources and mindset
safety of women.

Some of the key features of the safe city project include:

• Identification of sensitive hot spots in each city,


• Installation of CCTV surveillance covering the entire hot spot.
• Automated number plate reading machines to be deployed in extremely sensitive areas.
• Intensive patrolling in vulnerable areas beyond the identified hot spots.
• Improving street lighting and public toilet facilities for women.
• Others like setting up women help desks in police stations, augmentation of women
support centres etc.

QUESTION 47. MTU4OTEyK1IuUmFndWwrcm9icmFndWxAZ21haWwuY29tK1FVRVNUSU9OIDQ2


Ministry of Labour and Employment have started VISION ZERO conference. It is related to

a) Zero labour force without insurance

IASbaba
Web: http://ilp.iasbaba.com/ Score:
Email: ilp@iasbaba.com 0.00 / 198
Page 38
2019 - Test 32 -
Exam Title :
Government...
Email :
Contact :

b) Bringing workers into organized sector from unorganized sector


c) Occupational safety and health
d) Zero defects in the articles produced by MSMEs
Correct Answer: C
Your Answer: Unanswered
Explanation

Solution (c)

Ministry of Labour and Employment started VISION ZERO conference and its relevance to
Occupational Safety and Health.

Objectives

• The concept of ‘Vision Zero’ is fast gaining international acceptance and is expected to
leverage the efforts of the Government of India to raise the occupational safety and health
standards in the country so as to improve the occupational safety and health situation.
• The concept of Vision Zero is based on four fundamental principles viz.
◦ Life is non-negotiable,
◦ Humans are fallible,
◦ Tolerable limits are defined by human physical resistance, and
◦ People are entitled to safe transport and safe workplaces.

The Vision is based on principles of Controlling Risks, Ensuring Safety and Health in Machines,
Equipment and Workplaces and Skill Upgradation of Workforce.

QUESTION 48. MTU4OTEyK1IuUmFndWwrcm9icmFndWxAZ21haWwuY29tK1FVRVNUSU9OIDQ3


The Ministry of Women and Child Development conducted first National workshop on POSHAN
Abhiyaan involving all the States and Union Territories. Which of the following statements
regarding POSHAN scheme are correct?

1. The Abhiyaan targets to reduce stunting, under-nutrition, anemia (among young children,
women and adolescent girls) and reduce low birth weight by 50%, 50%, 80% and 50% per
annum respectively.

2. The target of the mission is to bring down stunting among children in the age group 0-6
years from 38.4% to 25% by 2022.

Select the code from following:

a) 1 only
b) 2 only
c) Both 1 and 2
d) Neither 1 nor 2
Correct Answer: B
Your Answer: Unanswered
Explanation

Solution (b)

IASbaba
Web: http://ilp.iasbaba.com/ Score:
Email: ilp@iasbaba.com 0.00 / 198
Page 39
2019 - Test 32 -
Exam Title :
Government...
Email :
Contact :

POSHAN Abhiyaan was launched on 8th March 2018 by the Prime Minister in Jhunjhunu. The
Abhiyaan targets to reduce stunting, under-nutrition, anemia (among young children, women
and adolescent girls) and reduce low birth weight by 2%, 2%, 3% and 2% per annum
respectively.The target of the mission is to bring down stunting among children in the age
group 0-6 years from 38.4% to 25% by 2022.

POSHAN Abhiyaan

• The POSHAN (PM’s Overarching Scheme for Holistic Nourishment) Abhiyaan is


aimed to ensure attainment of malnutrition free India by 2022.
• The programme targets reduction of under-nutrition, anemia and low birth weight by
ensuring convergence of evidence-based nutrition interventions and by creating a mass
movement (Jan Andolan) for food nutrition in India.
• One of the key nutrition interventions to meet these targets is to improve the quality of
daily diets by making them nutritionally rich and locally sustainable.
• Academic institutions were also encouraged to frame scientific methodologies and
prepare achievable action plans with clearly defined indicators.

QUESTION 49. MTU4OTEyK1IuUmFndWwrcm9icmFndWxAZ21haWwuY29tK1FVRVNUSU9OIDQ4


Which of the following is the Objective of Operation Digital Board (ODB)?

a) ODB aims at providing an online platform for filing in Tender applications and keep the
process transparent.
b) It aims at providing online interface for Board meetings in PSUs
c) ODB aims at converting a class room into a digital class room and in addition to availability
of e-resources at any time and at any place to students.
d) None of the above
Correct Answer: C
Your Answer: Unanswered
Explanation

Solution (c)

Operation Digital Board (ODB)

Operation Digital Board to leverage technology in order to boost quality education in the
country. Operation Digital Board is a revolutionary step which will make the learning as well as
the teaching process interactive and popularize flipped learning as a pedagogical approach.

Objectives

• ODB aims at converting a class room into a digital class room and in addition to
availability of e-resources at any time and at any place to students.
• It will also help in provisioning of personalised adaptive learning as well as Intelligent
Tutoring by exploiting emerging technologies like Machine Learning, Artificial
Intelligence & Data Analytics.
• An expert committee has worked out optimum configuration of the Digital Class Rooms
under ODB.

IASbaba
Web: http://ilp.iasbaba.com/ Score:
Email: ilp@iasbaba.com 0.00 / 198
Page 40
2019 - Test 32 -
Exam Title :
Government...
Email :
Contact :

Implementation in Higher Education Institutions (HEIs)

• UGC will be the implementing agency for ODB in HEIs.


• It is estimated that there would be 5 lakh classrooms in the institutions which are aided
by Centre or State Governments. Out of this, based on the readiness, UGC proposed to
take up 300 universities and about 10,000 colleges in the first phase covering 2 lakh
classrooms.
• For the 2 lakh class rooms, the cost is estimated at Rs. 2000 crores. This can be
implemented as a Central scheme, as a loan from HEFA.

Implementation in Schools

• Digital / SMART board will be provided in all Government and Government – aided schools
having Secondary and Sr. Secondary classes.
• Nearly 1.5 lakh Secondary / Sr. Secondary schools will be covered under the scheme in
collaboration with the State and UTs

QUESTION 50. MTU4OTEyK1IuUmFndWwrcm9icmFndWxAZ21haWwuY29tK1FVRVNUSU9OIDQ5


Pradhan Mantri Adarsh Gram Yojna targets villages with a majority of population of

a) Scheduled Castes
b) Scheduled Tribes
c) Minorities
d) Below Poverty Line
Correct Answer: A
Your Answer: Unanswered
Explanation

Solution (a)

Pradhan Mantri Adarsh Gram Yojana (PMAGY) was launched on a Pilot basis during 2009-10 , f
ollowing the Finance Minister’s declaration in his Budget Speech. The Scheme aims at
integrated development of villages in which the population of Scheduled Castes is above 50%. A
total of 1000 villages from Tamil Nadu (225), Rajasthan (225), Bihar (225), Himachal Pradesh
(225) and Assam (100) were selected for the Pilot phase.

QUESTION 51. MTU4OTEyK1IuUmFndWwrcm9icmFndWxAZ21haWwuY29tK1FVRVNUSU9OIDUw


Ministry of Women and Child Development launched Pradhan Mantri Matru Vandana Yojana
(PMMVY). Which of the following statements are correct regarding the scheme?

1. It is a maternity benefit program where all women employees working in Center as State
organizations are given paid maternity leave for 8 months.

2. Pregnant and Lactating AWWs/ AWHs/ ASHA may also avail the benefits under the PMMVY
subject to fulfilment of scheme conditionalities.

Select the code from following:

IASbaba
Web: http://ilp.iasbaba.com/ Score:
Email: ilp@iasbaba.com 0.00 / 198
Page 41
2019 - Test 32 -
Exam Title :
Government...
Email :
Contact :

a) 1 only
b) 2 only
c) Both 1 and 2
d) Neither 1 nor 2
Correct Answer: B
Your Answer: Unanswered
Explanation

Solution (b)

Pradhan Mantri Matru Vandana Yojana (PMMVY) is a Maternity Benefit Programme that is
implemented in all the districts of the country in accordance with the provision of the National
Food Security Act, 2013.

Objectives

1. Providing partial compensation for the wage loss in terms of cash incentive s so that the
woman can take adequate res t before and after delivery of the first living child.
2. The cash incentive provided would lead to improved health seeking behaviour amongst
the Pregnant Women and Lactating Mothers (PW& LM).

Target beneficiaries

1. All Pregnant Women and Lactating Mothers, excluding PW&LM who are in regular
employment with the Central Government or the State Governments or PSUs or those who
are in receipt of similar benefits under any law for the time being in force.
2. All eligible Pregnant Women and Lactating Mothers who have their pregnancy on or after
01.01.2017 for first child in family.
3. The date and stage of pregnancy for a beneficiary would be counted with respect to her
LMP date as mentioned in the MCP card.
4. Case of Miscarriage/Still Birth :
◦ A beneficiary is eligible to receive benefits under the scheme only once.
◦ In case of miscarriage/still birth, the beneficiary would be eligible to claim the
remaining instalment(s) in event of any future pregnancy.
◦ Thus, after receiving the 1st instalment, if the beneficiary has a miscarriage, she
would only be eligible for receiving 2nd and 3rd instalment in event of future
pregnancy subject to fulfilment of eligibility criterion and conditionalities of the
scheme. Similarly, if the beneficiary has a miscarriage or still birth after receiving 1
st and 2nd instalments, she would only be eligible for receiving 3rd instalment in
event of future pregnancy subject to fulfilment of eligibility criterion and
conditionalities of the scheme.
4. Case of Infant Mortality: A beneficiary is eligible to receive benefits under the scheme
only once. That is, in case of infant mortality, she will not be eligible for claiming benefits
under the scheme, if she has already received all the instalments of the maternity benefit
under PMMVY earlier.
5. Pregnant and Lactating AWWs/ AWHs/ ASHA may also avail the benefits under the
PMMVY subject to fulfilment of scheme conditionalities.

Benefits under PMMVY

• Cash incentive of Rs 5000 in three instalments i.e. first instalment of Rs 1000/ - on early
registration of pregnancy at the Anganwadi Centre (AWC) / approved Health facility as
may be identified by the respective administering State / UT, second instalment of Rs
2000/ - after six months of pregnancy on receiving at least one ante-natal check-up (ANC)

IASbaba
Web: http://ilp.iasbaba.com/ Score:
Email: ilp@iasbaba.com 0.00 / 198
Page 42
2019 - Test 32 -
Exam Title :
Government...
Email :
Contact :
and third instalment of Rs 2000/ - after child birth is registered and the child has received
the first cycle of BCG, OPV, DPT and Hepatitis - B, or its equivalent/ substitute.
• The eligible beneficiaries would receive the incentive given under the Janani Suraksha
Yojana (JSY) for Institutional delivery and the incentive received under JSY would be
accounted towards maternity benefits so that on an average a woman gets Rs 6000 / - .

QUESTION 52. MTU4OTEyK1IuUmFndWwrcm9icmFndWxAZ21haWwuY29tK1FVRVNUSU9OIDUx


Consider the following statements regarding ‘Yuva Swabhiman Yojna’:

1. The scheme has been launched by the Uttar Pradesh government for Economically Weaker
Sections.

2. The Scheme would guarantee 100 days of employment every year to the EWS youths.

3. Youths already employed under MGNREGA will not be eligible for the scheme.

Which of the above statements are correct?

a) 1 only
b) 2 and 3
c) 1 and 3
d) All of the above
Correct Answer: B
Your Answer: Unanswered
Explanation

Solution (b)

The Madhya Pradesh government has announced the launch of a scheme to ensure temporary
employment to the youths from the economically weaker sections (EWS) in the urban areas.

Yuva Swabhiman Yojana

1. The Scheme would guarantee 100 days of employment every year to the EWS youths.
2. During their employment, youths would be given skills training of their choice, so that
they can take benefits of the available job opportunities.
3. While those in rural areas get employment under the Mahatma Gandhi National Rural
Employment Guarantee Act (MNREGA), the urban poor youth are left out.
4. This scheme will effectively cover them.

QUESTION 53. MTU4OTEyK1IuUmFndWwrcm9icmFndWxAZ21haWwuY29tK1FVRVNUSU9OIDUy


PM has recently launched Pravasi Teerth Darshan Yojana. Which of the following statements are
correct regarding this scheme?

1. A group of Indian diaspora will be taken on a government-sponsored tour of religious places


in India twice a year.

2. The government will bear all the expenses including the airfare from their country of
residence.

IASbaba
Web: http://ilp.iasbaba.com/ Score:
Email: ilp@iasbaba.com 0.00 / 198
Page 43
2019 - Test 32 -
Exam Title :
Government...
Email :
Contact :

3. All people of Indian-origin aged 45 to 65 can apply and a group will be selected out of them
with first preference given to people from ‘Girmitiya countries.

Select the code from following:

a) 1 and 2
b) 2 and 3
c) 1 and 3
d) All of the above
Correct Answer: D
Your Answer: Unanswered
Explanation

Solution (d)

Pravasi Teerth Darshan Yojana

1. Hon’ble PM has launched the scheme under which a group of Indian diaspora will be
taken on a government-sponsored tour of religious places in India twice a year.
2. The first batch of 40 Indian-origin people at the Pravasi Bharatiya Divas will begin their
tour.
3. They will be taken to religious places of all major religions in India and the government
will bear all the expenses including the airfare from their country of residence.
4. All people of Indian-origin aged 45 to 65 can apply and a group will be selected out of
them with first preference given to people from ‘Girmitiya countries’ such as Mauritius,
Fiji, Suriname, Guyana, Trinidad and Tobago, and Jamaica.

Girmityas

1. Girmityas or Jahajis are descendants of indentured Indian labourers brought to Fiji,


Mauritius, South Africa, East Africa, the Malay Peninsula, Caribbean and South America
(Trinidad and Tobago, Guyana and Suriname).
2. They were hired to work on sugarcane plantations for the prosperity of the European
settlers and save the Fijians from having to work on these plantations and thus to
preserve their culture.
3. “Agreement” is the term that has been coined into “Girmit”, referring to the “Agreement”
of the British Government with the Indian labourers.
4. It was termed to the length of stay in Fiji and the Caribbean, and when they would be
allowed to go back to India.

QUESTION 54. MTU4OTEyK1IuUmFndWwrcm9icmFndWxAZ21haWwuY29tK1FVRVNUSU9OIDUz


Under Atal Solar Krishi Pump Yojana, government is providing 95% subsidy on solar
agricultural pumpsets. Which of the states have launched this scheme?

a) Madhya Pradesh
b) Maharashtra
c) Odisha
d) Assam
Correct Answer: B
Your Answer: Unanswered

IASbaba
Web: http://ilp.iasbaba.com/ Score:
Email: ilp@iasbaba.com 0.00 / 198
Page 44
2019 - Test 32 -
Exam Title :
Government...
Email :
Contact :
Explanation

Solution (b)

Atal Solar Krishi Pump (ASKP) Yojana

1. Maharashtra govt. has launched ASKP scheme for farmers to provide subsidy of upto 95%
on solar agriculture pumpsets.
2. Farmers with less than 5 acres of land just need to day 5% i.e. Rs. 12,000 and get three
horse power pump
3. Farmers with more than 5 acres of land just need to pay Rs. 30,000 and get five horse
power solar powered pump

QUESTION 55. MTU4OTEyK1IuUmFndWwrcm9icmFndWxAZ21haWwuY29tK1FVRVNUSU9OIDU0


Menstrual Hygiene Scheme have been launched by Ministry of Health and Family Welfare.
Which of the following statements are correct regarding the scheme?

1. It aims at increasing awareness among adolescent girls on Menstrual Hygiene.

2. Improving access to and use of high quality sanitary napkins by adolescent girls in urban
areas only.

3. Ensuring safe disposal of Sanitary Napkins in an environmentally friendly manner.

Select the code from following:

a) 1 only
b) 2 and 3
c) 1 and 3
d) All of the above
Correct Answer: C
Your Answer: Unanswered
Explanation

Solution (c)

Menstrual Hygiene Scheme

To address the need of menstrual hygiene among adolescent girls residing primarily in rural
areas, Government of India is supporting the Menstrual Hygiene Scheme, under which, funds
are provided to States/UTs through National Health Mission for decentralized procurement of
sanitary napkins packs for provision primarily to rural adolescent girls at subsidized rates as
per proposals received from the States and UTs in their Programme Implementation Plans.

This scheme encompasses the following:

• Increasing awareness among adolescent girls on Menstrual Hygiene

• Improving access to and use of high quality sanitary napkins by adolescent girls in rural
areas.

• Ensuring safe disposal of Sanitary Napkins in an environmentally friendly manner.

IASbaba
Web: http://ilp.iasbaba.com/ Score:
Email: ilp@iasbaba.com 0.00 / 198
Page 45
2019 - Test 32 -
Exam Title :
Government...
Email :
Contact :

• Provision of funds to ASHAs to hold monthly meeting with adolescents to discuss issues
related to menstrual hygiene.

A range of IEC material has been developed around Menstrual hygiene Scheme, using a 360
degree approach to create awareness among adolescent girls about safe & hygienic menstrual
health practices which includes audio, video and reading materials for adolescent girls and job-
aids for ASHAs and other field level functionaries for communicating with adolescent girls.

ASHAs across the country are trained and play a significant role in promotion of use and
distribution of the sanitary napkins.

Department of Health Research, under the Ministry of Health, is involved in assessment of all
newer, alternative, environment friendly menstrual hygiene products to look into their safety
and acceptability features.

QUESTION 56. MTU4OTEyK1IuUmFndWwrcm9icmFndWxAZ21haWwuY29tK1FVRVNUSU9OIDU1


India started its National Supercomputing Mission (NSM) to enable India to leapfrog to the
league of world class computing power nations. Which of the following statements regarding
NSM are correct?

1. The scheme is implemented and steered jointly by the Department of Science and Technology
(DST) and Department of Electronics and Information Technology (DeitY).

2. The NSM is divided into two key tracks, build and buy, which are being spearheaded by the
C-DAC and Bengaluru’s Indian Institute of Science respectively.

Select the code from following:

a) 1 only
b) 2 only
c) Both 1 and 2
d) Neither 1 nor 2
Correct Answer: C
Your Answer: Unanswered
Explanation

Solution (c)

National Supercomputing Mission (NSM): Building Capacity and Capability

The Mission would be implemented and steered jointly by the Department of Science and
Technology (DST) and Department of Electronics and Information Technology (DeitY) at an
estimated cost of Rs.4500 crore over a period of seven years. (Note: the mission was launched
in 2015. It was in news as it has been able to develop two fastest meteorological
supercomputers of India.)

The Mission envisages empowering our national academic and R&D institutions spread over the
country by installing a vast supercomputing grid comprising of more than 70 high-performance
computing facilities. These supercomputers will also be networked on the National
Supercomputing grid over the National Knowledge Network (NKN). The NKN is another
programme of the government which connects academic institutions and R&D labs over a high

IASbaba
Web: http://ilp.iasbaba.com/ Score:
Email: ilp@iasbaba.com 0.00 / 198
Page 46
2019 - Test 32 -
Exam Title :
Government...
Email :
Contact :
speed network. Academic and R&D institutions as well as key user departments/ministries
would participate by using these facilities and develop applications of national relevance. The
Mission also includes development of highly professional High Performance Computing (HPC)
aware human resource for meeting challenges of development of these applications.

The NSM is divided into two key tracks, build and buy, which are being spearheaded by the C-
DAC and Bengaluru’s Indian Institute of Science respectively.

While Phase I involves assembling of the supercomputers, in Phase II, certain components like
the motherboards would be manufactured locally, and in the third phase, the supercomputer
would be designed in India by C-DAC.

QUESTION 57. MTU4OTEyK1IuUmFndWwrcm9icmFndWxAZ21haWwuY29tK1FVRVNUSU9OIDU2


National Cooperative Development Corporation (NCDC)has embarked on a mission of ‘Sahakar
22’. The aim of the mission is

a) To double the farmers income by 2022


b) To double the number of start ups in rural areas by 2022
c) To double the number of cooperative societies by 2022
d) To double the number of SHGs in rural areas
Correct Answer: A
Your Answer: Unanswered
Explanation

Solution (a)

The National Cooperative Development Corporation (NCDC) has come up with a youth-friendly
scheme ‘ Yuva Sahakar-Cooperative Enterprise Support and Innovation Scheme” for
attracting them to cooperative business ventures.

The scheme will be linked to Rs 1000 crore ‘Cooperative Start-up and Innovation Fund (CSIF)’
created by the NCDC. It would have more incentives for cooperatives of North Eastern region,
Aspirational Districts and cooperatives with women or SC or ST or PwD members. The funding
for the project will be up to 80% of the project cost for these special categories as against 70%
for others. The scheme envisages 2% less than the applicable rate of interest on term loan for
the project cost up to Rs 3 crore including 2 years moratorium on payment of principal. All
types of cooperatives in operation for at least one year are eligible.

Sahakar 22

NCDC, being the most preferred financial institution in the world of cooperatives, has embarked
on Sahakar 22, a Mission for Doubling Farmers’ Income by 2022.

QUESTION 58. MTU4OTEyK1IuUmFndWwrcm9icmFndWxAZ21haWwuY29tK1FVRVNUSU9OIDU3


Swadesh Darshan Scheme is one of the flagship schemes of the Ministry of tourism, for
development of thematic circuits in the country. In which of the following states is the Tribal
Tourism circuit being developed?

a) Assam

IASbaba
Web: http://ilp.iasbaba.com/ Score:
Email: ilp@iasbaba.com 0.00 / 198
Page 47
2019 - Test 32 -
Exam Title :
Government...
Email :
Contact :

b) Chattisgarh
c) Uttarakhand
d) Rajasthan
Correct Answer: B
Your Answer: Unanswered
Explanation

Solution (b)

Swadesh Darshan scheme is one of the flagship schemes of the Ministry of tourism, for
development of thematic circuits in the country in a planned and prioritised manner. The
scheme was launched in 2014 -15 and as on date the Ministry has sanctioned 74 projects worth
Rs. 5997.47 Crore to 31 States and UTs. 30 projects / major components of these projects are
expected to be completed this year.

Development of Tribes and Tribal Culture is one of the prime area of focus for the Ministry of
Tourism. The Ministry is carrying out an array of activities for development and promotion of
tourism in the tribal region. The Ministry is developing the tourism infrastructure in the region
under its schemes of Swadesh Darshan. Under the tribal circuit theme of the scheme the
Ministry has sanctioned 4 projects to Nagaland, Telangana and Chhattisgarh for Rs. 381.37
Crores.

The first Tribal circuit is being developed in Chattisgarh. The project covers thirteen sites in
Chhattisgarh i.e. Jashpur, Kunkuri, Mainpat, Kamleshpur, Maheshpur, Kurdar, Sarodadadar,
Gangrel, Kondagaon, Nathiya Nawagaon, Jagdalpur, Chitrakoot, Tirthgarh.

QUESTION 59. MTU4OTEyK1IuUmFndWwrcm9icmFndWxAZ21haWwuY29tK1FVRVNUSU9OIDU4


The government has granted the status of a trust to ‘Bharat Ke Veer’. Which of the following
statements are correct regarding ‘Bharat ke Veer’?

a) It is an online initiative to highlight the role of martyrs of Indian paramilitary forces and
army who gave their life in the line of duty.
b) It is a grievance redressal portal for paramilitary forces where jawans can raise their
issues.
c) It is a portal where people can contribute for the families of jawans who gave their life in
the line of duty.
d) It is a portal where life style and training of Indian soldiers is discussed to create
awareness.
Correct Answer: C
Your Answer: Unanswered
Explanation

Solution (c)

Bharat ke Veer

1. The government has granted the status of a trust to ‘Bharat Ke Veer’, a private initiative
which aids families of paramilitary personnel killed in action.
2. Contributions to Bharat Ke Veer have been exempted from Income Tax, Home.

IASbaba
Web: http://ilp.iasbaba.com/ Score:
Email: ilp@iasbaba.com 0.00 / 198
Page 48
2019 - Test 32 -
Exam Title :
Government...
Email :
Contact :
3. Akshay Kumar and former national badminton champion Pullela Gopichand have been
included as trustees.
4. The initiative had now been formalized into a registered trust for providing a platform for
all citizens to contribute and provide assistance to the families of martyred personnel.
5. The public can visit the ‘Bharat Ke Veer’ application and website, and contribute to
support the families of jawans who die in the line of duty.

QUESTION 60. MTU4OTEyK1IuUmFndWwrcm9icmFndWxAZ21haWwuY29tK1FVRVNUSU9OIDU5


The Ministry of Heavy Industries has launched FAME India Scheme. Which of the following is
the objective of the scheme?

a) To promote sustainable practices in Indian heavy industries to minimize the carbon


footprint
b) To promote manufacturing of electric and hybrid vehicle technology
c) To use people with international influence to promote Indian industrial products worldwide
and increase Indian exports
d) All of the above
Correct Answer: B
Your Answer: Unanswered
Explanation

Solution (b)

In order to promote manufacturing of electric and hybrid vehicle technology and to ensure
sustainable growth of the same, Department of Heavy Industry is implementing FAME-India
Scheme.

Scheme proposes to give a push to electric vehicles (EVs) in public transport and seeks to
encourage adoption of EVs by way of market creation and demand aggregation. The draft
scheme envisages the holistic growth of EV industry, including providing for charging
infrastructure, research and development of EV technologies and push towards greater
indigenization.

In this spirit, support to fully electric buses have been added to the scheme to support
electrification of public transport.

QUESTION 61. MTU4OTEyK1IuUmFndWwrcm9icmFndWxAZ21haWwuY29tK1FVRVNUSU9OIDYw


Consider the following statements about National Service Scheme (NSS):

1. It provides opportunities to school and University students to take part in various


government led community service activities & programmes.

2. The sole aim of the NSS is to provide hands on experience to young students in delivering
community service.

3. The motto of National Service Scheme is ‘Not Me But You’.

Which of the above statements are correct?

IASbaba
Web: http://ilp.iasbaba.com/ Score:
Email: ilp@iasbaba.com 0.00 / 198
Page 49
2019 - Test 32 -
Exam Title :
Government...
Email :
Contact :

a) 1 and 2
b) 2 and 3
c) 1 and 3
d) All of the above
Correct Answer: D
Your Answer: Unanswered
Explanation

Solution (d)

National Service Scheme (NSS)

The National Service Scheme (NSS) is a Central Sector Scheme of Government of India,
Ministry of Youth Affairs & Sports. It provides opportunity to the student youth of 11th & 12th
Class of schools at +2 Board level and student youth of Technical Institution, Graduate & Post
Graduate at colleges and University level of India to take part in various government led
community service activities & programmes.The sole aim of the NSS is to provide hands on
experience to young students in delivering community service. Since inception of the NSS in
the year 1969, the number of students strength increased from 40,000 to over 3.8 million up to
the end of March 2018 students in various universities, colleges and Institutions of higher
learning have volunteered to take part in various community service programmes.

· All the youth volunteers who opt to serve the nation through the NSS led community service
wear the NSS badge with pride and a sense of responsibility towards helping needy.

· The Konark wheel in the NSS badge having 8 bars signifies the 24 hours of a the day,
reminding the wearer to be ready for the service of the nation round the clock i.e. for 24 hours.

· Red colour in the badge signifies energy and spirit displayed by the NSS volunteers.

· The Blue colour signifies the cosmos of which the NSS is a tiny part, ready to contribute its
share for the welfare of the mankind.

The motto of National Service Scheme is NOT ME BUT YOU

IASbaba
Web: http://ilp.iasbaba.com/ Score:
Email: ilp@iasbaba.com 0.00 / 198
Page 50
2019 - Test 32 -
Exam Title :
Government...
Email :
Contact :
QUESTION 62. MTU4OTEyK1IuUmFndWwrcm9icmFndWxAZ21haWwuY29tK1FVRVNUSU9OIDYx
Ministry of Youth Affairs and Sports has launched a Mission XI Million to popularize which of
the following sport?

a) Cricket
b) Kho-Kho
c) Wrestling
d) Football
Correct Answer: D
Your Answer: Unanswered
Explanation

Solution (d)

Mission XI Million is a joint programme, of this Ministry, All India Football Federation (AIFF)
and Federation International de Football Association (FIFA) to popularise football across the
country. The programme targets to reach 11 Million boys and girls across the country for
promotion of football.

Government of India has allocated around Rs.12.55 crores towards the said programme and an
equal amount will be spent by AIFF/ FIFA. Around 6 Million children have already been covered
under the said programme. The allocated resources are sufficient to cover 11 Million students,
parents, and coaches as part of Mission XI Million Programme to popularize football in the
country.

QUESTION 63. MTU4OTEyK1IuUmFndWwrcm9icmFndWxAZ21haWwuY29tK1FVRVNUSU9OIDYy


Consider the following statements regarding ‘Divyang Sarathi’ App:

1. It aims at providing all relevant information pertaining to the Department of Empowerment


of Persons with Disabilities (DEPwD).

2. The unique feature is its lip reading software which can recognized the word through camera
and convert them into text.

3. The mobile application is compliant with the principles of UN Convention on the Rights of
Persons with Disabilities (UNCRPD).

Which of the above statements are correct?

a) 1 and 2
b) 2 and 3
c) 1 and 3
d) All of the above
Correct Answer: C
Your Answer: Unanswered
Explanation

Solution (c)

IASbaba
Web: http://ilp.iasbaba.com/ Score:
Email: ilp@iasbaba.com 0.00 / 198
Page 51
2019 - Test 32 -
Exam Title :
Government...
Email :
Contact :

‘Divyang Sarathi’ —The accessible and comprehensive mobile application for easy
dissemination of information to ‘Divyangjans’. This mobile application aims at providing all
relevant information pertaining to the Department of Empowerment of Persons with Disabilities
(Divyangjan) including its various Acts, Rules and Regulations, Guidelines, Schemes, outreach
institutions, employment opportunities, and the disability market in an accessible format.

The unique features of the mobile application ‘Divyang Saathi’ are its audio notes (text-to-voice
conversion software) embedded in the application which converts the written information into
an audio file as well as the adjustable font size which can be altered as per the user’s
requirement. The mobile application will certainly have a wide outreach as it has been
developed bilingually i.e., the information is available in Hindi as well as English. The
application has been so developed that once downloaded, it can be accessed by any Android
smartphone User who may or may not has access to the internet. The mobile app is available
for download on Google Playstore.

QUESTION 64. MTU4OTEyK1IuUmFndWwrcm9icmFndWxAZ21haWwuY29tK1FVRVNUSU9OIDYz


Which of the following statements are correct regarding ‘Deen Dayal Antyodaya Yojna –
National Urban Livelihood Mission (DAY – NULM)?

1. It was launched by the Ministry of Housing and Urban Poverty Alleviation (MHUPA)

2. The primary target of NULM is the urban poor, including the urban homeless.

3. Funding will be shared between the Centre and the States in the ratio of 75:25 (Except for
NE states)

Select the code from following:

a) 1 and 2
b) 2 and 3
c) 1 and 3
d) All of the above
Correct Answer: D
Your Answer: Unanswered
Explanation

Solution (d)

National Urban Livelihoods Mission (NULM) was launched by the Ministry of Housing and
Urban Poverty Alleviation (MHUPA), Government of India in 23rd September, 2013 by replacing
the existing Swarna Jayanti Shahari Rozgar Yojana (SJSRY).The NULM will focus on organizing
urban poor in their strong grassroots level institutions, creating opportunities for skill
development leading to market-based employment and helping them to set up self-employment
venture by ensuring easy access to credit. The Mission is aimed at providing shelter equipped
with essential services to the urban homeless in a phased manner. In addition, the Mission
would also address livelihood concerns of the urban street vendors.

The Main Features of NULM are:

IASbaba
Web: http://ilp.iasbaba.com/ Score:
Email: ilp@iasbaba.com 0.00 / 198
Page 52
2019 - Test 32 -
Exam Title :
Government...
Email :
Contact :

Coverage: In the 12th Five Year Plan, NULM will be implemented in all districts headquarter
towns(irrespective of population) and all other towns with population of 1 lakh or more as per
Census 2011. At present 790 cities are under NULM. However, other towns may be allowed in
exceptional cases on request of the States.
Target Population: The primary target of NULM is the urban poor, including the urban
homeless.
Sharing of funding: Funding will be shared between the Centre and the States in the ratio of
75:25. For North Eastern and Special Category States (Arunachal Pradesh, Assam, Manipur,
Meghalaya, Mizoram, Nagaland, Sikkim, Tripura, Jammu & Kashmir, Himachal Pradesh and
Uttarakhand), this ratio will be 90:10.

QUESTION 65. MTU4OTEyK1IuUmFndWwrcm9icmFndWxAZ21haWwuY29tK1FVRVNUSU9OIDY0


Consider the following statements regarding ‘Skill for Life, Save a Life’ initiative:

1. It has been launched by Ministry of Skill development and entrepreneurship.

2. It aims to upscale the quantity and quality of trained professionals in the healthcare system.

Which of the above statements is/are correct regarding this initiative?

a) 1 only
b) 2 only
c) Both 1 and 2
d) Neither 1 nor 2
Correct Answer: B
Your Answer: Unanswered
Explanation

Solution (b)

‘Skills for Life, Save a Life’ Initiative’ aims to upscale the quantity and quality of trained
professionals in the healthcare system. Under this initiative various courses are planned to be

IASbaba
Web: http://ilp.iasbaba.com/ Score:
Email: ilp@iasbaba.com 0.00 / 198
Page 53
2019 - Test 32 -
Exam Title :
Government...
Email :
Contact :
initiated targeting specific competencies for healthcare professionals as well as for general
public.

The Initiative has been launched by Ministry of Health and Family Welfare.

This is part of a larger plan and program for ensuring Universal Health Coverage. This program
will provide trained and skilled people by broadening the base to include the community.
Through such programs, we create a mass of ‘first responder’ who complements the specialists/
experts to fill vacuum of adequate trained professionals.

QUESTION 66. MTU4OTEyK1IuUmFndWwrcm9icmFndWxAZ21haWwuY29tK1FVRVNUSU9OIDY1


‘Vatsalya – Maatri Amrit Kosh’, a National Human Milk Bank and Lactation Counselling Centre
have been inaugurated at the Lady Hardinge Medical College (LHMC) in New Delhi. Which of
the following statements is/are correct regarding this?

1. It has been established in collaboration with the Denmark government

2. It will collect, pasteurize, test and safely store milk donated by lactating mothers and make it
available for infants in need.

3. The centre will act as teaching, training and demonstration site for other milk banks that will
be established under the Ministry of Health and Family Welfare.

Select the code from following:

a) 1 and 2
b) 2 and 3
c) 1 and 3
d) All of the above
Correct Answer: B
Your Answer: Unanswered
Explanation

Solution (b)

‘Vatsalya – Maatri Amrit Kosh’, a National Human Milk Bank and Lactation Counselling Centre
has been opened at the Lady Hardinge Medical College (LHMC) in New Delhi. This centre
would be the largest public sector human milk bank and lactation counseling centre available in
North

• Salient Highlights “Vatsalya – Maatri Amrit Kosh” has been established in collaboration
with the Norwegian government, Oslo University and Norway India Partnership Initiative
(NIPI).
• Vatsalya – Maatri Amrit Kosh will collect, pasteurize, test and safely store milk donated by
lactating mothers and make it available for infants in need. With the establishment of this
donor human milk bank, all newborns in and around Delhi will have access to life saving
human milk irrespective of the circumstances of their birth.
• The centre will protect, promote and support breastfeeding by providing lactation support
to mothers with the help of dedicated lactation counsellors.
• In this regard, the Ministry of Health and Family Welfare has also launched Mothers
Absolute Affection (MAA) programme to create awareness regarding breastfeeding among
mothers for enhancing the child’s immunity.

IASbaba
Web: http://ilp.iasbaba.com/ Score:
Email: ilp@iasbaba.com 0.00 / 198
Page 54
2019 - Test 32 -
Exam Title :
Government...
Email :
Contact :
• The facility will act as a dedicated centre to support breastfeeding and improve infant
survival. The centre will act as teaching, training and demonstration site for other milk
banks that will be established under the Ministry of Health and Family Welfare.

QUESTION 67. MTU4OTEyK1IuUmFndWwrcm9icmFndWxAZ21haWwuY29tK1FVRVNUSU9OIDY2


Which of the following is the aim of Mission ‘Parivar Vikas’?

a) To increase the doubling rate of population of Parsi community


b) To increase the Total Fertility Rate(TFR) in North Eastern States by providing incentives
like free health and education to local children.
c) To decrease TFR in high population district by providing awareness to contraceptives and
family planning techniques
d) To increase the living standard of below poverty line families living in urban sprawls.
Correct Answer: C
Your Answer: Unanswered
Explanation

Solution (c)

The Ministry of Health and Family Welfare has launched “Mission Parivar Vikas” in 145 high
focus districts having the highest total fertility rates in the country. These 145 districts are in
the seven high focus, high TFR states of Uttar Pradesh, Bihar, Rajasthan, Madhya Pradesh,
Chhattisgarh, Jharkhand and Assam that constitute 44% of the country’s population. The main
objective of ‘Mission Parivas Vikas’ will be to accelerate access to high quality family planning
choices based on information, reliable services and supplies within a rights-based framework.

These 145 districts have been identified based on total fertility rate and service delivery
(PPIUCD and Sterilization performance) for immediate, special and accelerated efforts to reach
the replacement level fertility goals of 2.1 by 2025. Recent data suggests that these 145
districts have TFR of more than/equal to 3.0 (56% of the 261 districts in the 7 HFS) and are
home to 28% of India’s population (about 33 Crores). However, only 22% of India’s protected
couples and 40% of India’s couples with unmet need reside in these districts. These districts
also have a substantial impact on maternal and child health indicators as about 25-30% of
maternal deaths and 50% of infant deaths occur in these districts Moreover, 115 of these
districts (79%) have high percentage of adolescent mothers.

The key strategic focus of this initiative will be on improving access to contraceptives through
delivering assured services, dovetailing with new promotional schemes, ensuring commodity
security, building capacity (service providers), creating an enabling environment along with
close monitoring and implementation.

The Mission will be implemented in all the 145 districts at one go.

QUESTION 68. MTU4OTEyK1IuUmFndWwrcm9icmFndWxAZ21haWwuY29tK1FVRVNUSU9OIDY3


The Union Ministry of Health and Family Welfare launched Mission SAMPARK targeting people
suffering from

a) HIV

IASbaba
Web: http://ilp.iasbaba.com/ Score:
Email: ilp@iasbaba.com 0.00 / 198
Page 55
2019 - Test 32 -
Exam Title :
Government...
Email :
Contact :

b) Tuberculosis
c) Vector Borne diseases
d) Diarrhea
Correct Answer: A
Your Answer: Unanswered
Explanation

Solution (a)

The Union Ministry of Health and Family Welfare launched the National Strategic Plan 2017-24
aimed at eradicating HIV/AIDS by 2030.

Mission SAMPARK was also launched to trace those who are Left to Follow Up and are to be
brought under Antiretroviral therapy (ART) services.

90:90:90 Strategy:

It is a new HIV treatmentnarrative of UNAIDS programmewhich has set targets of

· 90% of all people living with HIV will know their HIV status (90% diagnosed),

· 90% of all people with diagnosed HIV infection will receive sustained antiretroviral therapy
(90% on HIV treatment) and

· 90% of all people receiving antiretroviral therapy will have viral suppression (90%
suppressed).

QUESTION 69. MTU4OTEyK1IuUmFndWwrcm9icmFndWxAZ21haWwuY29tK1FVRVNUSU9OIDY4


The Department of Justice has launched ‘Pro Bono Legal Services’ initiative. Which of the
following statements regarding the initiative are correct?

1. It is a web based platform, through which interested lawyers can register themselves to
volunteer pro bono services for the underprivileged litigants, who are unable to afford it.

2. It will also provide a platform for all litigants to apply for legal aid and advice from pro bono
lawyers.

Select the code from following:

a) 1 only
b) 2 only
c) Both 1 and 2
d) Neither 1 nor 2
Correct Answer: A
Your Answer: Unanswered
Explanation

Solution (a)

Pro bono legal Services

IASbaba
Web: http://ilp.iasbaba.com/ Score:
Email: ilp@iasbaba.com 0.00 / 198
Page 56
2019 - Test 32 -
Exam Title :
Government...
Email :
Contact :

The ‘Pro bono legal services’ initiative is a web based platform, through which interested
lawyers can register themselves to volunteer pro bono services for the underprivileged
litigants, who are unable to afford it. The Department of Justice has launched the online
application for this initiative on its website doj.gov.in. Through this online portal, litigants from
marginalised communities (including members of scheduled castes and scheduled tribes,
women, children, senior citizens, persons with low income and persons with disabilities) can
also apply for legal aid and advice from the pro bono lawyers.

QUESTION 70. MTU4OTEyK1IuUmFndWwrcm9icmFndWxAZ21haWwuY29tK1FVRVNUSU9OIDY5


Consider the following statements regarding COMMIT program:

1. It is an induction training program for State Government officials.

2. The programme is aimed to improve the public service delivery mechanism and provide
citizen centric administration through capacity building of officials who interact with the
citizens on day-to-day basis.

3. The programme will be implemented through State Administrative Training Institutes (ATIs).

Which of the above statements are correct?

a) 1 and 2
b) 2 and 3
c) 1 and 3
d) All of the above
Correct Answer: D
Your Answer: Unanswered
Explanation

Solution (d)

Ministry of Personnel, Public Grievances & Pensions has launched a training programme Comp
rehensive Online Modified Modules on Induction Training (COMMIT) for State
Government officials. The objective of this training programme is to improve the public service
delivery mechanism and provide citizen centric administration through capacity building of
officials who interact with the citizens on day-to-day basis.

The programme will supplement the earlier training programme and the advantage of COMMIT
is that it is cost effective and has the potential to cover about 3.3 lakh officials annually,
compared to 10,000 officials under existing 12-Day Induction Training Programme (ITP).

The COMMIT programme has been developed by DoPT in collaboration with United Nations
Development Programme (UNDP).

The programme will cover approximately 74,000 State Government officials in the financial year
2017-18. It will be of 28 hours duration which will include e-Modules for 20 hours and face-to-
face training for 8 hours. The 20 hours e-training would be imparted through specifically
developed 12 Generic and 3 Domain specific e-Modules. The modules on soft skills will be
delivered as e-Modules & through face-to-face training and the domain modules will be covered
through e-Modules only. The programme will be implemented through State Administrative
Training Institutes (ATIs).

IASbaba
Web: http://ilp.iasbaba.com/ Score:
Email: ilp@iasbaba.com 0.00 / 198
Page 57
2019 - Test 32 -
Exam Title :
Government...
Email :
Contact :

QUESTION 71. MTU4OTEyK1IuUmFndWwrcm9icmFndWxAZ21haWwuY29tK1FVRVNUSU9OIDcw


Which of the following statements are correct regarding ‘Deendayal Upadyay Gram Jyoti Yojana
(DDUGJY)’?

1. It aims to provide LED based street lighting in rural areas.

2. It aims to strengthen sub-transmission and distribution networkto prevent power losses.

3. It focuses on feeder separation for rural households and agricultural purpose.

Select the code from following:

a) 1 and 2
b) 2 and 3
c) 1 and 3
d) All of the above
Correct Answer: B
Your Answer: Unanswered
Explanation

Solution (b)

Deen Dayal Upadhyaya Gram Jyoti Yojana

Ministry of Power, Government of India has launched Deen Dayal Upadhyaya Gram Jyoti Yojana
for rural areas having following objectives:

· To provide electrification to all villages

· Feeder separation to ensure sufficient power to farmers and regular supply to other
consumers

· Improvement of Sub-transmission and distribution network to improve the quality and


reliability of the supply

· Metering to reduce the losses

QUESTION 72. MTU4OTEyK1IuUmFndWwrcm9icmFndWxAZ21haWwuY29tK1FVRVNUSU9OIDcx


Ministry of Rural development have launched ‘Aajeevika Gramin Express Yojana’. The aim of the
scheme is

a) To link all the rural areas across India with the urban areas through express trains to ease
the transport of freight.
b) To provide an alternative source of livelihoods to members of Self Help Groups (SHGs) by
facilitating them to operate public transport services in backward rural areas.
c) To provide discounted tickets to rural poor in sleeper class of express trains
d) To develop the areas around a railway station in rural areas by creating civic amenities
Correct Answer: B

IASbaba
Web: http://ilp.iasbaba.com/ Score:
Email: ilp@iasbaba.com 0.00 / 198
Page 58
2019 - Test 32 -
Exam Title :
Government...
Email :
Contact :
Your Answer: Unanswered
Explanation

Solution (b)

The Government of India has launched a sub-scheme named “Aajeevika Grameen Express
Yojana (AGEY)” as part of the Deendayal Antyodaya Yojana – National Rural Livelihoods Mission
(DAY-NRLM). The Self Help Groups under DAY-NRLM will operate road transport service in
backward areas. This will help to provide safe, affordable and community monitored rural
transport services to connect remote villages with key services and amenities (such as access to
markets, education and health) for the overall economic development of backward rural areas.
This will also provide an additional avenue of livelihood for SHGs.

The Community Investment Fund (CIF) provided to Community Based Organization (CBOs)
under DAY-NRLM will be utilized to support the SHG members in this new livelihoods initiative.
The beneficiary SHG member will be provided an interest free loan by the CBO from its
Community Investment Fund upto Rs.6.50 lakh for purchase of the vehicle. Alternative, CBO
will own the vehicle and lease it to an SHG member to operate the vehicle and pay lease rental
to the CBO

QUESTION 73. MTU4OTEyK1IuUmFndWwrcm9icmFndWxAZ21haWwuY29tK1FVRVNUSU9OIDcy


‘Hill Area Development program’ has been launched to develop the Hilly areas of

a) Uttarakhand
b) Western Ghats
c) North Eastern states
d) Madhya Pradesh and Chhattisgarh
Correct Answer: C
Your Answer: Unanswered
Explanation

Soution (c)

Ministry for Development of North-East Region has launched ‘Hill Area Development Program’.

The hilly areas of Manipur, Tripura and Assam have a distinct geo-physical entity and are
lagging in socio-economic development. As a result of peculiar topography there is a wide gap
between the hill and valley districts in terms of infrastructure, quality of roads, health and
education etc. The Hill Development Programme is inspired with a serious research and
deliberation of all these factors.

The program was initiated on pilot basis in Hilly districts of Manipur.

The Government will adopt two-fold approach:

· To ensure equitable developmentof every area, every section of society and every tribe living
in the North Eastern Region

· To bring the eight North Eastern Region States at par with the more developed States in the
rest of India

IASbaba
Web: http://ilp.iasbaba.com/ Score:
Email: ilp@iasbaba.com 0.00 / 198
Page 59
2019 - Test 32 -
Exam Title :
Government...
Email :
Contact :

QUESTION 74. MTU4OTEyK1IuUmFndWwrcm9icmFndWxAZ21haWwuY29tK1FVRVNUSU9OIDcz


Ministry of Science and Technology has launched Innovate in India (i3) Mission. Which of the
following statements is/are correct regarding the mission?

1. It aims at promoting research in cutting edge technology in the fields of Defence, Robotics,
nano sciences, biopharma and IT and communication.

2. This is the first ever Industry-Academia mission to accelerate biopharmaceutical


development in India.

Select the code from following:

a) 1 only
b) 2 only
c) Both 1 and 2
d) Neither 1 nor 2
Correct Answer: B
Your Answer: Unanswered
Explanation

Solution (b)

Innovate in India (i3) i.e. the National BiopharmaMission

With an aim of creating a globally competitive biopharmaceutical industry that can addresses
the country’s major concerns around barriers to affordable healthcare; innovate in India (i3)
programme was launched.

The $250 million programme of the Department of Biotechnology (DBT), Ministry of Science &
Technology, is a first- of-its-kind mission that brings together industry and academia to promote
entrepreneurship and indigenous manufacturing in the biopharmaceutical sector. This flagship
programme of the GOI in collaboration with World Bank, will be implemented by Biotechnology
Industry Research Assistance Council (BIRAC), a Public Sector Enterprise, set up by DBT.

QUESTION 75. MTU4OTEyK1IuUmFndWwrcm9icmFndWxAZ21haWwuY29tK1FVRVNUSU9OIDc0


Consider the following statements with respect to ‘Rio Declaration on Environment and
Development’

1. It proclaims indigenous people have a vital role in environmental management

2. It includes formulations of the precautionary principle and of the polluter pays principle

Select the correct statements

a) 1 Only
b) 2 Only
c) Both 1 and 2
d) Neither 1 nor 2

IASbaba
Web: http://ilp.iasbaba.com/ Score:
Email: ilp@iasbaba.com 0.00 / 198
Page 60
2019 - Test 32 -
Exam Title :
Government...
Email :
Contact :
Correct Answer: C
Your Answer: Unanswered
Explanation

Solution (c)

Both the statements are correct.

The Rio Declaration on Environment and Development, often shortened to Rio Declaration, was
a short document produced at the 1992 United Nations "Conference on Environment and
Development" (UNCED), informally known as the Earth Summit. The Rio Declaration consisted
of 27 principles intended to guide countries in future sustainable development. It was signed by
over 170 countries.

Indigenous people and their communities, and other local communities, have a vital role in
environmental management and development because of their knowledge and traditional
practices. States should recognize and duly support their identity, culture and interests and
enable their effective participation in the achievement of sustainable development.

In order to protect the environment, the precautionary approach shall be widely applied by
States according to their capabilities. Where there are threats of serious or irreversible
damage, lack of full scientific certainty shall not be used as a reason for postponing cost-
effective measures to prevent environmental degradation.

National authorities should endeavour to promote the internalization of environmental costs


and the use of economic instruments, taking into account the approach that the polluter should,
in principle, bear the cost of pollution, with due regard to the public interest and without
distorting international trade and investment.

QUESTION 76. MTU4OTEyK1IuUmFndWwrcm9icmFndWxAZ21haWwuY29tK1FVRVNUSU9OIDc1


India is a party/signatory to which of the following?

1. Universal Declaration of Human Rights (UDHR)

2. Declaration on the Rights of Indigenous Peoples (UNDRIP)

3. Indigenous and Tribal Peoples Convention, 1989

Select the correct code:

a) 1 and 2
b) 2 and 3
c) 1 and 3
d) All of the above
Correct Answer: A
Your Answer: Unanswered
Explanation

Solution (a)

India is not a signatory to the ‘Indigenous and Tribal Peoples Convention, 1989’, but is to the
Indigenous and Tribal Populations Convention, 1957.

IASbaba
Web: http://ilp.iasbaba.com/ Score:
Email: ilp@iasbaba.com 0.00 / 198
Page 61
2019 - Test 32 -
Exam Title :
Government...
Email :
Contact :

India is a signatory to the United Nations Declaration on the Rights of Indigenous Peoples
(UNDRIP or DOTROIP) and Universal Declaration of Human Rights (UDHR).

QUESTION 77. MTU4OTEyK1IuUmFndWwrcm9icmFndWxAZ21haWwuY29tK1FVRVNUSU9OIDc2


Consider the following statements with respect to ‘Geneva Conventions’

IASbaba
Web: http://ilp.iasbaba.com/ Score:
Email: ilp@iasbaba.com 0.00 / 198
Page 62
2019 - Test 32 -
Exam Title :
Government...
Email :
Contact :

1. The treatment of prisoners of war is dealt with by the Third Convention or treaty.

2. The first Convention deals with the treatment of civilians and their protection during
wartime.

Select the correct statements

a) 1 Only
b) 2 Only
c) Both 1 and 2
d) Neither 1 nor 2
Correct Answer: A
Your Answer: Unanswered
Explanation

Solution (a)

Geneva Conventions

· They are the rules protecting prisoners of war (POWs).

· They were first detailed in the 1929 Geneva Convention and later amended in the third 1949
Geneva Convention following the lessons of World War II.

· There are four Geneva Conventions.

· The third Geneva Convention defines and details who can be considered a POW and how he/
she must be treated.

Third Geneva Convention

· The third Geneva Convention provides a wide range of protection for prisoners of war. It
defines their rights and sets down detailed rules for their treatment and eventual release. The
status of POW only applies in international armed conflict.

· POWs are usually members of the armed forces of one of the parties to a conflict who fall into
the hands of the adverse party. The third 1949 Geneva Convention also classifies other
categories of persons who have the right to POW status or may be treated as POWs.

· POWs cannot be prosecuted for taking a direct part in hostilities. Their detention is not a form
of punishment, but only aims to prevent further participation in the conflict. They must be
released and repatriated without delay after the end of hostilities. The detaining power may
prosecute them for possible war crimes, but not for acts of violence that are lawful under
international humanitarian law.

· "POWs must be treated humanely in all circumstances. They are protected against any act of
violence, as well as against intimidation, insults, and public curiosity,"

In the case of armed conflict not of an international character occurring in the territory of one
of the High Contracting Parties, each Party to the conflict shall be bound to apply, as a
minimum, the following provisions:

(1) Persons taking no active part in the hostilities, including members of armed forces who have
laid down their arms and those placed hors de combat by sickness, wounds, detention, or any

IASbaba
Web: http://ilp.iasbaba.com/ Score:
Email: ilp@iasbaba.com 0.00 / 198
Page 63
2019 - Test 32 -
Exam Title :
Government...
Email :
Contact :
other cause, shall in all circumstances be treated humanely, without any adverse distinction
founded on race, colour, religion or faith, sex, birth or wealth, or any other similar criteria.

To this end the following acts are and shall remain prohibited at any time and in any place
whatsoever with respect to the above-mentioned persons:

· Violence to life and person, in particular murder of all kinds, mutilation, cruel treatment and
torture;

· Taking of hostages;

· Outrages upon personal dignity, in particular, humiliating and degrading treatment;

· The passing of sentences and the carrying out of executions without previous judgment
pronounced by a regularly constituted court affording all the judicial guarantees which are
recognized as indispensable by civilized peoples.

(2) The wounded and sick shall be collected and cared for.

· An impartial humanitarian body, such as the International Committee of the Red Cross, may
offer its services to the Parties to the conflict.

· The Parties to the conflict should further endeavour to bring into force, by means of special
agreements, all or part of the other provisions of the present Convention.

· The application of preceding provisions shall not affect the legal status of the Parties to the
conflict.

The Geneva Conventions have a system of “Protecting Powers” who ensure that the provisions
of the conventions are being followed by the parties in a conflict. In theory, each side must
designate states that are not party to the conflict as their “Protecting Powers”. In practice, the
International Committee of the Red Cross usually plays this role.

Conventions

· The First Geneva Convention "for the Amelioration of the Condition of the Wounded and Sick
in Armed Forces in the Field" (first adopted in 1864, revised in 1906, 1929 and finally 1949);

· The Second Geneva Convention "for the Amelioration of the Condition of Wounded, Sick and
Shipwrecked Members of Armed Forces at Sea" (first adopted in 1949, successor of the Hague
Convention (X) 1907);

· The Third Geneva Convention "relative to the Treatment of Prisoners of War" (first adopted in
1929, last revision in 1949);

· The Fourth Geneva Convention "relative to the Protection of Civilian Persons in Time of War"
(first adopted in 1949, based on parts of the Hague Convention (II) of 1899 and Hague
Convention (IV) 1907).

The 1949 conventions have been modified with three amendment protocols:

· Protocol I (1977) relating to the Protection of Victims of International Armed Conflicts

· Protocol II (1977) relating to the Protection of Victims of Non-International Armed Conflicts

· Protocol III (2005) relating to the Adoption of an Additional Distinctive Emblem

IASbaba
Web: http://ilp.iasbaba.com/ Score:
Email: ilp@iasbaba.com 0.00 / 198
Page 64
2019 - Test 32 -
Exam Title :
Government...
Email :
Contact :

Note – India is party to the four conventions but not the additional protocols

QUESTION 78. MTU4OTEyK1IuUmFndWwrcm9icmFndWxAZ21haWwuY29tK1FVRVNUSU9OIDc3


‘TORCH Infections’ is associated with

1. Sexually Transmitted Diseases

2. Foetal abnormalities

Select the correct code:

a) 1 Only
b) 2 Only
c) Both 1 and 2
d) Neither 1 nor 2
Correct Answer: C
Your Answer: Unanswered
Explanation

Solution (c)

TORCH Infections

· TORCH, which includes Toxoplasmosis, Other (syphilis, varicella-zoster, parvovirus B19),


Rubella, Cytomegalovirus (CMV), and Herpes infections, are some of the most common
infections associated with congenital anomalies.

· Most of the TORCH infections cause mild maternal morbidity, but have serious foetal
consequences, and treatment of maternal infection frequently has no impact on foetal outcome.

· TORCH infections are known to cause foetal abnormalities, including microcephaly, among
newborns.

Note – CMV, Herpes are STDs

QUESTION 79. MTU4OTEyK1IuUmFndWwrcm9icmFndWxAZ21haWwuY29tK1FVRVNUSU9OIDc4


Consider the following statement with respect to ‘National Pharmaceutical Pricing Authority
(NPPA)’?

1. NPPA fixes prices of drugs placed in the National List of Essential Medicines (NLEM) under
Schedule-I of the Drugs (Prices Control) Order, 2013

2. Non-scheduled drugs are allowed an increase of up to 10% in prices every year, which is
monitored by the NPPA.

Select the INCORRECT statements

a) 1 Only

IASbaba
Web: http://ilp.iasbaba.com/ Score:
Email: ilp@iasbaba.com 0.00 / 198
Page 65
2019 - Test 32 -
Exam Title :
Government...
Email :
Contact :

b) 2 Only
c) Both 1 and 2
d) Neither 1 nor 2
Correct Answer: D
Your Answer: Unanswered
Explanation

Solution (d)

Both the statements are correct.

News: National Pharmaceutical Pricing Authority (NPPA) has invoked extraordinary powers in
public interest, under Para 19 of the Drugs (Prices Control) Order, 2013 to bring 42 non-
scheduled anti-cancer drugs under price control through trade margin rationalisation , capping
trade margin at 30%, which would reduce their retail prices by up to 85%.

NPPA currently fixes prices of drugs placed in the National List of Essential Medicines (NLEM)
under Schedule-I of the DPCO

Non-scheduled drugs are allowed an increase of up to 10% in prices every year, which is
monitored by the NPPA.

QUESTION 80. MTU4OTEyK1IuUmFndWwrcm9icmFndWxAZ21haWwuY29tK1FVRVNUSU9OIDc5


Consider the following statements with respect to ‘Enzyme-linked immunosorbent Assay’

1. It is a tool for determining serum antibody concentrations

2. It is used for detection of Mycobacterium antibodies in tuberculosis

3. It is used for detection of HIV antibodies in blood samples

Select the correct statements

a) 1 and 3
b) 3 Only
c) 1, 2 and 3
d) 1 and 2
Correct Answer: C
Your Answer: Unanswered
Explanation

Solution (c)

ELISA

· ELISA (enzyme-linked immunosorbent assay) is a plate-based assay technique designed for


detecting and quantifying substances such as peptides, proteins, antibodies and hormones.

· Other names, such as enzyme immunoassay (EIA), are also used to describe the same
technology.

IASbaba
Web: http://ilp.iasbaba.com/ Score:
Email: ilp@iasbaba.com 0.00 / 198
Page 66
2019 - Test 32 -
Exam Title :
Government...
Email :
Contact :

· In an ELISA, an antigen must be immobilized on a solid surface and then complexed with an
antibody that is linked to an enzyme.

· Detection is accomplished by assessing the conjugated enzyme activity via incubation with a
substrate to produce a measureable product. The most crucial element of the detection strategy
is a highly specific antibody-antigen interaction.

Applications

· Because the ELISA can be performed to evaluate either the presence of antigen or the
presence of antibody in a sample, it is a useful tool for determining serum antibody
concentrations (such as with the HIV test or West Nile virus).

· It has also found applications in the food industry in detecting potential food allergens, such as
milk, peanuts, walnuts, almonds, and eggs and as serological blood test for coeliac disease.

· ELISA can also be used in toxicology as a rapid presumptive screen for certain classes of
drugs.

· Dr Dennis E Bidwell and Alister Voller created the ELISA test to detect various kind of
diseases, such as dengue, malaria, Chagas disease, Johne's disease, and others.

· ELISA tests also are used as in in vitro diagnostics in medical laboratories.

The other uses of ELISA include:

· detection of Mycobacterium antibodies in tuberculosis

· detection of rotavirus in faeces

· detection of hepatitis B markers in serum

· detection of enterotoxin of E. coli in faeces

· detection of HIV antibodies in blood samples

QUESTION 81. MTU4OTEyK1IuUmFndWwrcm9icmFndWxAZ21haWwuY29tK1FVRVNUSU9OIDgw


Consider the following statements with respect to ‘SHREYAS’ Scheme

1. It establishes an ‘earn while you learn’ system into higher education

2. It will be implemented by the Sector Skill Councils (SSCs)

3. It especially caters to the engineering graduates

Select the correct statements

a) 1 Only
b) 1 and 2
c) 2 Only
d) 1 and 3
Correct Answer: B

IASbaba
Web: http://ilp.iasbaba.com/ Score:
Email: ilp@iasbaba.com 0.00 / 198
Page 67
2019 - Test 32 -
Exam Title :
Government...
Email :
Contact :
Your Answer: Unanswered
Explanation

Solution (b)

Scheme for Higher Education Youth in Apprenticeship and Skills (SHREYAS)

· It is launched by the Ministry of Human Resource Development for providing industry


apprenticeship opportunities to the general graduates exiting in April 2019 through the
National Apprenticeship Promotional Scheme (NAPS)

· It aims to enhance the employability of Indian youth by providing ‘on the job work exposure’
and earning of stipend.

· SHREYAS is a programme conceived for students in degree courses, primarily non-technical,


with a view to introduce employable skills into their learning, promote apprenticeship as
integral to education and also amalgamate employment facilitating efforts of the Government
into the education system so that clear pathways towards employment opportunities are
available to students during and after their graduation.

· SHREYAS is a programme basket comprising the initiatives of three Central Ministries, namely
the Ministry of Human Resource Development, Ministry of Skill Development &
Entrepreneurship and the Ministry of Labour& Employment viz the National Apprenticeship
Promotion Scheme (NAPS), the National Career Service (NCS)and introduction of BA/BSc/
BCom (Professional) courses in the higher educational institutions.

· During the apprenticeship period, the student would get a monthly stipend of about Rs. 6,000
per month by the industry, 25% of which would be reimbursed under the NAPS programme. At
the end of the apprenticeship period, there would be test conducted by the Sector Skill Council
concerned and successful students would get skills certificate in addition to their degree
certificate.

· Under the NAPS scheme, Central Government shares 25% of the stipend per month subject to
a maximum of Rs.1500 p.m during the period of the apprenticeship. Apart from that, an
amount upto Rs.7500 will be met towards basic training cost, where needed.

Objectives

· To improve employability of students by introducing employment relevance into the learning


process of the higher education system

· To forge a close functional link between education and industry/service sectors on a


sustainable basis

· To provide skills which are in demand, to the students in a dynamic manner

· To establish an ‘earn while you learn’ system into higher education

· To help business/industry in securing good quality manpower

· To link student community with employment facilitating efforts of the Government

Operations

· The primary scheme will be operated in conjunction with National Apprenticeship Promotion
Scheme (NAPS) which provides for placing of apprentices upto 10% of the total work force in
every business/industry.

IASbaba
Web: http://ilp.iasbaba.com/ Score:
Email: ilp@iasbaba.com 0.00 / 198
Page 68
2019 - Test 32 -
Exam Title :
Government...
Email :
Contact :

· The scheme will be implemented by the Sector Skill Councils (SSCs) , initially the Banking
Finance Insurance Services (BFSI), Retail, Health care, Telecom, Logistics, Media, Management
services, ITeS and Apparel.

· More sectors would be added over time with emerging apprenticeship demand and curriculum
adjustments.

QUESTION 82. MTU4OTEyK1IuUmFndWwrcm9icmFndWxAZ21haWwuY29tK1FVRVNUSU9OIDgx


Which of the following events took place first?

a) Operation Meghdoot
b) Battle of Tithwal
c) Battle of Longewala
d) Battle of Rezang La
Correct Answer: B
Your Answer: Unanswered
Explanation

Solution (b)

Battle of Tithwal (1947), Rezang La(1962), Gangasagar (1971), Longewala (1971), Operation
Trident (1971), Operation Meghdoot (1987)

QUESTION 83. MTU4OTEyK1IuUmFndWwrcm9icmFndWxAZ21haWwuY29tK1FVRVNUSU9OIDgy


‘Phen Wildlife Sanctuary’ is located in

a) Maharashtra
b) Jharkhand
c) Madhya Pradesh
d) Gujarat
Correct Answer: C
Your Answer: Unanswered
Explanation

Solution (c)

Phen Wildlife Sanctuary

· It is a popular buffer zone of Kanha national park.

· It is located near Mukki entrance gate of Kanha tiger reserve.

· It is popularly known as Phen Wildlife Sanctuary and lies in Southern region of Kanha tiger
reserve, close to Madhya Pradesh and Chattisgarh state borders.

IASbaba
Web: http://ilp.iasbaba.com/ Score:
Email: ilp@iasbaba.com 0.00 / 198
Page 69
2019 - Test 32 -
Exam Title :
Government...
Email :
Contact :
QUESTION 84. MTU4OTEyK1IuUmFndWwrcm9icmFndWxAZ21haWwuY29tK1FVRVNUSU9OIDgz
Recently, ‘Chagos Archipelago’ was in news in what context?

a) Sovereignty dispute between Mauritius and the United Kingdom


b) It filed a case against India accusing of not fulfilling its obligations relating to the cessation
of the nuclear arms race at an early date and to nuclear disarmament
c) India’s long-range missile test facility
d) India’s overseas military base in Seychelles to counter China's growing influence over the
Indian Ocean
Correct Answer: A
Your Answer: Unanswered
Explanation

Solution (a)

Chagos Archipelago

News: UK must return the Chagos Islands to Mauritius "as rapidly as possible,":ICJ

About

· They are a group of seven atolls comprising more than 60 individual tropical islands in the
Indian Ocean about 500 kilometres south of the Maldives archipelago.

· This chain of islands is the southernmost archipelago of the Chagos-Laccadive Ridge, a long
submarine mountain range in the Indian Ocean.

· The islands, which are home to US military base Diego Garcia, were separated from the
former British territory of Mauritius during decolonization in 1968.

· The international Court of Justice (ICJ) has ruled that act was illegal under international law.

· The decision by the ICJ is merely advisory.

· The matter of who holds sovereignty over the Islands, located more than 2,000 miles off the
east coast of Africa, will now be debated by the United Nations General Assembly -- which
referred the case to the ICJ despite London's protests.

QUESTION 85. MTU4OTEyK1IuUmFndWwrcm9icmFndWxAZ21haWwuY29tK1FVRVNUSU9OIDg0


Consider the following statements with respect to ‘Global Digital Health Partnership (GDHP)’

1. It was initiated by India

2. It is a collaboration of governments, government agencies and World Health Organisation

Select the correct statements

a) 1 Only
b) 2 Only
c) Both 1 and 2
d) Neither 1 nor 2

IASbaba
Web: http://ilp.iasbaba.com/ Score:
Email: ilp@iasbaba.com 0.00 / 198
Page 70
2019 - Test 32 -
Exam Title :
Government...
Email :
Contact :
Correct Answer: B
Your Answer: Unanswered
Explanation

Solution (b)

Global Digital Health Partnership (GDHP)

· News: India hosts ‘4th Global Digital Health Partnership Summit’

· It is an international collaboration of governments, government agencies and multinational


organisations (WHO) dedicated to improving the health and well-being of their citizens through
the best use of evidence-based digital technologies.

· Governments are making significant investments to harness the power of technology and
foster innovation and public-private partnerships that support high quality, sustainable health
and care for all.

· The GDHP was initiated by the Australian Digital Health Agency

· The GDHP facilitates global collaboration and co-operation in the implementation of digital
health services.

· There are currently 23 countries, territories and the World Health Organization participating
in the GDHP.

· Participants in the GDHP include senior digital health officials from: Argentina, Australia,
Austria, Brazil, Canada, Estonia, Hong Kong SAR, India, Indonesia, Italy, Japan, Kingdom of
Saudi Arabia, New Zealand, Poland, Portugal, Singapore, South Korea, Sweden, Ukraine, the
United Kingdom, the United States, Uruguay and the World Health Organization.

· The GDHP Secretariat is currently supported by the Australian Digital Health Agency, tasked
with improving health outcomes for all Australians through the delivery of digital healthcare
systems and the National Digital Health Strategy for Australia.

QUESTION 86. MTU4OTEyK1IuUmFndWwrcm9icmFndWxAZ21haWwuY29tK1FVRVNUSU9OIDg1


‘Chicxulub crater’ is located in

a) Patagonia
b) Yucatan Peninsula
c) Dead Sea
d) Golan Heights
Correct Answer: B
Your Answer: Unanswered
Explanation

Solution (b)

Chicxulub crater

· It is an impact crater buried underneath the Yucatán Peninsula in Mexico

IASbaba
Web: http://ilp.iasbaba.com/ Score:
Email: ilp@iasbaba.com 0.00 / 198
Page 71
2019 - Test 32 -
Exam Title :
Government...
Email :
Contact :

· The date of the impact coincides precisely with the Cretaceous–Paleogene boundary (K–Pg
boundary), slightly less than 66 million years ago, and a widely accepted theory is that
worldwide climate disruption from the event was the cause of the Cretaceous–Paleogene
extinction event, a mass extinction in which 75% of plant and animal species on Earth became
extinct, including all non-avian dinosaurs.

QUESTION 87. MTU4OTEyK1IuUmFndWwrcm9icmFndWxAZ21haWwuY29tK1FVRVNUSU9OIDg2


‘Amnesty International’ is

a) An agency of the United Nations to help refugees of civil wars


b) A non-governmental voluntary organization to help very poor people
c) An inter-governmental agency to cater to medical emergencies in war-ravaged regions
d) A global Human Rights Organisation
Correct Answer: D
Your Answer: Unanswered
Explanation

Solution (d)

Amnesty International is a London-based non-governmental organization focused on human


rights. The organization says it has more than seven million members and supporters around
the world.

QUESTION 88. MTU4OTEyK1IuUmFndWwrcm9icmFndWxAZ21haWwuY29tK1FVRVNUSU9OIDg3


Countries included in the ‘FATF Grey List’ imply that

a) The country is not doing enough to fight money laundering and terrorist financing
b) The country’s market is not open enough to trade with world
c) The country is accused of severe human rights violation
d) The country needs to be bailed out by the IMF
Correct Answer: A
Your Answer: Unanswered
Explanation

Solution (a)

FATF Grey List

· FATF’s grey list includes countries that are not doing enough to fight money laundering and
terrorist financing.

· It conveys the impression that a country’s financial system is weak and effective measures
aren’t being taken to halt money laundering or financing groups that have been banned for
indulging in terrorist activities.

· Countries placed on these lists see a decrease in foreign investment and foreign companies
hesitate to invest considering the potential ties to terrorist activities.

IASbaba
Web: http://ilp.iasbaba.com/ Score:
Email: ilp@iasbaba.com 0.00 / 198
Page 72
2019 - Test 32 -
Exam Title :
Government...
Email :
Contact :

· Foreign loans from IMF, World Bank or Asian Development Bank etc are hit. It could also
prove hard to raise debts from international markets.

QUESTION 89. MTU4OTEyK1IuUmFndWwrcm9icmFndWxAZ21haWwuY29tK1FVRVNUSU9OIDg4


Consider the following statements with respect to ‘Domain Name System (DNS)’

1. It is the way that internet domain names are located and translated into internet protocol (IP)
addresses.

2. It is maintained by the Google Inc.

Select the correct statements

a) 1 Only
b) 2 Only
c) Both 1 and 2
d) Neither 1 nor 2
Correct Answer: A
Your Answer: Unanswered
Explanation

Solution (a)

Domain Name System (DNS)

News: Government will soon roll out a public DNS, for India aimed at providing a faster and
more secure browsing experience for Internet users in the country, while ensuring that citizens’
data is stored locally.

About

· The main aim of bringing our own public DNS is to ensure availability, particularly for smaller
Interest Service Providers (ISPs) who don’t have credible DNS.

· The roll-out will be executed by the National Informatics Centre

· It is not that users will compulsorily need to shift to India public DNS. A user is free to choose
any DNS

What?

· A DNS is a like a directory for the Internet.

· It is a hierarchical and decentralized naming system for computers, services, or other


resources connected to the Internet or a private network.

· It associates various information with domain names assigned to each of the participating
entities.

· Most prominently, it translates more readily memorized domain names to the numerical IP
addresses needed for locating and identifying computer services and devices with the
underlying network protocols.

IASbaba
Web: http://ilp.iasbaba.com/ Score:
Email: ilp@iasbaba.com 0.00 / 198
Page 73
2019 - Test 32 -
Exam Title :
Government...
Email :
Contact :

· It delegates the responsibility of assigning domain names and mapping those names to
Internet resources by designating authoritative name servers for each domain.

· It helps to convert domain names that are easy for people to remember into IP addresses,
which are used by computers/machines to communicate.

· If the DNS is either slow or fails to work, users will not be able to locate web addresses

QUESTION 90. MTU4OTEyK1IuUmFndWwrcm9icmFndWxAZ21haWwuY29tK1FVRVNUSU9OIDg5


‘Torres Strait’ is located between

a) South Korea and Japan


b) North Korea and China
c) Australia and New Zealand
d) Papua New Guinea and Australia
Correct Answer: D
Your Answer: Unanswered
Explanation

Solution (d)

Torres Strait is a strait which lies between Australia and Papua New Guinea connecting Indian
and Pacific Oceans.

QUESTION 91. MTU4OTEyK1IuUmFndWwrcm9icmFndWxAZ21haWwuY29tK1FVRVNUSU9OIDkw


Consider the following statements with respect to ‘Cay’

1. It is formed when a glacier cuts a U-shaped valley by ice segregation and abrasion of the
surrounding bedrock

2. It is generally found in Arctic Region

Select the correct statements

a) 1 Only
b) 2 Only
c) Both 1 and 2
d) Neither 1 nor 2
Correct Answer: D
Your Answer: Unanswered
Explanation

Solution (d)

Cay

· Also spelled caye or key, is a small, low-elevation, sandy island on the surface of a coral reef.

IASbaba
Web: http://ilp.iasbaba.com/ Score:
Email: ilp@iasbaba.com 0.00 / 198
Page 74
2019 - Test 32 -
Exam Title :
Government...
Email :
Contact :

· Cays occur in tropical environments throughout the Pacific, Atlantic and Indian Oceans
(including in the Caribbean and on the Great Barrier Reef and Belize Barrier Reef).

· A cay forms when ocean currents transport loose sediment across the surface of a reef to a
depositional node, where the current slows or converges with another current, releasing its
sediment load.

· Gradually, layers of deposited sediment build up on the reef surface. Such nodes occur in
windward or leeward areas of reef where surfaces sometimes occur around an emergent
outcrop of old reef or beach rock.

· The island resulting from sediment accumulation is made up almost entirely of biogenic
sediment – the skeletal remains of plants and animals – from the surrounding reef ecosystems.

· If the accumulated sediments are predominantly sand, then the island is called a cay; if they
are predominantly gravel, the island is called a motu.

· Cay sediments are largely composed of calcium carbonate (CaCO3), primarily of aragonite,
calcite, and high magnesium calcite.

· They are produced by myriad plants (e.g., coralline algae, species of the green algae
Halimeda) and animals (e.g., coral, molluscs, foraminifera). Small amounts of silicate sediment
are also contributed by sponges and other creatures.

· Over time, soil and vegetation may develop on a cay surface, assisted by the deposition of sea
bird guano.

QUESTION 92. MTU4OTEyK1IuUmFndWwrcm9icmFndWxAZ21haWwuY29tK1FVRVNUSU9OIDkx


Consider the following statements with respect to ‘Middle East Security Alliance (MESA)’

1. It was launched at the 46th session of the Organisation of Islamic Cooperation’s (OIC)
Council of Foreign Ministers

2. It is brainchild of the Kingdom of Saudi Arabia (KSA)

Select the correct code:

a) 1 Only
b) 2 Only
c) Both 1 and 2
d) Neither 1 nor 2
Correct Answer: D
Your Answer: Unanswered
Explanation

Solution (d)

Middle East Security Alliance (MESA)

· It is a proposed U.S. sponsored alliance of Arab countries on the lines of North Atlantic Treaty
Organisation.

IASbaba
Web: http://ilp.iasbaba.com/ Score:
Email: ilp@iasbaba.com 0.00 / 198
Page 75
2019 - Test 32 -
Exam Title :
Government...
Email :
Contact :

· It is being referred to as the “Arab NATO”.

· Prospective members are Saudi Arabia, Bahrain, Qatar, Kuwait, the United Arab Emirates,
Oman, Jordan, and Egypt.

· Defensive in nature, the alliance’s structure is intended to advance participating nations’


common objectives and address threats to regional stability, security, and prosperity.

QUESTION 93. MTU4OTEyK1IuUmFndWwrcm9icmFndWxAZ21haWwuY29tK1FVRVNUSU9OIDky


Consider the following statements with respect to ‘Mycetoma’

1. It is an inflammatory disease which affects only the feet of humans and animals

2. It is a viral disease

3. The disease is common among barefoot populations

Select the correct statements

a) 1 and 2
b) 2 Only
c) 3 Only
d) 1 and 3
Correct Answer: C
Your Answer: Unanswered
Explanation

Solution (c)

Mycetoma

· It is a chronic, progressively destructive morbid inflammatory disease usually of the foot but
any part of the body can be affected.

· Infection is most probably acquired by traumatic inoculation of certain fungi or bacteria into
the subcutaneous tissue

· Mycetoma was described in the modern literature in 1694 but was first reported in the
mid-19th century in the Indian town of Madura, and hence was initially called Madura foot

· Mycetoma commonly affects young adults, particularly males aged between 20 and 40 years,
mostly in developing countries.

· People of low socioeconomic status and manual workers such as agriculturalists, labourers
and herdsmen are the worst affected.

· The causative organisms of mycetoma are distributed worldwide but are endemic in tropical
and subtropical areas in the ‘Mycetoma belt’, which includes the Bolivarian Republic of
Venezuela, Chad, Ethiopia, India, Mauritania, Mexico, Senegal, Somalia, Sudan and Yemen.

· Transmission occurs when the causative organism enters the body through minor trauma or a
penetrating injury, commonly thorn pricks.

IASbaba
Web: http://ilp.iasbaba.com/ Score:
Email: ilp@iasbaba.com 0.00 / 198
Page 76
2019 - Test 32 -
Exam Title :
Government...
Email :
Contact :

· There is a clear relationship between mycetoma and individuals who walk barefooted and are
manual workers. The disease is common among barefoot populations who live in rural areas in
endemic regions but no person is exempted.

· The treatment depends on the causative organisms for the bacterial; it is a long term
antibiotics combination whereas for fungal type it is combined antifungals drugs and surgery.

QUESTION 94. MTU4OTEyK1IuUmFndWwrcm9icmFndWxAZ21haWwuY29tK1FVRVNUSU9OIDkz


‘TIDE Scheme’ is associated with

a) Tidal Energy
b) Tech Entrepreneurship
c) Vocational Training
d) Educational Loans
Correct Answer: B
Your Answer: Unanswered
Explanation

Solution (b)

Technology Incubation and Development of Entrepreneurs (TIDE 2.0) Scheme

· The Scheme has been devised to promote the momentum into the tech entrepreneurship
landscape.

· The Scheme provides financial and technical support to incubators engaged in supporting ICT
startups using emerging technologies such as IoT, AI, Block-chain, Robotics etc. in seven pre-
identified areas of societal relevance.

QUESTION 95. MTU4OTEyK1IuUmFndWwrcm9icmFndWxAZ21haWwuY29tK1FVRVNUSU9OIDk0


Which of the following country is not part of the ‘South Asian Free Trade Area (SAFTA)’?

a) Maldives
b) Sri Lanka
c) Myanmar
d) Pakistan
Correct Answer: C
Your Answer: Unanswered
Explanation

Solution (c)

South Asian Free Trade Area (SAFTA)

· It is an agreement reached at the 12th SAARC summit in Islamabad, Pakistan.

IASbaba
Web: http://ilp.iasbaba.com/ Score:
Email: ilp@iasbaba.com 0.00 / 198
Page 77
2019 - Test 32 -
Exam Title :
Government...
Email :
Contact :

· It created a free trade area of 2.08 billion people in Afghanistan, Bangladesh, Bhutan, India,
Maldives, Nepal, Pakistan and Sri Lanka

Principles

· overall reciprocity and mutuality of advantages so as to benefit equitably all Contracting


States, taking into account their respective level of economic and industrial development, the
pattern of their external trade, and trade and tariff policies and systems;

· negotiation of tariff reform step by step, improved and extended in successive stages through
periodic reviews;

· recognition of the special needs of the Least Developed Contracting States and agreement on
concrete preferential measures in their favour;

· inclusion of all products, manufactures and commodities in their raw, semi-processed and
processed forms.

QUESTION 96. MTU4OTEyK1IuUmFndWwrcm9icmFndWxAZ21haWwuY29tK1FVRVNUSU9OIDk1


Consider the following statements with respect to ‘Startup Ranking Framework’

1. It is released by NITI Aayog

2. Top three states in the ranking are provided with Rupees 100 crore each to support the start-
ups

Select the correct statements

a) 1 Only
b) 2 Only
c) Both 1 and 2
d) Neither 1 nor 2
Correct Answer: D
Your Answer: Unanswered
Explanation

Solution (d)

Startup Ranking Framework

· Department for Promotion of Industry and Internal Trade (DPIIT) released second edition of
Startup Ranking for 2019.

· The Startup Ranking framework aims to rank the States/UTs for establishing a robust
ecosystem for supporting Startups.

· The framework also encourages States and UTs to identify, learn and replicate good practices
from each other.

· The Ranking Framework 2019 comprises of 7 pillars and 30 action points.

IASbaba
Web: http://ilp.iasbaba.com/ Score:
Email: ilp@iasbaba.com 0.00 / 198
Page 78
2019 - Test 32 -
Exam Title :
Government...
Email :
Contact :

· The pillars will assess States’/UTs efforts across institutional support, simplifying regulations,
easing public procurement, incubation support, seed funding support, venture funding support
and awareness and outreach related activities.

· The ranking exercise aims to evaluate measures taken by States/UTs during the assessment
period from May 1, 2018 to June 30, 2019.

QUESTION 97. MTU4OTEyK1IuUmFndWwrcm9icmFndWxAZ21haWwuY29tK1FVRVNUSU9OIDk2


Consider the following statements with respect to ‘SWAYATT’

1. It brings together the key stakeholders within the Indian entrepreneurial ecosystem to
Government e-Marketplace

2. It aims to develop women entrepreneurship and encourage participation of MSME sector and
Start-ups in public procurement

Select the correct statements

a) 1 Only
b) 2 Only
c) Both 1 and 2
d) Neither 1 nor 2
Correct Answer: C
Your Answer: Unanswered
Explanation

Solution (c)

SWAYATT

· It is an initiative to promote Start-ups, Women and Youth Advantage Through eTransactions on


Government e Marketplace (GeM).

· This will bring together the key stakeholders within the Indian entrepreneurial ecosystem to
Government e-Marketplace the national procurement portal.

· It would further seek to promote inclusiveness by catapulting various categories of sellers and
service providers, take proactive steps to facilitate the training and registrations of such
specific category of manufacturers and sellers, develop women entrepreneurship and
encourage participation of MSME sector and Start-ups in public procurement.

QUESTION 98. MTU4OTEyK1IuUmFndWwrcm9icmFndWxAZ21haWwuY29tK1FVRVNUSU9OIDk3


Which of the following islands is not located in the Andamans?

a) Hutbay Island
b) Minicoy
c) Swaraj Dweep

IASbaba
Web: http://ilp.iasbaba.com/ Score:
Email: ilp@iasbaba.com 0.00 / 198
Page 79
2019 - Test 32 -
Exam Title :
Government...
Email :
Contact :

d) Shaheed Dweep
Correct Answer: B
Your Answer: Unanswered
Explanation

Solution (b)

Minicoy is located in Lakshadweep.

QUESTION 99. MTU4OTEyK1IuUmFndWwrcm9icmFndWxAZ21haWwuY29tK1FVRVNUSU9OIDk4


‘Bramble Cay melomys’ is a species of

a) Bees
b) Rodents
c) Seals
d) Porpoise
Correct Answer: B
Your Answer: Unanswered
Explanation

Solution (b)

Bramble Cay melomys

· It is an extinct species of rodent in the family Muridae and subfamily Murinae.

· It was an endemic species of the isolated Bramble Cay, a vegetated coral cay located at the
northern tip of the Great Barrier Reef.

· Considered the only mammal endemic to the Reef, it became the first documented extinction
of a mammal species due to climate change.

IASbaba
Web: http://ilp.iasbaba.com/ Score:
Email: ilp@iasbaba.com 0.00 / 198
Page 80
2019 - Test 32 -
Exam Title :
Government...
Email :
Contact :
Review in Hindi
QUESTION 1.
SPARC योजना�के�बारे�म���न�न�ल�खत�कथन��पर��वचार�क��जए।�

1. इस�योजना�का�उ�े �य�र�ा��व�नमा�ण�उ�ोग��म��भारत��व�श��शोध�को�बढ़ावा�दे ना�है।�

2. इं�डयन�इं��ट�ूट�ऑफ़�साइंस, ब�गलु��SPARC का��कम�के�काया��वयन�हेतु�रा�ीय�सम�वयक�सं�थान�है।�

3. वे�सभी�भारतीय�सं�थान��ज�ह�ने�इं�डया�र��क��स�(NIRF) के�सव��च�100 अथवा��ेणीवार�सव��च�100 म���थान�बनाया�है, आवेदन�के


पा��ह�।�

उपयु���म��से�कौन�से�कथन�स�य�ह��?

a) केवल�1 और�2
b) केवल�2 और�3
c) केवल�3
d) उपयु���सभी�
Correct Answer: C
Your Answer:
Explanation

Solution (c)

क���य�मानव�संसाधन�मं�ी��काश�जावड़ेकर�ने�‘ SPARC ’ योजना�शु��क�।�

इस�योजना�का�उ�े �य�सामा�जक�एवं��वशु���व�ान��म��भारत��व�श��शोध�को�बढ़ावा�दे ना�है।�‘SPARC’ का�पूण���व�यास�है�- ' �क�म�फॉर


�मोशन�ऑफ़�अकेड�मक�एंड��रसच��कोलैबोरेशन�' । "SPARC �ौ�ो�गक�, �व�ान�एवं�सामा�जक��व�ान��स�हत�फोकस�ए�रयाज�म��उन
संयु��शोध�प�रयोजना��पर��यान�क���त�करेगा�जो�भारत�के��लए��ासं�तक�ह�।�

SPARC योजना�का�उ�े �य�भारतीय�सं�थान��और��व��के�सव��े��सं�थान��के�बीच�शै��क�व�शोध�सहयोग��को�सु�वधाजनक�बना�भारत�के


उ�च��श�ण�सं�थान��म��शोध�पा�र��थ�तक��को�सुधारना�है।�

भारतीय��ौ�ो�गक��सं�थान�खड़गपुर�SPARC का��कम�के�काया��वयन�हेतु�रा�ीय�सम�वयक�सं�थान�है।�

वे�सभी�भारतीय�सं�थान��ज�ह�ने�इं�डया�र��क��स�(NIRF) के�सव��च�100 अथवा��ेणीवार�सव��च�100 म���थान�बनाया�है, आवेदन�के�पा�


ह�।�

केवल�वे��नजी�सं�थान�जो�उपरो���ेणी�म��आते�ह�, तथा�वे�भी�जो�UGC अ�ध�नयम�क��धारा�12(8) के�अंतग�त�आते�ह�, पा��ह�।�

QUESTION 2.
हाल�ही�म��सरकार�ने�सामा�जक��व�ान�म��नी�तगत�शोध�को��ो�सा�हत�करने�के�उ�े �य�के�साथ�IMPRESS योजना�शु��क��है।�IMPRES
S श�द�का��या�अथ��है�?

a) इ�पै�टफुल�पॉ�लसी��रसच��इन�सोशल�साइंसेज�
b) इ�पै�टफुल�पॉ�लसी��रसच��इन�सोशल�से�टर�
c) इ��ेस�सव�पॉ�लसी��रसच��इन�सोशल�से�टर�
d) इ�प��सव�पॉ�लसी��रसच��इन�सोशल�साइंसेज�
Correct Answer: A
Your Answer:
Explanation

Solution (d)

IASbaba
Web: http://ilp.iasbaba.com/ Score:
Email: ilp@iasbaba.com 0.00 / 198
Page 81
2019 - Test 32 -
Exam Title :
Government...
Email :
Contact :

सामा�जक��व�ान��े���म��नी�तगत�शोध�को��ो�सा�हत�करने�के�उ�े �य�के�साथ�क���सरकार�ने�दे श�म��‘ IMPRESS ’ योजना�शु��क��है।�

‘IMPRESS’ का�पूण���व�यास�है�- इ�पै�टफुल�पॉ�लसी��रसच��इन�सोशल�साइंसेज�

यह�योजना�दे श�के��कसी�भी�सं�थान�(�जसम��क���य�एवं�रा�य�स�हत�सभी��व��व�ालय�शा�मल�ह�) के�सामा�जक��व�ान�शोधा�थ�य��हेतु�एक


अवसर��दान�करेगी।�

योजना�का�काया��वयन�इं�डयन�क��सल�ऑफ़�सोशल�साइंस�एंड��रसच��(ICSSR) �ारा��कया�जाएगा।�

IMPRESS के�अंतग�त�, 1,500 शोध�प�रयोजना��को�दो�वष��के��लए�उ�चतर��श�ण�सं�थान��म��सामा�जक��व�ान�शोध�को�समथ�न�दे ने


हेतु�सहायता�द��जायेगी।�

QUESTION 3.
�डजी�– या�ा�के�बारे�म���न�न�ल�खत�कथन��पर��वचार�क��जए।�

1. इसे�क���य�नाग�रक�उ�यन�मं�ालय��ारा�शु���कया�गया�था।�

2. इस�पहल�के��ारा�, �टकट-बु�क�ग�, एअरपोट� �एं���तथा�बो�ड�ग�पास��स�यू�रट��चेक-इन�को��ड�जटल�बना��दया�जाएगा।�

3. यह�योजना�उन�पय�टक��को��न:शु�क�गे�ट�हाउसेस�भी��दान�करती�है�जो�पहली�बार�भारत�क��या�ा�कर�रहे�ह�।�

उपयु���म��से�कौन�से�कथन�स�य�ह��?

a) केवल�1 और�2
b) केवल�2 और�3
c) केवल�1 और�3
d) उपयु���सभी�
Correct Answer: A
Your Answer:
Explanation

Solution (c)

क���य�नाग�रक�उ�यन�मं�ालय�ने�एयरपोट्� स�पर�या��य��क��बायोमे��क�आधा�रत��ड�जटल��ोसे�स�ग�पर�नी�त�जारी�क��है��जसे��डजी�– या�ा�


कहा�जाता�है।�

इस�पहल�का�उ�े �य�पेपर�वहीन�एवं�क�ठनाई-र�हत�वायु�या�ा�को�बढ़ावा�दे ना�है।�

यह�फरवरी�2019 तक�ब�गलु��एवं�हैदराबाद�एयरपोट्� स�पर�चालू�हो�जाएगी।�

बाद�के�चरण�म��, एयरपोट्� स�ऑथो�रट��ऑफ़�इं�डया�(AAI) इस�पहल�को�कोलकाता, वाराणसी, पुणे�एवं��वजयवाड़ा�हवाई�अ���पर�अ�ैल�2


019 तक�लागू�करेगी।�

मह�वपूण���ब���

1) �डजी�– या�ा�के�अंतग�त�, ID का��योग�कर�पहली�बार�या�ा�करते�समय���थान�हवाई�अ�े�पर�वन-टाइम�स�यापन�होगा।�

2) सफल�स�यापन�के�प�ात�, चेहरे�क��पहचान�कर�उसे��डजी�– या�ा�ID म��सं��हत��कया�जाएगा।�

3) इस�पहल�के�साथ�, �टकट-बु�क�ग�, एअरपोट� �एं���तथा�बो�ड�ग�पास��स�यू�रट��चेक-इन�को��ड�जटल�बना��दया�जाएगा।�

4) इस��स�टम�के��लए�, या��य��को�क���कृत�तं��के�मा�यम�से�पंजीकृत��कया�जाएगा�तथा�उ�ह���डजी�– या�ा�ID �दान�क��जाएगी।�

QUESTION 4.

IASbaba
Web: http://ilp.iasbaba.com/ Score:
Email: ilp@iasbaba.com 0.00 / 198
Page 82
2019 - Test 32 -
Exam Title :
Government...
Email :
Contact :

उ�म�अ�भलाषा�योजना�के�बारे�म���न�न�ल�खत�म��से�कौन�से�कथन�सही�नह��ह��?

1. यह�योजना�क���य��म�एवं�रोजगार�मं�ालय��ारा�शु��क��गई�थी।�

2. इसे�नी�त�आयोग��ारा�28 रा�य��के��च��त�सभी�115 आकां�ी��जल��म��शु���कया�गया�था।�

3. यह�आकां�ी��जल��म���ामीण�युवा��क��उनके��वयं�के�उ�म��था�पत�करने�म��सहायता�कर�उ�ह��उ�मी�बनने�हेतु��ो�सा�हत�करने�का
उ�े �य�रखता�है।�

गलत�कूट�चु�नए�

a) केवल�1 और�2
b) केवल�1
c) केवल�2
d) उपरो��म��से�कोई�नह��
Correct Answer: B
Your Answer:
Explanation

Solution (b)

भारतीय�लघु�उ�ोग��वकास�ब�क�(SIDBI) ने�उ�म�अ�भलाषा�नाम�का�एक�रा�ीय��तर�का�उ��मता�जाग�कता�अ�भयान�शु���कया�है।�

इसे�नी�त�आयोग��ारा�28 रा�य��के��च��त�सभी�115 आकां�ी��जल��म��शु���कया�गया�था।�

उ�म�अ�भलाषा�के�उ�े �य�

1) आकां�ी��जल��म���ामीण�युवा��क��उनके��वयं�के�उ�म��था�पत�करने�म��सहायता�कर�उ�ह��उ�मी�बनने�हेतु��ो�सा�हत�करना।�

2) दे श�भर�म���ड�जटल�मा�यम�से���श�ण�दे ना।�

3) CSC-VLEs हेतु��वसा�यक�अवसर�पैदा�करना।�

4) म�हला�उ��मय��को��ो�सा�हत�करने�हेतु�आकां�ी��जल��म��म�हला�अ�य�थ�य��पर��यान�क���त�करना।�

5) भागीदार��को�ब�कयो�य�बनने�तथा�अपने��वयं�के�उ�म��था�पत�करने�हेतु�ब�क��से�साख�सु�वधा��ा�त�करने�म��सहायता�करना।�

5)

QUESTION 5.
�न�न�ल�खत�म��से�कौन�से�रा�य�ने��नमा�ण�मज�र��के�ब�च��को�रा�य�म��उनक��तकनीक���श�ा�हेतु��व�ीय�सहायता�उपल�ध�करवाने�हेतु�' �नर्
माण�कुसुम�' काय��म�आरंभ��कया�है�?

a) म�य��दे श�
b) �बहार�
c) ओ�डशा�
d) प. बंगाल�
Correct Answer: C
Your Answer:
Explanation

Solution (c)

IASbaba
Web: http://ilp.iasbaba.com/ Score:
Email: ilp@iasbaba.com 0.00 / 198
Page 83
2019 - Test 32 -
Exam Title :
Government...
Email :
Contact :

ओ�डशा�सरकार�ने��नमा�ण�मज�र��के�ब�च��को�रा�य�म��उनक��तकनीक���श�ा�हेतु��व�ीय�सहायता�उपल�ध�करवाने�हेतु�' �नमा�ण�कुसुम�' का
य��म�आरंभ��कया�है�

QUESTION 6.
अटल�बी�मत�����क�याण�योजना�के�बारे�म���न�न�ल�खत�कथन��पर��वचार�क��जए।�

1. इसे�मानव�संसाधन�एवं��वकास�मं�ालय��ारा�शु���कया�गया�था।�

2. यह�कम�चारी�रा�य�बीमा�अ�ध�नयम�के�अंतग�त�आने�वाले�बी�मत����य��हेतु�सम�प�त�है।�

3. यह�योजना�उनके�बेरोजगार�होने�एवं�नई�नौकरी�क��तलाश�के�दौरान�उनके�ब�क�खाते�म��सीधे�नकद��के�राहत�भुगतान�से�संबं�धत�है।�

उपयु���म��से�कौन�से�कथन�स�य�ह��?

a) केवल�1 और�2
b) केवल�2 और�3
c) केवल�1 और�3
d) उपयु���सभी�
Correct Answer: B
Your Answer:
Explanation

Solution (b)

कम�चारी�रा�य�बीमा��नगम�(ESIC) ने�कम�चारी�रा�य�बीमा�अ�ध�नयम�के�अंतग�त�आने�वाले�बी�मत����य��हेतु�अटल�बी�मत����
क�याण�योजना�नामक�एक�नई�योजना�का�अनुमोदन��कया�है।�

मह�वपूण���ब���

1) यह�योजना�उनके�बेरोजगार�होने�एवं�नई�नौकरी�क��तलाश�के�दौरान�उनके�ब�क�खाते�म��सीधे�नकद��के�राहत�भुगतान�से�संबं�धत�है।�

2) आ��त��हेतु�सुपर��पेश�लट��उपचार�लेने�क��पा�ता�को�अब�सुगम�बना��दया�गया�है�तथा�इसे�(पा�ता�को) अब�156 �दन��के�योगदान


स�हत�एक�वष��का�बी�मत�रोजगार�कर��दया�गया�है।�

3) ESIC ने�बी�मत�����क��मृ�यु�पर�अं�तम�सं�कार�हेतु��दए�जाने�वाले��य��को�वत�मान�10 हजार��पय��से�बढ़ाकर�15 हजार��पये


करने�के���ताव�को�भी�अनुमो�दत��कया�है।�

QUESTION 7.
नीलाकु�र�जी�पौधे�के�बारे�म���न�न�ल�खत�कथन��पर��वचार�क��जए।�

1) इस�पौधे�पर�14 वष��म��केवल�एक�बार�फूल�आते�ह�।�

2) यह�एक�उ�णक�टबंधीय�पौधे�क���जा�त�है�जो�प��मी�घाट�के�शोला�वन��हेतु��था�नक�है।�

उपयु���म��से�कौन�से�कथन�स�य�ह��?

a) केवल�1
b) केवल�2
c) 1 और�2 दोन��
d) उपरो��म��से�कोई�नह��
Correct Answer: D
Your Answer:
Explanation

IASbaba
Web: http://ilp.iasbaba.com/ Score:
Email: ilp@iasbaba.com 0.00 / 198
Page 84
2019 - Test 32 -
Exam Title :
Government...
Email :
Contact :

Solution (d)

त�मलनाडु �सरकार�ने�12 वष��म��केवल�एक�बार�फूल�दे ने�वाले��व�श��पादप�नीलाकु�र�जी�(Strobilanthus kunthianus) के�संर�ण


हेतु�योजना�क��घोषणा�क��है।�

यह�योजना�इन��ल�भ�एवं�पा�र��थ�तक�य��प�से��व�श��फूल��को�पैक�कर�वा�ण��यक�आधार�पर��व�य�करने�क���शकायत��के�बाद�आई�है।�

नीलाकु�र�जी�पौधा�

1) नीलाकु�र�जी�एक�उ�णक�टबंधीय�पौधे�क���जा�त�है�जो�प��मी�घाट�के�शोला�वन��हेतु��था�नक�है।�

2) यह�पूव��घाट�क��शेरवॉय�पहा�ड़य�, अ�ामलाई�पहा�ड़य��एवं�केरल�क��अगाली�पहा�ड़य��तथा�कणा�टक�क��सं���पहा�ड़य��म��भी�दे खी
जाती�है।�

QUESTION 8.
�धानमं�ी�अ�दाता�आय�संर�ण�अ�भयान�(PM-AASHA) के�बारे�म���न�न�ल�खत�म��से�कौन�से�कथन�स�य�ह��?

1. यह�एक�अ��ेला�काय��म�था��जसक��घोषणा�2018 के�क���य�बजट�म��क��गई�थी।�

2. इस�योजना�का�उ�े �य��कसान��हेतु�उनक��उपज�का�पा�रतोषक�मू�य�सु�न��त�करना�है।�

3. यह�रा�य��को�तीन�योजना��म��से�चुनने�क��अनुम�त�दे ता�है।�

सही�कूट�चु�नए-

a) केवल�1 और�2
b) केवल�2 और�3
c) केवल�1 और�3
d) उपयु���सभी�
Correct Answer: D
Your Answer:
Explanation

Solution (d)

�धानमं�ी�क��अ�य�ता�म��क���य�कै�बनेट�ने�एक�नई�अ��ेला�योजना�“ �धानमं�ी�अ�दाता�आय�संर�ण�अ�भयान�” (PM-AASHA) को


अनुमो�दत��कया�है।�2018 के�क���के�बजट�क��घोषणा�के�अनुसार�इस�योजना�का�उ�े �य��कसान��हेतु�उनक��उपज�का�पा�रतोषक�मू�य
सु�न��त�करना�है।�

उ�े �य�– इस�योजना�का�उ�े �य��कसान��हेतु�उनक��उपज�का�पा�रतोषक�मू�य�सु�न��त�करना�है।�

इसका�उ�े �य��कसान��को�सश��बनाना�एवं�कृ�ष��े��को�मजबूत�करना�है।�

आवंटन�- कै�बनेट�ने�PM-AASHA के�काया��वयन�हेतु�अगले�दो��व�ीय�वष��म��15,053 करोड़��पये��वीकृत��कए�ह�, �जसम��से�6,250


करोड़��पए�इस�वष���य��कए�जाय�गे।�

PM-AASHA के�अवयव�: PM-AASHA के�अंतग�त�, रा�य��को�तीन�योजना��म��से�चुनने�का��वक�प�होगा�–

1) मू�य�समथ�न�योजना�(PSS),

2) मू�य�हीनता�भुगतान�योजना�(PDPS)

3) पायलट�ऑफ़��ाइवेट��ो�योरम�ट�एंड��टॉ�क�ट��क�म�/ (PPPS)

IASbaba
Web: http://ilp.iasbaba.com/ Score:
Email: ilp@iasbaba.com 0.00 / 198
Page 85
2019 - Test 32 -
Exam Title :
Government...
Email :
Contact :
QUESTION 9.
अटल�प�शन�योजना�के�बारे�म���न�न�ल�खत�म��से�कौन�से�कथन�सही�ह��?

1. इसने�पूव��क��सरकार�सम�थ�त��वावलंबन�योजना�का��थान��लया�था।�

2. आयु�सीमा�18 से�65 वष��के�बीच�है।�

3. यह�10,000 �पए�क��अ�ध�वकष��सु�वधा�और�2 लाख��पए�का��घ�टना�बीमा�कवर�करती�है।�

सही�कूट�चु�नए-

a) केवल�1 और�2
b) केवल�2 और�3
c) केवल�1 और�3
d) उपयु���सभी�
Correct Answer: A
Your Answer:
Explanation

Solution (a)

क���य�मं��मंडल�ने�और�लोग��क��योजना�म��भागीदारी�को��ो�सा�हत�करने�के��लए�अटल�प�शन�योजना�(जो�अग�त�2018 म��साम�त�हो�रही
थी) को�अ�न��त�काल�हेतु��व�ता�रत�करने�का��नण�य��लया�है।�

अटल�प�शन�योजना�(APY) –

यह�सरकार��ारा�1,000 �पए�से�5,000 �पए�तक�क��प�रभा�षत�प�शन��दान�करने�क��एक�सामा�जक�सुर�ा�योजना�है��जसे�2015 म�


शु���कया�गया�था।�

इसने�पहले�क��सरकार�सम�थ�त�प�शन�क���वावलंबन�योजना�का��थान��लया�है।�

योजना�क��मु�य��वशेषता�:

1) अ�ध�वकष��सु�वधा�दोगुनी�– �व��मं�ी�अ�ण�जेटली�ने�कहा��क�अ�ध�वकष��सु�वधा�को�दोगुना�कर�इस�योजना�के�अंतग�त�5,000 से�10,


000 �पए�कर��दया�गया�है।�

2) आयु�सीमा�बढ़��- आयु�सीमा�को�संशो�धत�कर�18 से�65 वष��के�बीच�कर��दया�गया�है�जो�पहले�18 से�60 वष��के�बीच�थी।�

3) �घ�टना�बीमा�कवर�बढ़ा�- �घ�टना�बीमा�कवर�को�1 लाख��पए�से�बढ़ाकर�2 लाख��पए�कर��दया�गया�है।�

QUESTION 10.
�वदे श�दश�न�योजना�के�बारे�म���न�न�ल�खत�कथन��पर��वचार�क��जए।�

1. इसे�सं�कृ�त�मं�ालय��ारा�आरंभ��कया�गया�था।�

2. इस��च��त�तीथ�या�ा��थल��के��वकास�और�स�दय�करण�पर��यान�क���त�करती�है।�

3. �वदे श�दश�न�योजना�के�अंतग�त�पहली�प�रयोजना�‘नॉथ��ई�ट�स�क�ट: इ�फाल�एंड�ख��गओम’ का�उ�ाटन�म�णपुर�म���कया�गया�था।�

उपयु���म��से�कौन�से�कथन�स�य�ह��?

a) केवल�1 और�2
b) केवल�2 और�3
c) केवल�3

IASbaba
Web: http://ilp.iasbaba.com/ Score:
Email: ilp@iasbaba.com 0.00 / 198
Page 86
2019 - Test 32 -
Exam Title :
Government...
Email :
Contact :

d) उपयु���सभी�
Correct Answer: C
Your Answer:
Explanation

Solution (c)

‘�वदे श�दश�न’ एवं�‘�साद’ योजना�के�अंतग�त�पय�टन�मं�ालय�कोष��क��उपल�धता, लं�बत�उपयो�गता��माणप���क��समा��त�एवं�योजना�के


�दशा�नद� श��क��अनुपालना�के�अधीन�रहते��ए�रा�य�सरकार�/संघ��े���के��शासन�को�क���य��व�ीय�सहायता�(CFA) �दान�करता�है।�

�वदे श�दश�न�योजना�के�अंतग�त��वकास�हेतु�तेरह��थमै�टक�स�क�ट्स�क��पहचान�क��गई�है��जनके�नाम�ह��: नाथ�-ई�ट�इं�डया�स�क�ट�, बौ�


स�क�ट�, �हमालयन�स�क�ट�, को�टल�स�क�ट�, कृ�णा�स�क�ट�, डेजट� �स�क�ट�, �ाइबल�स�क�ट�, इको�स�क�ट�, वाइ�डलाइफ�स�क�ट�, �रल
स�क�ट�, ��प�रचुअल�स�क�ट�, रामायण�स�क�ट�तथा�हे�रटे ज�स�क�ट।�

�साद�योजना�के�अंतग�त�तेरह��थल��को��वकास�हेतु��च��त��कया�गया�है, �जनके�नाम�ह��: अमृतसर�, अजमेर�, �ारका�, मथुरा�, वाराणसी�,


गया�, पुरी�, अमरावती�, कांचीपुरम�, वे�लनक�ली�, केदारनाथ�, काम�या�एवं�पटना।�

‘�साद’ योजना�के�अंतग�त��च��त�तीथ��थल��के��वकास�एवं�स�दय�करण�पर�जोर�रहेगा।�वह��, �वदे श�दश�न�योजना�के�अंतग�त�पहचाने�गए


‘��प�रचुअल�स�क�ट’ के�तहत, �कसी�रा�य�और�संघ�शा�सत��दे श�के��व�भ��धा�म�क/आ�या��मक��थल��के��वकास�पर�जोर�रहेगा।�

�वदे श�दश�न�योजना�के�अंतग�त�पहली�प�रयोजना�‘नॉथ��ई�ट�स�क�ट: इ�फाल�एंड�ख��गओम’ का�उ�ाटन�म�णपुर�म���कया�गया�था।�

प�रयोजना�दो��थल��को�कवर�करती�है�अथा�त�म�णपुर�का�कांगला�फोट� �तथा�ख��गओम।�

QUESTION 11.
�न�न�ल�खत�म��से�कौन�से�मं�ालय�ने�O-SMART योजना�शु��क��है�?

a) भू-�व�ान�मं�ालय�
b) �व�ान�एवं��ौ�ो�गक��मं�ालय�
c) इले��ॉ�न�स�एवं�सूचना��ौ�ो�गक��मं�ालय�
d) �बजली�मं�ालय�
Correct Answer: A
Your Answer:
Explanation

Solution (a)

O-SMART योजना�भू-�व�ान�मं�ालय��ारा�शु��क��गई�है।�

यह�योजना�कुल�16 उप-प�रयोजना��को�कवर�करती�है�जो�सेवा�, �ो�ो�गक�, संसाधन, �े�ण�एवं��व�ान�जैसी�महासागर��वकास


ग�त�व�धय��का�समाधान�करती�ह�।�

O-SMART के�अंतग�त��द��सेवाएं�तट�य�एवं�महासागरीय��े���के�अनेक���यो�ा�समुदाय��(नामत: मा���यक�, अपतट�य�उ�ोग, तट�य


रा�य�, र�ा, जहाजरानी, बंदरगाह�इ�या�द) को�आ�थ�क�लाभ��दान�करेगी।�वत�मान�म��, पाँच�लाख�मछु आरा�समुदाय���त�दन�इस�सूचना�को
मोबाइल�के�मा�यम�से��ा�त�कर�रहे�ह���जसके�अंतग�त�तट�य�जल�म��मछली�संभावना��का�आवंटन�तथा��थानीय�मौसम�दशाएं�शा�मल�है।�इसे
मछु आर��के��लए�खोज�समय�म��कमी�लाने�म��मदद��मलेगी��जससे��धन�लागत�बचेगी।�

O-SMART का�काया��वयन�सतत��वकास�ल�य�-14 से�जुड़े�मु���के�समाधान�म��सहायता�करेगा��जसका�उ�े �य�सतत��वकास�हेतु


महासागर��के��योग�एवं�समु���संसाधन��का�संर�ण�करना�है।�यह�योजना�(O-SMART) �लू�इकॉनमी�के��व�भ��पहलु��के�काया��वयन
हेतु�आव�यक�अ�नवाय��वै�ा�नक�एवं��ो�ो�गक�य�पृ�भू�म�भी��दान�करती�है।�

O-SMART योजना�के�अंतग�त��था�पत�उ�त�अल��वा�न�ग��स�ट�स�सुनामी�एवं�तूफ़ान�महो�म�य��जैसी�महासागरीय�आपदा��से��नपटने�म�
मदद�कर�गे।�

IASbaba
Web: http://ilp.iasbaba.com/ Score:
Email: ilp@iasbaba.com 0.00 / 198
Page 87
2019 - Test 32 -
Exam Title :
Government...
Email :
Contact :

इस�योजना�के�अंतग�त��वक�सत�क��जा�रही��ौ�ो�गक�य���ारा�भारत�के�आस�पास�के�सागर��से�जी�वत�एवं�गैर-जीवी�संसाधन��के��वशाल
भंडार�के�दोहन�म��सहायता��मलेगी।�

QUESTION 12.
�माट� �इं�डया�हैकाथन�2019 के�बारे�म���न�न�ल�खत�कथन��पर��वचार�क��जए।�

1. यह�मानव�संसाधन�एवं��वकास�मं�ालय�क��एक�पहल�है।�

2. यह��व��का�सबसे�बड़ा�सॉ�टवेर�तथा�हाड�वेयर�हैकाथन�है।�

3. यह�हमारे��ारा���त�दन�के�जीवन�म��सामना�क��जाने�वाली�कुछ�बड़ी�सम�या��के�समाधान�का�मंच��दान�करता�है�तथा�इस��कार�उ�पाद
नवाचार�व�सम�या�समाधान�क���वृ�त�क��सं�कृ�त�को�आ�मसात�करता�है।�

उपयु���म��से�कौन�से�कथन�स�य�ह��?

a) केवल�1 और�2
b) केवल�2 और�3
c) केवल�1 और�3
d) उपयु���सभी�
Correct Answer: D
Your Answer:
Explanation

Solution (d)

�माट� �इं�डया�हैकाथन�2019 एक�रा��ापी�पहल�है�जो�छा���को���त�दन�के�जीवन�म��सामना�क��जाने�वाली�कुछ�बड़ी�सम�या��के�समाधान


का�मंच��दान�करती�है�और�इस��कार�उ�पाद�नवाचार�एवं�सम�या�समाधान�क���वृ�त�क��सं�कृ�त�को�आ�मसात�करवाती�है।�

हैकाथन�के��पछले�सं�करण�म���व�भ��इंजी�नय�र�ग�कॉलेज��से�पचास�लाख�से�अ�धक�लोग��क��भागीदारी�रही�जो�35 से�अ�धक��थान��पर
सव��च��थान�पाने�के��लए���त�पधा��कर�रहे�थे।�

SIH 2019 म��, छा���के�पास��नजी��े��के�संगठन��के�भीतर�सामना�क��जाने�वाली�चुनौ�तय��पर�काय��करने�एवं��व��क��कुछ�सव��े�


कंप�नय��हेतु��व���तरीय�समाधान�बनाने�तथा�इस��कार��नजी��े��को�दे श�भर�से�सव��े��बु��मान�लोग��को�काम�पर�रखने�म��मदद�करने�का
अवसर�भी�होगा।�

SIH 2019 �या�है�?

1) HRD मं�ालय�, AICTE, पर�स�ट� ट��स�ट�स�, i4c तथा�रामभाऊ�महा�गी��बो�धनी�क��एक�पहल�

2) इसम��भारत�भर�से�1 लाख�से�अ�धक�तकनीक��छा�, 3000 से�अ�धक�तकनीक��सं�थान�, 200 से�अ�धक�संगठन�शा�मल�ह�।�

3) �व��क��सबसे�बड़ी�सॉ�टवेर�एवं�हाड�वेयर�हैकाथन�

4) �माट� �इं�डया�हैकाथन�पहल�का�तीसरा�अ�यंत�सफल�सं�करण�

5) भारत�भर�से��ौ�ो�गक��छा��सम�या��के�सृजना�मक�ढं ग�से�समाधान�एवं�तकनीक��समाधान���तुत�करने�हेतु���त�पधा��करते�ह��

6) यह�IISc, IITs, NITs और�AICTE/UGC अनुमो�दत�सं�थान��के�छा���क���वशेष�ता�का�दोहन�करती�है�

SIH 2019 से��य��जुड़��?

1) अपनी�सम�या��हेतु�लागत��भावी�ढं ग�से�नवाचारी�समाधान��ा�त�करने�के��लए�

2) रा�ीय��प�से�अपने�संगठन�क���ां�ड�ग�का�अवसर�

IASbaba
Web: http://ilp.iasbaba.com/ Score:
Email: ilp@iasbaba.com 0.00 / 198
Page 88
2019 - Test 32 -
Exam Title :
Government...
Email :
Contact :

3) भारत�के�सभी�तकनीक��सं�थान��म��आपके�संगठन�हेतु�मा�यता�और��दश�न�

4) दे श�भर�से�तकनीक�के�जानकार�युवा�आपक��सम�या��का�नवाचारी�समाधान��दान�करते�ह��

5) �व��क��सबसे�बड़ी�ओपन�इनोवेशन�मूवम�ट�का�भाग�बनना�

6) दे श�के�कुछ�सवा��धक���तभावान�लोग��के�साथ�काय��करने�का�अवसर�

QUESTION 13.
IMPRINT 2 के�बारे�म���न�न�ल�खत�म��से�कौन�से�कथन�सही�ह��?

1. इसे�मानव�संसाधन��वकास�मं�ालय��ारा�शु���कया�गया�है।�

2. �व�ान�एवं��ौ�ो�गक��(DST) �वभाग�म��SERB ( साइंस�एंड�इंजी�नय�र�ग��रसच��बोड��) को�IMPRINT-2 पहल�के�काया��वयन�हेतु


नोडल�एज�सी�बनाए�जाने�का���ताव�है।�

3. इसके�अंतग�त�केवल�IITs और�IISC को�उन��मुख��े���क��पहचान�करने�को�कहा�जाता�है�जहाँ�भारत�इंजी�नय�र�ग�और�टे �नोलॉजी


संबंधी�चुना�तय��का�सामना�कर�रहा�है।�

सही�कूट�चु�नए-

a) केवल�1 और�2
b) केवल�2 और�3
c) केवल�1 और�3
d) उपयु���सभी�
Correct Answer: A
Your Answer:
Explanation

Solution (a)

के���य�मानव�संसाधन��वकास�मं�ालय�(MHRD) 112 करोड़��पए�के�122 नए�शोध�प�रयोजना���ताव��को�अनुमो�दत��कया�है��ज�ह��I


MPRINT-II ( इ�प��टं ग��रसच��इनोवेशन�एंड�टे �नोलॉजी�) योजना�के�तहत��व�पो�षत��कया�जाएगा।�

IMPRINT �मुख��व�ान�एवं�इंजी�नय�र�ग�चुऔ�तय��के�समाधान�तथा�दे श�के�10 �ो�ो�गक���च��त��े���म��मौ�लक�वै�ा�नक�एवं


�ौ�ो�गक�य�शोध�को�बढ़ावा�दे ने�संबं�धत�MHRD सम�थ�त�अपने�तरह�क��पहली�योजना�है।�

इसे�नवंबर�2015 म��नई�इंजी�नय�र�ग��श�ा�नी�त�के��वकास�एवं�इंजी�नय�र�ग�चुनौ�तय��का�सामना�करने�का�खाका�ख�चने�हेतु�शु���कया�गया
था।�

�व�ान�एवं��ौ�ो�गक��(DST) �वभाग�म��SERB ( साइंस�एंड�इंजी�नय�र�ग��रसच��बोड��) को�रा�ीय�सम�वयक�के�साथ�काय��करने��ए�IMP


RINT-2 पहल�के�काया��वयन�हेतु�नोडल�एज�सी�बनाए�जाने�का���ताव�है।�SERB आवं�टत�संसाधन��के�साथ�अपने�सम��ढाँचे�के�अंतग�त
एक�पृथक�वट�कल�एवं�IMPRINT के�अंतग�त�शोध�प�रयोजना��के�चयन, फं�ड�ग�एवं��बंधन�हेतु�एक�समयब��काय�वाही�योजना
बनाएगा।�स�चव�(HE) तथा�स�चव�(DST) इस�वट�कल�क��सह-अ�य�ता�कर�गे।�प�रयोजना�समी�ा�एवं��ान��बंधन�हेतु�इं�डयन�नेशनल
अकादमी�ऑफ़�इंजी�नय�र�ग�(INAE) के��वशेष��पूल�का�भी�उपयोग��कया�जाएगा।�

IMPRINT एक�अ�यंत�मह�वपूण�����कोण��दान�करता�है�जो�उन��े���म��शोध��नद� �शत�होता�है�जो��मुख��प�से�समाज�के��लए��ासं�गक


ह�।�आरंभ�म��IMPRINT-I के�अंतग�त�, IITs एवं�IISC को�उन��मुख��े���क��पहचान�करने�को�कहा�गया�था��जनम��भारत�इंजी�नय�र�ग
एवं��ौ�ो�गक�य�चुनौ�तय��का�सामना�कर�रहा�है।�अब�इसका�दायरा�IMPRINT-II के�अंतग�त�IITs एवं�IISc स�हत��नजी�सं�थान��तक
�व�ता�रत�कर��दया�गया�है�तथा�इसे�MHRD एवं��व�ान�एवं��ौ�ो�गक��(DST) �वभाग��ारा�संयु���प�से��व�पो�षत�एवं�संचा�लत��कया
जाता�है।�

QUESTION 14.

IASbaba
Web: http://ilp.iasbaba.com/ Score:
Email: ilp@iasbaba.com 0.00 / 198
Page 89
2019 - Test 32 -
Exam Title :
Government...
Email :
Contact :

�धानमं�ी�उ��वला�योजना�के�बारे�म���न�न�ल�खत�कथन��पर��वचार�क��जए।�

1. इसे�‘ �व�छ�इंधन, बेहतर�जीवन’ �वषय�के�साथ�2016 म��शु���कया�गया�था।�

2. प. बंगाल�ने�इस�योजना�के�अंतग�त�सवा��धक�LPG कने�शन��ा�त��कए�ह�।�

3. इसे�नवीन�एवं�नवीकरणीय�उजा��मं�ालय��ारा�काया���वत��कया�जा�रहा�है।�

उपयु���म��से�कौन�से�कथन�स�य�ह��?

a) केवल�1 और�2
b) केवल�1
c) केवल�1 और�3
d) उपयु���सभी�
Correct Answer: B
Your Answer:
Explanation

Solution (b)

क���सरकार�ने�घोषणा�क��है��क�इसने��धानमं�ी�उ��वला�योजना�(PMUY) के�अंतग�त�5 करोड़�LPG कने�शन��दान�करने�के�ल�य�को


तय�समय�से�लगभग�आठ�महीने�पूव��( अथा�त�35 महीन��क��बजाए�27 महीन��म��) ही�पूरा�कर��लया�है।�

�व���वा��य�संगठन�(WHO) ने�PMUY को�इंडोर�वायु���षण�(जो�दे श�म����त�वष��दस�लाख�लोग��क��मृ�यु�का�कारण�बनता�है) के


समाधान�हेतु�सरकार�के��नणा�यक�ह�त�ेप�म��से�एक�माना�है।�

मह�वपूण���ब���:

1) क���सरकार�ने�मई�2016 म��PMUY को�‘ �व�छ�इंधन, बेहतर�जीवन’ �वषय�के�साथ�म��शु���कया�गया�था।�

2) इसका�ल�य�गरीब�प�रवार��को�खाना�पकाने�हेतु��व�छ��धन�उपल�ध�करवाना�है�जो�अ�यथा�इंडोर�वायु���षण�से�जुडी�अनेक���वा��य
सम�या��के���त�सुभे��ह��तथा�उनके�जीवन��तर��म��गुणव�ापरक�प�रवत�न�लाना�है।�

3) PMUY को�पे�ो�लयम�एवं��ाकृ�तक�गैस�मं�ालय��ारा�काया���वत��कया�जा�रहा�है।�

4) इसक���वशाल�सफलता�को�दे खते��ए�सरकार�ने�इसके�ल�य�को�बढ़ाकर�12,800 करोड़��पए�के�आवंटन�के�साथ�8 करोड़�कर��दया�है।�

5) उ�र��दे श�को�सवा��धक�सं�या�म��LPG कने�शन�(87 लाख�) �ा�त��ए�ह���जसके�बाद�प. बंगाल�(67 लाख�) एवं��बहार�(61 लाख�)


का��थान�है।�

QUESTION 15.
सुक�या�समृ�ध�योजना�(SSY) के�बारे�म���न�न�ल�खत�म��से�कौन�से�कथन�सही�ह��?

1. इस�योजना�के�अंतग�त�खाता�खोलने�हेतु��यूनतम�आरं�भक�जमा�रा�श�250 �पए��नयत�क��गई�है।�

2. यह�योजना�18 वष��से�कम�आयु�क��बा�लका��हेतु�है।�

3. एक�बा�लका�इस�योजना�के��लए�तभी�पा��है�जब�खाता�खोले�जाते�समय�वह�एक��नवासी�भारतीय�नाग�रक�है।�

सही�कूट�चु�नए-

a) केवल�1 और�2
b) केवल�2 और�3
c) केवल�1 और�3

IASbaba
Web: http://ilp.iasbaba.com/ Score:
Email: ilp@iasbaba.com 0.00 / 198
Page 90
2019 - Test 32 -
Exam Title :
Government...
Email :
Contact :

d) उपयु���सभी�
Correct Answer: D
Your Answer:
Explanation

Solution (d)

क���सरकार�ने�सुक�या�समृ�ध�योजना�के�अंतग�त�खाते�म���यूनतम�वा�ष�क�जमा�आव�यकता�को�1000 �पए�से�घटाकर�250 �पए�कर��दया


है।�

खाता�खोलने�क��आरं�भक�जमा�आव�यकता�को�भी�घटाकर�250 �पए�कर��दया�गया�है।�

इस�कदम�का�उ�े �य�अ�धक�लोग��को�बा�लका�बचत�योजना�का�लाभ�लेने�म��स�म�बनाना�है।�

मह�वपूण���ब���:

ल�य�- योजना�का�उ�े �य�बा�लका�को�उसका��ववाह�होने�तक��व�ीय�सुर�ा��दान�करना�है।�

आयु�सीमा�- सुक�या�समृ�ध�खाता�योजना�10 वष��से�कम�आयु�क��बा�लका��के��लए�है।�

यह�खाता�21 वष��म��प�रप�व�होता�है, �जससे�पहले�यह�लॉक-इन�अव�ध�म��होता�है, �जसके�दौरान�इससे�पैसा�नह���नकाला�जा�सकता�है।�

साथ�ही�, जुलाई-�सत�बर�हेतु��याज�दर�8.1% है।�

लोचशील�जमा�खाता�: खाता��यूनतम�250 �पए�क��रा�श�तथा�उसके�प�ात�100 �पए�के�गुणक�म��खोला�जा�सकता�है।�

��त�वष��अ�धकतम�1.5 लाख��पये�खाते�म��जमा��कये�जा�सकते�ह�।�

एक�बा�लका�तभी�SSY खाते�हेतु�पा��है�जब�वह�खाता�खोले�जाते�समय�एक��नवासी�भारतीय�नाग�रक�है�तथा�खाते�क��प�रप�वता�अथवा
उसके�बंद�होने�तक�नाग�रक�बनी�रहती�है।�

अ�नवासी�भारतीय�अब�SSY खाता�नह��खोल�सकते।�असल�म��, य�द�आपके�अथवा�आपके�ब�चे�का��नवास�दजा��बदलकर�अ�नवासी�का�हो


जाता�है�अथवा�वह�योजना�क��अव�ध�के�दौरान�अ�य�दे श�क��नाग�रकता�ले�लेती�है, तब�उसे�नाग�रकता�लेने�अथवा��नवासी�दजा��प�रव�त�त�होने
क��तारीख�से�कोई��याज�भुगतान�नह���कया�जाएगा�तथा�खाते�को�बंद�माना�जाएगा।�

QUESTION 16.
�न�न�ल�खत�म��से��कसने��व�छ�सव��ण��ामीण�2018 �कया�है�?

a) पेयजल�एवं��व�छता�मं�ालय�
b) �ामीण��वकास�एवं�पंचायती�राज�सं�थाएं�मं�ालय�
c) �वा��य�एवं�प�रवार�क�याण�मं�ालय�
d) नी�त�आयोग�
Correct Answer: A
Your Answer:
Explanation

Solution (a)

पेयजल�एवं��व�छता�मं�ालय�ने�मा�ा�मक�एवं�गुणव�ापरक��व�छता�पैमान��के�आधार�पर�भारत�के�सभी��जल��क��र��क�ग��वक�सत�करने�हेतु
एक��वतं��एज�सी�के�मा�यम�से�“ �व�छ�सव��ण��ामीण�2018” (SSG 2018) शु���कया�है।�यह�र��क�ग�पैमान��के�एक��ापक�समु�चय
�जसम���कूल�, आंगनवा�ड़य�, PHCs, हाट�/ बाजार��, पंचायत�जैसे�साव�ज�नक��थल��का�सव��ण, �व�छता�के���त�नाग�रक��का�नज�रया
तथा�काय��म�म��सुधार�के��लए�उनक��अनुशंसाएं�व�SBM-G IMIS से��ा�त�डेटा�शा�मल�है, के�आधार�पर��कया�गया�है।�

IASbaba
Web: http://ilp.iasbaba.com/ Score:
Email: ilp@iasbaba.com 0.00 / 198
Page 91
2019 - Test 32 -
Exam Title :
Government...
Email :
Contact :

ह�रयाणा�को�सव��े��रा�य�और�महारा��रा�य�के�सतारा�को��व�छ�सव��ण��ामीण�2018 के��हसाब�से�सव��े���जले�क��र��क�ग��दान�क��गई।
उ�र��दे श�को�नाग�रक��क��अ�धकतम�भागीदारी�हेतु�पुर�कार��दया�गया।�बाद�म��पेयजल�एवं��व�छता�मं�ी�सु�ी�उमा�भारती�ने�सव��च�तीन
रा�य��एवं�दे श�के�सभी�जोन��से�सव��च�तीन��जल��को��वासी�भारतीय�क��, नई��द�ली�म��पुर�कार��दान��कए।�

QUESTION 17.
सौर�चरखा��मशन�के�बारे�म���न�न�ल�खत�कथन��पर��वचार�क��जए।�

1. इसे�नवीन�एवं�नवीकरणीय�उजा��मं�ालय��ारा�शु���कया�गया�था।�

2. खाद��एवं��ामो�ोग�आयोग�(KVIC) इस��मशन�हेतु�काया��वयन�एज�सी�है।�

3. �ामीण�अथ��व�था�को�बढ़ावा�दे ना�और��ामीण�से�शहरी��े���क��ओर���जन�को��नयं��त�करना�इसके�उ�े �य��म��से�एक�है।�

उपयु���म��से�कौन�से�कथन�स�य�ह��?

a) केवल�1 और�2
b) केवल�2 और�3
c) केवल�1 और�3
d) उपयु���सभी�
Correct Answer: B
Your Answer:
Explanation

Solution (b)

सौर�चरखा��मशन�सू�म, लघु�एवं�म�यम�उ�म�(MSME) मं�ालय�क��एक�पहल�है��जसे�जून�2018 म��आर�भ��कया�गया�था।�खाद��एवं


�ामो�ोग�आयोग�(KVIC) इस�काय��म�का�काया��वयन�करेगी।�

इसे�रा�प�त�राम�नाथ�को�व�द��ारा�शु���कया�गया�था��जसके�अंतग�त�सरकार�हजार���श��पय��(जो��ामीण��े���म��रोजगार�पैदा�करते�ह�) को�
550 करोड़��पए�क��स��सडी��दान�करेगी।�

इसे��व��MSME �दवस�( �जसे�27 जून�को�मनाया�जाता�है�) के�अवसर�पर�उ�म�संगम�( रा�ीय�MSME कॉ��लेव�) काय��म�के�दौरान


शु���कया�गया�था।�

योजना�के�उ�े �य��न�न�ल�खत�ह��:

1) �ामीण��े���म��सौर�चरखा��ल�टस��के�मा�यम�से�रोजगार�सृजन�(�वशेषकर�म�हला��एवं�युवा��के��लए) एवं�सतत��वकास��ारा
समावेशी�संवृ���सु�न��त�करना।�

2) �ामीण�अथ��व�था�को�बढ़ावा�दे ना�तथा��ामीण�से�शहरी��े���क��ओर�होने�वाले��वज�न�को��नयं��त�करना।�

3) कम�लागत�क��नवाचारी��ौ�ो�गक�य��एवं�जी�वका�हेतु����या��का��योग�करना।�

QUESTION 18.
सेवा�भोज�योजना�के�बारे�म���न�न�ल�खत�कथन��पर��वचार�क��जए।�

1. इसे�क���य�सं�कृ�त�मं�ालय��ारा�शु���कया�गया�था।�

2. यह�धमा�थ��धा�म�क�सं�थान��(CRIs) �ारा��व�श��भो�य�मद��क��लोग��म���वत�रत��कए�जाने�हेतु�क��जाने�वाली�खरीद�पर��व�ीय�सहायता
�दान�करता�है।�

3. इस�योजना�के�अंतग�त�धा�म�क�सं�थान���ारा�खरीद��जाने�वाली�क�ची�भो�य�साम��य��पर�लगे�क���य�व�तु�एवं�सेवा�कर�(CGST) को
पूण�त: वापस�कर��दया�जाएगा।�

IASbaba
Web: http://ilp.iasbaba.com/ Score:
Email: ilp@iasbaba.com 0.00 / 198
Page 92
2019 - Test 32 -
Exam Title :
Government...
Email :
Contact :

उपयु���म��से�कौन�से�कथन�स�य�ह��?

a) केवल�1 और�2
b) केवल�2 और�3
c) केवल�1 और�3
d) उपयु���सभी�
Correct Answer: D
Your Answer:
Explanation

Solution (d)

क���य�सं�कृ�त�मं�ालय�ने�धमा�थ��धा�म�क�सं�थान��(CRIs) �ारा��व�श��भो�य�मद��क��लोग��म���न:शु�क��वत�रत��कए�जाने�हेतु�क��जाने
वाली�खरीद�पर��व�ीय�सहायता��दान�करने�वाली�सेवा�भोज�योजना�नामक�एक�नई�योजना�शु��क��है।�

योजना�के�अंतग�त�, धा�म�क�सं�थान���ारा��य�क��जाने�वाली�क�ची�भोजन�साम��य��पर�लगने�वाले�क���य�व�तु�एवं�सेवा�कर�(CGST) म�
क���सरकार�के��ह�से�को�वापस�लौटा��दया�जाएगा।�

इस�योजना�को�325 �पये�के�कुल�आवंटन�के�साथ��व�ीय�वष��2018-19 से�2019-20 के��लए�शु���कया�गया�है।�

QUESTION 19.
सौभा�य�योजना�के�बारे�म���न�न�ल�खत�कथन��पर��वचार�क��जए।�

1. इसका�उ�े �य�अं�तम�����तक�उजा��प�ँच��दान�करना�एवं�बचे��ए�सभी�अ�व�ु�तकृत�घर��को��बजली�कने�शन��दान�करना�है।�

2. �बजली�मं�ा�य�इस�योजना�का�काया��वयन��ा�धकरण�है।�

3. इस�योजना�के�अंतग�त�केवल��ामीण�घर�आते�ह�।�

उपयु���म��से�कौन�से�कथन�स�य�ह��?

a) केवल�1 और�2
b) केवल�2 और�3
c) केवल�1 और�3
d) उपयु���सभी�
Correct Answer: A
Your Answer:
Explanation

Solution (a)

�धानमं�ी�सहज��बजली�हर�घर�योजना�- सौभा�य�योजना�का�उ�े �य�अं�तम�����तक�उजा��प�ँच��दान�करना�एवं�बचे��ए�सभी�(�ामीण�एवं


शहरी�दोन�) अ�व�ु�तकृत�घर��को��बजली�कने�शन��दान�करना�है�ता�क�दे श�म��प�रवार��का�साव�भौम��व�ुतीकरण�हा�सल��कया�जा�सके।�

घर��को��दए�जाने�वाले��बजली�कने�शन�म��नजद�क��पोल�से�घर�के�आँगन�तक�स�व�स�केबल�लाना, �बजली�मीटर�लगाना, LED ब�ब�के


साथ�एक��बजली�पॉइंट�वाय�र�ग�एवं�एक�मोबाइल�चा�ज�ग�पॉइंट�शा�मल�है।�य�द�स�व�स�केबल�लाने�के��लए��नकट�कोई�पोल�उप��थत�नह��है,
कंड�टर�के�साथ-साथ�अ�त�र��पोल�लगाना�तथा�सहायक�साम�ी�भी�इस�योजना�म��कवर�क��जाएगी।�

मु�य�बात��

1) इस�योजना�को�पं�डत�द�नदयाल�उपा�याय�क��ज�मशती�के�अवसर�पर�25 �सतंबर�को�शु���कया�गया�है।�

2) योजना�का�ल�य�2019 तक�दे श�भर�के�सभी�घर��को��बजली�कने�शन��दान�कर�24x7 �बजली�का�ल�य�हा�सल�करना�है।�

IASbaba
Web: http://ilp.iasbaba.com/ Score:
Email: ilp@iasbaba.com 0.00 / 198
Page 93
2019 - Test 32 -
Exam Title :
Government...
Email :
Contact :

3) योजना�के�अंतग�त��ांसफाम�स�, वायस��एवं�मीटस��हेतु�स��सडी��दान�क��जाएगी।�

4) �बजली�मं�ालय�इसका�काया��वयन��ा�धकरण�होगी।�

5) दे श�के��ामीण�एवं�शहरी�दोन���े���म���बजली�कने�शन��दए�जाय�गे।�

QUESTION 20.
हाल�ही�म��सरकार�ने�घोषणा�क���क�अं�तम�अ�व�ुतीकृत��ाम�लीस�ग�को���ड�से�जोड़��दया�गया�है।�लीस�ग��न�न�ल�खत�म��से�कौन�से�रा�य�म�
��थत�है�?

a) असम�
b) म�णपुर�
c) �मजोरम�
d) नागाल�ड�
Correct Answer: B
Your Answer:
Explanation

Solution (b)

लीस�ग�म�णपुर�म����थत�है।�

QUESTION 21.
�ा��त�ए�प�के�बारे�म���न�न�ल�खत�म��से�कौन�सा�कथन�स�य�है�?

a) इसे��बजली�मं�ालय��ारा�आरंभ��कया�गया�है�ता�क�उ�पादक��और��ड�कॉ�स�के�बीच�के��बजली��य�लेनदे न��म��पारद�श�ता�लाई�जा
सके।�
b) इसे��बजली�मं�ालय��ारा�आरंभ��कया�गया�है�ता�क�प�रवार���ारा��बजली��य�को�पारदश��बनाया�जा�सके।�
c) इसे�कोयला�आवंटन��को�पारदश��बनाने�हेतु�कोयला�मं�ालय��ारा�आरंभ��कया�गया�है।�
d) इसे��बजली�चोरी�के�मु�े�के�समाधान�हेतु��बजली�मं�ालय��ारा�आरंभ��कया�गया�है।�
Correct Answer: A
Your Answer:
Explanation

Solution (a)

उ�पादक��और��ड�कॉ�स�के�बीच�के��बजली��य�लेनदे न��म��पारद�श�ता�लाने�हेतु�एक�वेब�पोट� ल�www.praapti.in और�एक�ए�प��जसका


नाम��ा��त�(पेम�ट�रे�ट�फकेशन�एंड�एना�ल�सस�इन�पॉवर��ो�योरम�ट�फॉर�����ग�ग��ांसपेर�सी�इन�इनवॉइ�स�ग�ऑफ़�जनरेटस��) शु���कया�गया
था।�

ए�प�तथा�वेब�पोट� ल�उ�पादक��से��व�भ��लंबी�अव�ध�के�PPAs हेतु�इनवॉइ�स�ग�एवं�भुगतान�संबंधी�डेटा�को�दे खेगा।�यह��ड�कॉ�स�के��बजली


�य�के�सापे��अद��रा�श�के�माहवार�एवं�पुराने�डेटा�को�पाने�म���हतधारक��क��मदद�करेगा।�

यह�ए�प��यो�ा��को��ड�कॉ�स��ारा��बजली�उ�पादक�कंपनी�को��कए�गये�भुगतान��तथा�वे��जस��कार��कये�गए�थे, से�संबं�धत��ववरण
जानने�म��मदद�करेगी।�

�ा��त�उपभो�ा��को�उ�पादक�कंप�नय��को��कये�गए�भुगतान��के�सापे��उनके��ड�कॉ�स�के��व�ीय��दश�न�के�आंकलन�म��भी�सहायता
करेगी।�

पोट� ल��ड�कॉ�स�एवं�उ�पादक�कंप�नय��को�अद��भुगतान��के�समाधान�म��भी�सहायता�करेगा।�

QUESTION 22.

IASbaba
Web: http://ilp.iasbaba.com/ Score:
Email: ilp@iasbaba.com 0.00 / 198
Page 94
2019 - Test 32 -
Exam Title :
Government...
Email :
Contact :

आईपी�नानी�के�बारे�म���न�न�ल�खत�म��से�कौन�सा�कथन�सही�है�?

1. यह�एक�तकनीक�क��जानकार�नानी�माँ�है�जो�सरकार�और��वत�न�अ�भकरण��क��मानव���ा�पार�से��नपटने�म��सहायता�करती�है।�

2. इसे�नई��द�ली�म��रा�ीय�बौ��क�संपदा�अ�धकार�नी�त�पर�आयो�जत�कां��स�म��शु���कया�गया�था।�

सही�कूट�चु�नए�

a) केवल�1
b) केवल�2
c) 1 और�2 दोन��
d) उपरो��म��से�कोई�नह��
Correct Answer: B
Your Answer:
Explanation

Solution (b)

क���य�मं�ी��ी�सुरेश��भु�ने�नई��द�ली�म��आयो�जत�एक�कां��स�म��बौ��क�संपदा�(IP) शुभकंर�IP Nani लांच��कया।�

शुभकंर�IP Nani तकनीक�क��जानकार�एक�नानी�माँ�है�जो�सरकार�और��वत�न�अ�भकरण��क��अपने�पौ��“ छोटू �” अथा�त�आ�द�य�क�


मदद�से�IP अपराध��से��नपटने�म��सहायता�करती�है।�

IP शुभकंर�लोग��म��(�वशेषकर�ब�च��म�) सु��चपूण��ढं ग�से�बौ��क�संपदा�अ�धकार��(IPRs) के�मह�व�के�बारे�म��जाग�कता�फैलाने�म��मदद


करेगा।�

QUESTION 23.
‘�वयं’ के�बारे�म���न�न�ल�खत�म��से�कौन�से�कथन�स�य�ह��?

1. इसे�मानव�संसाधन��वकास�मं�ालय��ारा�आरंभ��कया�गया�था।�

2. यह�आठव��से��नातक�तक�क�ा�म��पढ़ाई�जाने�वाली��श�ा�साम�ी�को�ऑनलाइन�लाने�हेतु�मै�सव�ओपन�ऑनलाइन�कोस�ज��लेटफाम��का
�योग�करता�है��जस�तक�कोई�भी, कही�भी, �कसी�भी�समय�प�ँच�बना�सकता�है।�

3. �वयं��लेटफाम��को�माइ�ोसॉ�ट�क��मदद�से�मानव�संसाधन��वकास�मं�ालय�(MHRD) तथा�अ�खल�भारतीय�तकनीक���श�ा�प�रषद�(A
ICTE) �ारा��वदे शी��प�से��वक�सत��कया�गया�है।�

सही�कूट�चु�नए-

a) केवल�1 और�2
b) केवल�2 और�3
c) केवल�1 और�3
d) उपयु���सभी�
Correct Answer: C
Your Answer:
Explanation

Solution (c)

मानव�संसाधन��वकास�मं�ालय�ने��ड�जटल��लेटफॉ�स��का��योग�कर��श�क��को���श��त�करने�हेतु�‘�वयं’ नामक�एक�पहल�शु��क��है।�

सरकार�मै�सव�ओपन�ऑनलाइन�कोस�ज��लेटफाम���वयं�का��योग�15 लाख�उ�चतर��श�ा�फैक�ट��को�(नए�और�उभरते��झान��पर��यान�के
साथ) उनक���श�ण�तकनीक�को�अ�तन�बनाने�हेतु�करेगी।�

IASbaba
Web: http://ilp.iasbaba.com/ Score:
Email: ilp@iasbaba.com 0.00 / 198
Page 95
2019 - Test 32 -
Exam Title :
Government...
Email :
Contact :

इसे�हा�सल�करने�के��लए�, पहले�चरण�म��, 75 संकाय��व�श��नेशनल��रसोस��स�टस��(NRCs) क��पहचान�ऑनलाइन��े �न�ग�साम�ी�तैयार


करने�एवं�ऑनलाइन��र�ेशर�कोस��के�मा�यम�से��श�क��को�उनके�संबं�धत��वषय��म��अ�भनव��ग�तय��के�बारे�म��पूण�त: सू�चत�रखने�हेतु�क�
गई�है।�

क���य, रा�य�एवं�मु���व��व�ालय��जैसे�अनेक�सं�थान, नेशनल�इं��ट�ूट�फॉर�टे ��नकल�ट�चस���े �न�ग�, IITs, IIITs, NITs एवं�अ�य�


को�इस�संबंध�म��NRCs के��प�म��अ�धसू�चत��कया�गया�है।�

�वयं��लेटफाम��को�माइ�ोसॉ�ट�क��मदद�से�मानव�संसाधन��वकास�मं�ालय�(MHRD) तथा�अ�खल�भारतीय�तकनीक���श�ा�प�रषद�(AIC
TE) �ारा��वदे शी��प�से��वक�सत��कया�गया�है�तथा�यह�अंततोग�वा�2000 कोस�ज�एवं�80000 अ�धगम�घंट��को�समा�हत�करने�म��स�म
होगा, �जसम���कूल, अंडर-�ेजुएट, परा�नातक, इंजी�नय�र�ग, लॉ�एवं�अ�य�पेशेवर�कोस�ज�कवर��कए�जाय�गे।�

सभी�कोस�स�अंत:��या�वाले�ह���ज�ह��दे श�के�सव��े���श�क���ारा�तैयार��कया�गया�है�तथा�वे�भारत�के��नवा�सय��हेतु��न:शु�क�उपल�ध�ह�।�दे
श�भर�से�1,000 से�अ�धक��व�श���प�चय�नत�फैक�ट��एवं��श�क��ने�इन�कोस�ज�को�बनाने�म��भागीदारी�क��है।�

QUESTION 24.
अटल��यू�इं�डया�चैल�ज�के�बारे�म���न�न�ल�खत�कथन��पर��वचार�क��जए।�

1. इसे�अटल�इ�ोवेशन��मशन�(AIM) के�अंतग�त�लोग��हेतु��ासं�गक�नवाचार��तथा��ौ�ो�गक�य��को�लाने�के�उ�े �य�से�शु���कया�गया�था।�

2. अटल�इ�ोवेशन��मशन�के�अंतग�त��मता, इरादा�एवं��ौ�ो�गक�य��के�उ�पाद�करण�क��संभा�ता�दशा�ने�वाले�आवेदक��को�1 करोड़��पए


तक�के�अनुदान��दए�जाय�गे।�

3. इसे�नी�त�आयोग��ारा�शु���कया�गया�था।�

उपयु���म��से�कौन�से�कथन�स�य�ह��?

a) केवल�1 और�2
b) केवल�2 और�3
c) केवल�1 और�3
d) उपयु���सभी�
Correct Answer: D
Your Answer:
Explanation

Solution (d)

अटल��यू�इं�डया�चैल�ज�– 26 अ�ैल�2018 को�नी�त�आयोग�ने�अटल�इ�ोवेशन��मशन�(AIM) के�अंतग�त�लोग��हेतु��ासं�गक�नवाचार��तथा


�ौ�ो�गक�य��को�लाने�के�उ�े �य�से�अटल��यू�इं�डया�चैल�ज�शु���कया।�

अटल�इ�ोवेशन��मशन�के�अंतग�त��मता, इरादा�एवं��ौ�ो�गक�य��के�उ�पाद�करण�क��संभा�ता�दशा�ने�वाले�आवेदक��को�1 करोड़��पए�तक


के�अनुदान��दए�जाय�गे।�

इस�पहल�का�उ�े �य�17 �व�भ���े���क��सम�या��का�समाधान�करना�है��जनका�नाग�रक��के�जीवन�सुधार�एवं�रोजगार�उ�प���पर���य�


�भाव�होगा।�

इस�नई�पहल�के�तहत�, अटल�इ�ोवेशन��मशन�ने�सड़क�प�रवहन�एवं�राजमाग��मं�ालय, आवास�एवं�शहरी�मामले�मं�ालय, कृ�ष�एवं�प�रवार


क�याण�मं�ालय, पेयजल�एवं��व�छता�मं�ालयतथा�रेलवे�बोड��के�साथ�साझीदारी�क��है।�

QUESTION 25.
उ�त�भारत�अ�भयान�के�बारे�म���न�न�ल�खत�कथन��पर��वचार�क��जए।�

1. इसे��ामीण��वकास�मं�ालय��ारा�आरंभ��कया�गया�था।�

IASbaba
Web: http://ilp.iasbaba.com/ Score:
Email: ilp@iasbaba.com 0.00 / 198
Page 96
2019 - Test 32 -
Exam Title :
Government...
Email :
Contact :

2. यह�एक�समावेशी�भारत�के�ढाँचे�के��नमा�ण�म��मदद�हेतु��ान�सं�थान��का��योग�कर��ामीण��वकास����या��म���पांतरणीय�प�रवत�न�लाने
का����कोण�रखता�है।�

3. इसके�अंतग�त���येक��वधायक�एक�नजद�क��गाँव�को�गोद�लेगा�तथा�गाँव�के�लोग��के�जीवन�तथा�उन��ारा�दै �नक�जीवन�म��सामना�क��जाने
वाली�सम�या��से��ब��होगा।�

उपयु���म��से�कौन�से�कथन�स�य�ह��?

a) केवल�1 और�2
b) केवल�2
c) केवल�1 और�3
d) उपयु���सभी�
Correct Answer: B
Your Answer:
Explanation

Solution (b)

मानव�संसाधन��वकास�मं�ालय�(HRD) ने�नई��द�ली�म��उ�त�भारत�अ�भयान�के��सरे�सं�करण�का�आरंभ��कया।�

उ�त�भारत�अ�भयान�एक�समावेशी�भारत�के�ढाँचे�के��नमा�ण�म��मदद�हेतु��ान�सं�थान��का��योग�कर��ामीण��वकास����या��म���पांतरणीय
प�रवत�न�लाने�के����कोण�से��े�रत�है।�

इस��मशन�का�उ�े �य�दे श�भर�के�750 महा�व�ालय��एवं��व��व�ालय��के�छा���को�एक�नजद�क��गाँव�को�गोद�लेने�तथा�उसक��या�ा�कर�गाँव


के�लोग��के�जीवन�तथा�उन��ारा�दै �नक�जीवन�म��सामना�क��जाने�वाली�सम�या��से��ब��होने�म��स�म�बनाना�है।�

उ�त�भारत�अ�भयान�का��मशन�उ�चतर��श�ा�सं�थान��को��वकासा�मक�चुनौ�तय��क��पहचान�करने�हेतु��ामीण�भारत�के�लोग��के�साथ�काम
करने�म��स�म�बनाना�तथा�सतत�संवृ���को�ग�त�दे ने�हेतु�उ�चत�समाधान��का��वकास�करना�है।�यह�उभरते��वसाय��हेतु��ान�और�अ�यास�
को��दान�कर�समाज�एवं�समावेशी�अकाद�मक�त���के�बीच�एक�गुणा�मक�च��पैदा�करने�एवं��ामीण�भारत�क���वकासा�मक�आव�यकता�
क����तपु���म��साव�ज�नक�व��नजी��े���दोन��क���मता��को�अ�तन�बनाने�का�ल�य�भी�रखता�है।�

QUESTION 26.
�रवाइटलाइ�ज�ग�इ��ा���चर�एंड��स�ट�स�इन�एजुकेशन�(RISE) योजना�के�बारे�म���न�न�ल�खत�कथन��पर��वचार�क��जए।�

1. इसक��घोषणा�2017-18 के�क���य�बजट�म��क��गई�थी��जसका�उ�े �य�सरकारी�उ�च��श�ण�सं�थान��को�स�ती�लागत�के�कोष��दान


करना�है।�

2. इसके�अंतग�त�, केवल�IITs, IIMs, NITs तथा�IISERs �व�तार�एवं�नई�अवसंरचना��नमा�ण�हेतु�अगले�4 वष��म��1,00,000 करोड़


�पए�के�काप�स�से�उधार�ले�सकते�ह�।�

3. हायर�एजुकेशन�फाइन��स�ग�एज�सी�(HEFA) जो��क�एक�गैर-ब��क�ग��व�ीय�कंपनी�है, इस�योजना�हेतु�एक��व�ीय�एज�सी�के��प�म��काय�


करती�है।�

उपयु���म��से�कौन�से�कथन�स�य�ह��?

a) केवल�1 और�2
b) केवल�2 और�3
c) केवल�1 और�3
d) उपयु���सभी�
Correct Answer: C
Your Answer:
Explanation

IASbaba
Web: http://ilp.iasbaba.com/ Score:
Email: ilp@iasbaba.com 0.00 / 198
Page 97
2019 - Test 32 -
Exam Title :
Government...
Email :
Contact :

Solution (c)

RISE योजना�क��घोषणा�2017-18 के�क���य�बजट�म��क��गई�थी।�इसका�उ�े �य�सरकारी�उ�च��श�ण�सं�थान��को�स�ती�लागत�के�कोष


�दान�करना�है।�इसके�तहत�, सभी�क����व�पो�षत�सं�थान�(CFIs), �जनम��क���य��व��व�ालय, IITs, IIMs, NITs तथा�IISERs शा
�मल�ह�, �व�तार�एवं�नई�अवसंरचना��नमा�ण�हेतु�अगले�4 वष��म��1,00,000 करोड़��पए�के�काप�स�से�उधार�ले�सकते�ह�।�इसका��व�यन
पुनस�र�चत�हायर�एजुकेशन�फाइन��स�ग�एज�सी�(HEFA) जो��क�एक�गैर-ब��क�ग��व�ीय�कंपनी�है, �ारा��कया�जाएगा।�

मह�वपूण���ब���

· RISE के�आने�के�साथ�, उ�चतर��श�ा�के�सभी�CFIs म��अवसंरचना��वकास�का�सारा��व�यन�HEFA के�मा�यम�से��कया�जाएगा��जसे


सरकार��ारा�2017 म��एक�धारा�8 कंपनी�( धमा�थ��उ�े �य��वाली�कंपनी�) म���था�पत��कया�गया�था�ता�क�क����ारा�संचा�लत�सं�थान��हेतु
बाजार�से�कोष�जुटाए�जा�सक��और�उ�ह��10 वष�य�ऋण��दान��कए�जा�सक�।�

· समता�अंश�: RISE के�अंतग�त�1 लाख�करोड़�का�काप�स�फ�ड�जुटाने�हेतु�, HEFA को�10,000 करोड़��पए�क��इ��वट��क�


आव�यकता�होगी��जसम��से�8,500 करोड़�सरकार��ारा�और�शेष�केनरा�ब�क�(�जसने�HEFA क���थापना�हेतु�सरकार�के�साथ�भागीदारी�क�
है) एवं�अ�य��नगम���ारा��दान��कया�जायेगा।�

· ल�य�: HEFA �ारा��वीकृत�सभी�अवसंरचना�एवं�शोध�प�रयोजनाएं��दसंबर�2022 तक�पूरी�क��जानी�ह�।�

· कोष�जुटाना�: HEFA काय�कारी�एज�सी�एवं��श�ण�सं�थान���ारा��माणीकरण��कये�जाने�पर��व�ेता��अथवा�ठे केदार��को�सीधे�धन�जारी


करेगा।�RISE काय��म�के�अंतग�त�HEFA से��लये�गए�ऋण��का�भुगतान�10 वष��म���कया�जाएगा।��व�भ��सं�थान��हेतु�उनके�आंत�रक
राज�व�के�आधार�पर�ऋण�पुनभु�गतान�क���व�भ���व�धयाँ�ह�गी।�

QUESTION 27.
कुसुम�योजना�के�बारे�म���न�न�ल�खत�म��से�कौन�से�कथन�स�य�ह��?

1. इसे�नवीन�एवं�नवीकरणीय�उजा��मं�ालय��ारा�बनाया�गया�है।�

2. इ�छु क��कसान��को�सौर�प�प�उपल�ध�करवाना�इस�योजना�के��ाथ�मक�उ�े �य��म��से�एक�है।�

3. यह��ामीण��े���म����ड�से�जुड़े�सौर�उजा��संयं���(�जनक����येक�क���मता�2 MW तक�क��होगी) क���थापना�का���ताव�करती�है।�

सही�कूट�चु�नए-

a) केवल�1 और�2
b) केवल�2 और�3
c) केवल�1 और�3
d) उपयु���सभी�
Correct Answer: B
Your Answer:
Explanation

Solution (b)

नवीन�एवं�नवीकरणीय�उजा��मं�ालय�ने�‘�कसान�उजा��सुर�ा�एवं�उ�थान�महा�भयान�(KUSUM)’ नामक�एक�योजना�तैयार�क��है।�

कुसुम�योजना�का���ताव��न�न�ल�खत��ावधान�करता�है�:-

1) �ामीण��े���म����ड�से�जुड़े�सौर�उजा��संयं���(�जनक����येक�क���मता�2 MW तक�क��होगी) क���थापना।�

2) ��ड�से�नह��जुड़े��कसान��क���स�चाई�आव�यकता��क��पूत��हेतु��ट� डअलोन�ऑफ-��ड�सौर�प�प�क���थापना�; तथा�

3) ��ड�से�जुड़े��व�मान�कृ�ष�प�प��का�सौरकरण�ता�क�उ�ह����ड�स�लाई�से��वतं��बनाया�जा�सके�तथा�वे�उस��ारा�उ�पा�दत�अ�धशेष�सौर
श���को��ड�कॉ�स�को�बेचकर�अ�त�र��आय�कमाने�म��स�म�हो�सक�।�

IASbaba
Web: http://ilp.iasbaba.com/ Score:
Email: ilp@iasbaba.com 0.00 / 198
Page 98
2019 - Test 32 -
Exam Title :
Government...
Email :
Contact :

इ�छु क��कसान��को�सौर�प�प�उपल�ध�करवाना�इस�योजना�के��ाथ�मक�उ�े �य��म��से�एक�है।�सरकार�कहती�है��क�कृ�ष���मक��को�17.5 ला


ख�सौर�श���से�संचा�लत�प�प��दान��कये�जाय�गे।�

सरकार�ने�यह�भी�घोषणा�क��है��क�यह�संयं���के��नमा�ण�क��भी�पहल�करेगी�जो�सौर�उजा��उ�पा�दत�कर�गे।�मसौदे �के�अनुसार�, ये�संयं��केवल


बंजर��े���म��लगाए�जाय�गे��जनक��कुल�उ�पादन��मता�28000 MW होगी।�

सौर�उजा��संयं���के�अ�त�र��, सरकार�खेत��म��सौर�प�प�लगाने�क���दशा�म��काय��करेगी�जहाँ�वत�मान�म��डीजल�पंप�ह�।�इन�प�पस�क���मता�7
20 MW होगी।��वतरण�के�अलावा�, यह�योजना�सभी��कसान��को�सौर�प�प�लगाकर�अ�धक�आय�कमाने�का�अवसर�भी��दान�करेगी।
�कसान���ारा�उ�पा�दत�उजा��क��अ�धशेष�मा�ा�को���ड�को�बेचा�जा�सकेगा।�

QUESTION 28.
ऑपरेशन��ीन�के�बारे�म���न�न�ल�खत�म��से�कौन�से�कथन�सही�नह��ह��?

1. यह�एक�मू�य��नयत�करने�वाली�योजना�है��जसका�उ�े �य�यह�सु�न��त�करना�है��क��कसान��को�उनक��उपज�का�सही�मू�य��दया�जाए।�

2. इसका�उ�े �य��कसान��क��सहायता�करना�तथा�केवल�टमाटर, �याज�एवं�आलू�क��क�मत��म���ु�टपूण��उतार-चढ़ाव��को��नयं��त�व�सी�मत


करना�है।�

सही�कूट�चु�नए�

a) केवल�1
b) केवल�2
c) 1 और�2 दोन��
d) उपरो��म��से�कोई�नह��
Correct Answer: D
Your Answer:
Explanation

Solution (c)

इसे�ऑपरेशन��लड�क��तज��पर�500 करोड़��पए�के�आवंटन�के�साथ�शु���कया�जाएगा।�इसका�ल�य��कसान�उ�पादक�संगठन�, �सं�करण


सु�वधा�, ए�ी-लो�ज��ट�स�एवं�पेशेवर��बंधन�को�बढ़ावा�दे ना�है।�

इसका�उ�े �य��कसान��क��सहायता�करना�तथा�केवल�टमाटर, �याज�एवं�आलू�(TOP) क��क�मत��म���ु�टपूण��उतार-चढ़ाव��को��नयं��त�व


सी�मत�करना�भी�है।�यह�मूल�प�से�मू�य��नयत�योजना�है��जसका�उ�े �य�यह�सु�न��त�करना�है��क��कसान��को�उनक��उपज�का�सही�मू�य
�मले।�इसके�पीछे ��वचार�है��क��कसान��क��आय�2022 तक�दोगुनी�क��जा�सके।�

QUESTION 29.
एकल��मॉडल�रे�जड��शयल��कू�ज�(EMRS) के�बारे�म���न�न�ल�खत�कथन��पर��वचार�क��जए।�

1. इस�योजना�का�काया��वयन�जनजा�त�मामले�मं�ालय��ारा��कया�जा�रहा�है।�

2. इ�ह��भारतीय�सं�वधान�के�अनु�छे द�275(1) के�अंतग�त�अनुदान��से�रा�य�/संघ�शा�सत��दे श��म���था�पत��कया�जाता�है।�

3. 2018-19 के�बजट�के�अनुसार�, 50% से�अ�धक�एवं��यूनतम�20,000 अनुसू�चत�जनजा�त�जनसँ�या�वाले���येक��लॉक�म��2022


तक�एक�एकल��मॉडल�रे�जड��शयल��कूल�होगा।�

उपयु���म��से�कौन�से�कथन�स�य�ह��?

a) केवल�1 और�2
b) केवल�2 और�3
c) केवल�1 और�3

IASbaba
Web: http://ilp.iasbaba.com/ Score:
Email: ilp@iasbaba.com 0.00 / 198
Page 99
2019 - Test 32 -
Exam Title :
Government...
Email :
Contact :

d) उपयु���सभी�
Correct Answer: D
Your Answer:
Explanation

Solution (d)

दे श�म��सभी��े���एवं�अ�धवास��म���श�ा�को�बढ़ावा�दे ने�हेतु�गुणव�ापरक�रे�जड��शयल��कू�ज��था�पत�करने�के��झान�के�संदभ��म��एसट��छा��
हेतु�जवाहर�नवोदय��व�ालय�, क�तूरबा�गाँधी�बा�लका��व�ालय��तथा�क���य��व�ालय��म��एकल��मॉडल�रे�जड��शयल��कू�ज�(EMRS)
खोले�जाय�गे।�एकल��मॉडल�रे�जड��शयल��कू�ज�(EMRS) भारतीय�सं�वधान�के�अनु�छे द�275(1) के�अंतग�त�अनुदान��से�रा�य�/संघ
शा�सत��दे श��म���था�पत��कया�जाता�है।�

इस�योजना�का�काया��वयन�भारत�सरकार�के�जनजा�त�मामले�मं�ालय��ारा��कया�जा�रहा�है।�

EMRS का�उ�े �य�सु�र��े���क��अनुसू�चत�जनजा�तय��(ST) को�गुणव�ापरक�मा�य�मक�एवं�उ�च��तरीय��श�ा��दान�करना�है��जससे�वे


न�केवल�उ�च�एवं�पेशेवर�शै��णक�कोस�ज�और�सरकारी�तथा��नजी��े��क��नौक�रय��म��आर�ण��ा�त�कर�पाएं, अ�पतु�गैर-एसट��जनसँ�या
के�समक���श�ा�म��सव��म�अवसर��तक�प�ँच�भी�बना�पाएं।�

2018-19 के�बजट�के�अनुसार�, 50% से�अ�धक�एवं��यूनतम�20,000 अनुसू�चत�जनजा�त�जनसँ�या�वाले���येक��लॉक�म��2022 तक


एक�एकल��मॉडल�रे�जड��शयल��कूल�होगा।�

QUESTION 30.
2018 के�बजट�म���कसान��े�डट�योजना�का��व�तार��न�न�ल�खत�म��से��कन�तक��कया�गया�?

1. मछु आर��तक�

2. मवेशी�रखने�वाल��तक�

3. बागान�मा�लक��तक�

4. खनन���मक��तक�

सही�कूट�चु�नए�

a) केवल�1, 2 और�3
b) केवल�1 और�2
c) केवल�2, 3 और�4
d) केवल�1 और�3
Correct Answer: B
Your Answer:
Explanation

Solution (b)

�कसान��े�डट�काड��का��व�तार�मछु आर��एवं�मवेशी�पालक��तक�भी��कया�गया�था।�यह�उ�ह��आसानी�से�ऋण��ा�त�करने�म��मदद�करेगा।�यह
�ामीण��े���म����ध�उ�पादन�के��वसाय�से�जुड़े�लोग��तथा�मछु आर��को��व�ीय�सहायता��दान�कर�मदद�करेगा।�

QUESTION 31.
अफोड�बल�हाउ�स�ग�फ�ड�के�बारे�म���न�न�ल�खत�कथन��पर��वचार�क��जए।�

1. यह��सडबी�के�अंतग�त��था�पत�कोष��का�कोष�है।�

2. इसक��घोषणा�2018-19 के�आ�थ�क�सव��ण�म��क��गई�थी।�

IASbaba
Web: http://ilp.iasbaba.com/ Score:
Email: ilp@iasbaba.com 0.00 / 198
Page 100
2019 - Test 32 -
Exam Title :
Government...
Email :
Contact :

3. इसे�भारत�सरकार��ारा�अ�धकृत��ायो�रट��से�टर�ल��ड�ग�शोट� फाल�एवं�पूण�त�से�वत�बांड्स�से��व�पो�षत��कया�जाता�है।�

उपयु���म��से�कौन�से�कथन�स�य�ह��?

a) केवल�1 और�2
b) केवल�2 और�3
c) केवल�3
d) उपयु���सभी�
Correct Answer: C
Your Answer:
Explanation

Solution (c)

क���य�बजट�2018 म���व��मं�ी�अ�ण�जेटली�ने�कम�लागत�पर�घर�दे ने�हेतु�एक�नई�योजना�क��घोषणा�क��है।�

इसका�उ�े �य�कम-लागत�वाले�घर��क��माँग�एवं�आपू�त��को�बढ़ावा�दे ना�है।�

सरकार�एक�सम�प�त�अफोड�बल�हाउ�स�ग�फ�ड�(AHF) क���थापना�रा�ीय�आवास�ब�क�म��करेगी��जसे�भारत�सरकार��ारा�अ�धकृत��ायो�रट�
से�टर�ल��ड�ग�शोट� फाल�एवं�पूण�त�से�वत�बांड्स�से��व�पो�षत��कया�जाएगा।�

QUESTION 32.
�न�न�ल�खत�म��से��कसने��व�थ�ब�चे, �व�थ�भारत�नामक�काय��म�क��पहल�क��है�?

a) क���य��व�ालय�संगठन�
b) मानव�संसाधन��वकास�मं�ालय�
c) नी�त�आयोग�
d) �वा��य�एवं�प�रवार�क�याण�मं�ालय�
Correct Answer: A
Your Answer:
Explanation

Solution (a)

�व�थ�ब�चे, �व�थ�भारत�मानव�संसाधन��वकास�मं�ी��ी��काश�जावडेकर��ारा�को�ची, केरल�म��शु���कया�गया�था।�

यह�काय��म�12 लाख�से�अ�धक�के���य��व�ालय�छा���हेतु�शारी�रक��वा��य�एवं��फटनेस��ोफाइल�काड��तैयार�करने�क��क���य��व�ालय
संगठन�क��पहल�है।�

QUESTION 33.
व�र��प�शन�बीमा�योजना�के�बारे�म���न�न�ल�खत�कथन��पर��वचार�क��जए।�

1. भारतीय�जीवन�बीमा��नगम�(LIC) इस�योजना�हेतु�काया��वयन��नकाय�होगा।�

2. इसका�उ�े �य�60 वष��आयु�के�व�र��लोग��को�उनक��वृ�ाव�था�के�दौरान�सामा�जक�सुर�ा��दान�करना�है।�

3. योजना�के�अंतग�त�, 8% वा�ष�क�क��गारंट�कृत�लाभ�दर�के�साथ�एक�सु�न��त�प�शन�दस�वष��के��लए��दान�क��जाएगी।�

उपयु���म��से�कौन�से�कथन�स�य�ह��?

a) केवल�1 और�2
b) केवल�2 और�3

IASbaba
Web: http://ilp.iasbaba.com/ Score:
Email: ilp@iasbaba.com 0.00 / 198
Page 101
2019 - Test 32 -
Exam Title :
Government...
Email :
Contact :

c) केवल�3
d) उपयु���सभी�
Correct Answer: D
Your Answer:
Explanation

Solution (d)

�व�ीय�समावेशन�एवं�सामा�जक�सुर�ा�क��सरकार�क����तब�ता�के�एक�भाग�के��प�म��क���य�कै�बनेट�ने�व�र��प�शन�बीमा�योजना�क�
शु�आत�को�अपना�प�ातवत��अनुमोदन��दया�है।�

�व�मान��व�ीय�वष��म��भारतीय�जीवन�बीमा��नगम�(LIC) इस�योजना�हेतु�काया��वयन��नकाय�होगा।�

योजना�का�उ�े �य�60 वष��आयु�के�व�र��लोग��को�उनक��वृ�ाव�था�के�दौरान�सामा�जक�सुर�ा��दान�करना�है।�

योजना�के�अंतग�त�, 8% वा�ष�क�क��गारंट�कृत�लाभ�दर�के�साथ�एक�सु�न��त�प�शन�दस�वष��के��लए��दान�क��जाएगी।�इसके�अ�त�र��, ना
मां�कत�����के�पास�मा�सक/�तमाही/छ:माही�अथवा�वा�ष�क�आधार�पर�प�शन�का��वक�प�होगा।�

QUESTION 34.
मॉ�डफाइड��पेशल�इंस��टव�पैकेज��क�म�(M-SIPS) के�बारे�म���न�न�ल�खत�कथन��पर��वचार�क��जए-

1. यह�2020 तक��नवल�शू�य�आयात��का�ल�य��ा�त�करने�हेतु�इले��ॉ�नक��स�टम��डज़ाइन�एंड�मै�युफै�च�र�ग�(ESDM) म��बड़े��तर�के


�व�नमा�ण�को��ो�सा�हत�करती�है।�

2. M-SIPS के�अंतग�त�, सरकार��वशेष�आ�थ�क��े���(SEZs) म��पूँजी��नवेश�पर�25 ��तशत�स��सडी��दान�करेगी।�

सही�कूट�चु�नए�

a) केवल�1
b) केवल�2
c) 1 और�2 दोन��
d) उपरो��म��से�कोई�नह��
Correct Answer: A
Your Answer:
Explanation

Solution (c)

सरकार�ने�इले��ॉ�नक��स�टम��डज़ाइन�एंड�मै�युफै�च�र�ग�(ESDM) �े��म��बड़े��तर�पर��व�नमा�ण�को��ो�सा�हत�करने�के��लए��पेशल
इंस��टव�पैकेज�अनुमो�दत��कया�है।�इस�योजना�को�मॉ�डफाइड��पेशल�इंस��टव�पैकेज��क�म�(M-SIPS) कहा�जाता�है।�

M-SIPS के�अंतग�त�, सरकार����गत�कंप�नय��को��वशेष�आ�थ�क��े���(SEZs) म��पूँजी��नवेश�पर�20 ��तशत�तथा�गैर- �वशेष


आ�थ�क��े���(SEZs) म��पूँजी��नवेश�पर�25 ��तशत�स��सडी��दान�करेगी।�यह�गैर- SEZ इकाइय��के�पूंजीगत�उपकरण��हेतु�CVD/ उत्
पाद�कर�के�वापसी�भुगतान�का��ावधान�करती�है।�फै��केटस��जैसी�उ�च��ौ�ो�गक��और�उ�च�पूँजी��नवेश�इकाइय��हेतु�भी�क���य�कर��और
शु�क��क��वापसी�क��जाएगी।�प�रयोजना�म���कये�गए��नवेश��हेतु��ो�साहन�अनुमोदन�क��तारीख�से�10 वष��के�भीतर�उपल�ध�ह�गे।
मॉ�डफाइड��पेशल�इंस��टव�पैकेज��क�म�(M-SIPS) भारत�म��इले��ॉ�नक��स�टम��डज़ाइन�एंड�मै�युफै�च�र�ग��े��म���नवेश��को��ो�सा�हत
करती�है।�

यह��ो�साहन�ESDM उ�पाद��क��29 �े�णय��म��उपल�ध�होगा��जनम��टे �लकॉम�, IT हाड�वेयर�, कं�यूमर�इले��ॉ�न�स�, मे�डकल


इले��ॉ�न�स�, ऑटोमे�टव�इले��ॉ�न�स�, सोलर�फोटोवो�टे क�, LEDs, LCDs, साम�रक�इले��ॉ�न�स�, ए�वयो�न�स�, इंड���यल
इले��ॉ�न�स�, नैनो-इले��ॉ�न�स�, सेमीकंड�टर��च�स�एवं��चप�कंपोन�ट्स�, अ�य�इले��ॉ�नक�कंपोन�ट्स�शा�मल�ह�।�वै�यू�चैन�क��इकाइय�
�जनम��क�ची�साम��यां�(इन��े�णय��के�उ�पाद��क��अस�बली, टे ��टं ग�पैके�ज�ग�एवं�ए�सेसरीज�स�हत) आती�ह�, इसम��शा�मल�ह�।�

QUESTION 35.

IASbaba
Web: http://ilp.iasbaba.com/ Score:
Email: ilp@iasbaba.com 0.00 / 198
Page 102
2019 - Test 32 -
Exam Title :
Government...
Email :
Contact :

�धानमं�ी��म�योगी�मानधन�योजना�के�बारे�म���न�न�ल�खत�कथन��पर��वचार�क��जए।�

1. यह�असंग�ठत��े��के�उन�कामगार��को�प�शन�लाभ��दान�करेगी��जनक��मा�सक�आय�15000 �पए�तक�है।�

2. वे�कामगार��जनक��आयु�18 से�40 वष��के�बीच�है, इसम��भागीदारी�करने�के�पा��ह�।�

3. LIC इसक��प�शन�फ�ड�मेनेजर�होगी�और�इस�योजना�के�अंतग�त�प�शन�भुगतान�हेतु��ज�मेदार�होगी।�

उपयु���म��से�कौन�से�कथन�स�य�ह��?

a) केवल�1 और�2
b) केवल�2 और�3
c) केवल�1 और�3
d) उपयु���सभी�
Correct Answer: D
Your Answer:
Explanation

Solution (d)

�धानमं�ी��म�योगी�मानधन�योजना�क��घोषणा��व��मं�ी��ी��पयूष�गोयल��ारा�2019 के�बजट�म��क��गई�थी।��धानमं�ी��म�योगी�मानधन
योजना�के�अंतग�त�असंग�ठत��े��के�वे�कामगार��जनक��मा�सक�आय�15000 �पए�से�कम�है, लाभा��वत�ह�गे।�मा��100 �पए�मा�सक�के
योगदान��ारा�लाभाथ��को�60 वष��आयु�के�प�ात�3000 �पए�मा�सक�प�शन�का�लाभ��मलेगा।��धानमं�ी��म�योगी�मानधन�योजना�को
आ�थ�क��प�से�कमजोर�तबक��हेतु�सामा�जक�सुर�ा�प�शन�योजना�के�नाम�से�भी�जाना�जाता�है।�

PM-SYM योजना�एक�क���सरकार�क��योजना�है��जसे��म�एवं�रोजगार�मं�ालय��ारा��शा�सत��कया�जाता�है।�

�धानमं�ी��म�योगी�मानधन�योजना�भारतीय�जीवन�बीमा��नगम�एवं�CSC ईगवन�स�स�व�सेज�इं�डया��ल�मटे ड�(CSC SPV) के�मा�यम�से


काया���वत�क��जा�रही�है।�

LIC इसक��प�शन�फ�ड�मेनेजर�होगी�और�इस�योजना�के�अंतग�त�प�शन�भुगतान�हेतु��ज�मेदार�होगी।�

PM-SYM प�शन�योजना�को�भारत�सरकार��ारा��च��त��नवेश��ा�प�के�अनुसार��नवे�शत��कया�जायेगा।�

य�द�याचक�10 वष��से�कम�क��अव�ध�म��योजना�से�बाहर�आता�है, उसे�उसके��ह�से�का�योगदान�केवल�बचत�ब�क��याज�दर�के�साथ�लौटा��दया


जाएगा।�

QUESTION 36.
नेशनल��रल�इकनो�मक��ांसफॉम�शन��ोजे�ट�(NRETP) के�काया��वयन�हेतु�भारत�ने��न�न�ल�खत�म��से�कौन�से�दे श�के�साथ�साझीदारी�क�
है�?

a) �व��ब�क�
b) ए�शयाई��वकास�ब�क�
c) यूरोपीय�संघ�
d) जापान�
Correct Answer: A
Your Answer:
Explanation

Solution (a)

क���य�कै�बनेट�ने�द�नदयाल�अं�योदय�योजना-रा�ीय��ामीण�आजी�वका��मशन�(DAY-NRLM) के�अंतग�त��व��ब�क�से��ा�त�ऋण�सहायता
से�ए�सटन�ली�ए�डड��ोजे�ट��जसका�नाम�“ नेशनल��रल�इकनो�मक��ांसफॉम�शन��ोजे�ट�(NRETP) है, को�अनुमो�दत��कया�है।�

IASbaba
Web: http://ilp.iasbaba.com/ Score:
Email: ilp@iasbaba.com 0.00 / 198
Page 103
2019 - Test 32 -
Exam Title :
Government...
Email :
Contact :

NRETP �ारा��द��तकनीक��सहायता�और�प�रयोजना��ारा��दए�गए�उ�चतर��तर�के�ह�त�ेप�आजी�वका��ो�साहन�तथा��व��तक�प�ँच�को
बढ़ाएंगे�एवं��ड�जटल��व��व�आजी�वका�ह�त�ेप��पर�पहल��को�ऊपर�ले�जाएंगे।�

DAY-NRLM सबसे�गरीब�एवं�सवा��धक�सुभे��समुदाय��को�ल��त�करने�एवं�उनके��व�ीय�समावेशन�पर��वशेष�जोर�दे ता�है।�NRETP के


अंतग�त��व�ीय�समावेशन�के�वैक��पक�मा�यम��को�आगे�ले�जाने�के��लए�नवाचारी�प�रयोजनाएं�शु��क��जाएंगी�जो��ामीण�उ�पाद��के�इद� �गद�
वै�यू�चे�स��न�म�त�कर�गे, आजी�वका��ो�साहन�म��नवाचारी�मॉड�स�व��व��तक�प�ँच�लाय�गे�तथा��ड�जटल��व��व�आजी�वका�ह�त�ेप��पर
पहल��को�ऊपर�ले�जाएंगे।�DAY-NRLM पंचायती�राज�सं�थान��(PRIs) एवं�समुदाय�आधा�रत�संगठन��(CBOs) के�बीच�आपसी
लाभकारी�काय�कारी�संबंध�एवं�परामश��हेतु�औपचा�रक��लेटफॉ�स��का��ावधान�करता�है।�NRLM ने�ह�त�ेप��के��व�भ���े���म��अ�भसरण
को�सु�वधाजनक�बनाने�हेतु�ग�त�व�ध�मान�च��को�भी��वक�सत��कया�है�जहाँ�NRLM सं�थान�तथा�PRIs साथ�काय��कर�सक��तथा��जसे
सभी�रा�य��ामीण�आजी�वका��मशन��तक��सा�रत�कर��दया�गया�है।�

QUESTION 37.
भारत�को�जानो�काय��म�के�बारे�म���न�न�ल�खत�म��से�कौन�से�कथन�सही�ह��?

1. �वदे श�मामले�मं�ालय�चय�नत�साझीदार�रा�य�के�साथ��मलकर�इस�काय��म�का�काया��वयन�करता�है।�

2. यह�काय��म�अ�नवासी�भारतीय��स�हत�भारतीय�मूल�के�युवा��हेतु�खुला�है�तथा�इसके�अंतग�त��गर�म�टया�दे श��के�युवा��को��ाथ�मकता
द��जाती�है।�

3. यह�योजना�के�भाग�के��प�म���नातक�क���यूनतम�यो�यता�अथवा��नातक�म��भत���ए�होने�को�आव�यक�बनाता�है।�

उपयु���म��से�कौन�से�कथन�स�य�ह��?

a) केवल�1 और�2
b) केवल�2 और�3
c) केवल�1 और�3
d) उपयु���सभी�
Correct Answer: C
Your Answer:
Explanation

Solution (c)

भारत�को�जानो�काय��म�(KIP) डाय�पोरा�के�साथ�संपक��बढाने�हेतु�एक��लैग�शप�काय��म�है।�इस�काय��म�का�उ�े �य�भारतीय�मूल�के


युवा��(18-30 वष��) को�उनक��भारतीय�जड़��तथा�वत�मान�भारत�के�साथ�जोड़ना�है।�

भारत�को�जानो�काय��म�डाय�पोरा�के�साथ�एक�तीन�स�ताह�का�ओ�रएंटेशन�काय��म�है।�इस�काय��म�का�उ�े �य�भारत�म��जीवन�के��व�वध
�प��एवं�दे श��ारा��व�भ���े���जैसे�आ�थ�क, औ�ो�गक, �श�ा, �व�ान�एवं��ौ�ो�गक�, संचार�एवं�सूचना��ौ�ो�गक��तथा�सं�कृ�त�म��क��गई
�ग�तय��के�बारे�म��जाग�कता�पैदा�करना�है।�

यह�काय��म�भारतीय�मूल�के�युवा��(अ�नवासी�भारतीय��के�अलावा) हेतु�खुला�है�तथा�इसके�अंतग�त��गर�म�टया�दे श��(मॉ�रशस, �फजी,


सूरीनाम, गुयाना, ���नदाद�व�टोबेगो, जमैका�आ�द) के�युवा��को��ाथ�मकता�द��जाती�है।�यह�योजना�के�भाग�के��प�म���नातक�क���यूनतम
यो�यता�अथवा��नातक�म��भत���ए�होने�को�आव�यक�बनाता�है।�

�वदे श�मामले�मं�ालय�चय�नत�साझीदार�रा�य�के�साथ��मलकर�इस�काय��म�का�काया��वयन�करता�है।�

QUESTION 38.
�न�न�ल�खत�म��से�कौन�सही�सुमे�लत�ह��?

योजना�रा�य�

1. रैयत�बंधु�A) आं���दे श�

IASbaba
Web: http://ilp.iasbaba.com/ Score:
Email: ilp@iasbaba.com 0.00 / 198
Page 104
2019 - Test 32 -
Exam Title :
Government...
Email :
Contact :

2. �वयं�स�ा�B) प. बंगाल�

3. ख़ुशी�C) म�य��दे श�

सही�कूट�चु�नए-

a) केवल�1 और�2
b) केवल�2 और�3
c) केवल�2
d) केवल�1 और�3
Correct Answer: C
Your Answer:
Explanation

Solution (c)

1) ओ�डशा�सरकार�ने�ख़ुशी�योजना�शु��क��है।�

इसका�उ�े �य�रा�य�भर�म���कूली�क�या��को�सेनेटरी�नैप�क�स�दे ना�है।�

2) रैयत�बंधु�योजना�तेलंगाना�सरकार��ारा�शु��क��गई�है।�

रैयत�बंधु�का�अथ��है��कसान�का��म�।�

रैयत�बंधु�(कृ�ष��नवेश�समथ�न�योजना) ��येक��कसान�क��आर��भक��नवेश�आव�यकता��का��यान�रखती�है।�

यह���येक��कसान�को���येक�मौसम�म��बीज, उव�रक, क�टनाशक, �म�एवं�अ�य��नवेश��जैसी�आगत��हेतु�4000 �पए���त�एकड़�का


अनुदान�दे ती�है।�

3) �वयं�स�ा�योजना�प. बंगाल�सरकार��ारा�आरंभ�क��गई�थी।�

इस�योजना�का�उ�े �य�युवा�लड़क��और�लड़�कय��को�सू�चत�चयन�हेतु�सश��बनाना�है�ता�क�वे���ा�पार�व�बाल��ववाह�के���त�कम�सुभे��रह�।

�वयं�स�ा�का�अथ��है��व-�व�ास�तथा�इसे�प. बंगाल�क��पु�लस��ारा�काया���वत��कया�जायेगा।�

QUESTION 39.
�कसान�स�मान��न�ध�योजना�से�कौन�लाभा��वत�होगा�?

a) संबं�धत�रा�य/संघ�शा�सत��दे श�के�भू-लेख��के�अनुसार�2 हे�टे यर�तक�क��जोतयो�य�भू�म�वाले�छोटे �एवं�सीमांत��कसान�।


b) संबं�धत�रा�य/संघ�शा�सत��दे श�के�भू-लेख��के�अनुसार�4 हे�टे यर�तक�क��जोतयो�य�भू�म�वाले�छोटे �एवं�सीमांत��कसान�।
c) संबं�धत�रा�य/संघ�शा�सत��दे श�के�भू-लेख��के�अनुसार�5 हे�टे यर�तक�क��भू�म�वाला�कोई��कसान�।
d) संबं�धत�रा�य/संघ�शा�सत��दे श�के�भू-लेख��के�अनुसार�6 हे�टे यर�तक�क��गैर-कृ�ष�भू�म�वाला�कोई��कसान�।
Correct Answer: A
Your Answer:
Explanation

Solution (a)

2019 के�अंत�रम�बजट�ने�एक��कसान�आय�समथ�न�योजना�पीएम��कसान�स�मान��न�ध�योजना�(PM-KISAN) क��घोषणा�क��है।�पीएम


�कसान�स�मान��न�ध�योजना�क���वशेषता�म��छोटे �व�सीमांत��कसान��को���त�वष��6,000 �पए�क��रा�श�ह�तांत�रत�करना�शा�मल�है।�

6000 �पए�क��सहायता�पा���कसान��के�खात��म��2000 �पए�क��तीन�बराबर��क�त��म���दान�क��जाएगी।�योजना�के�अंतग�त�, उन��कसान


प�रवार��को�भी���तवष��6,000 �पये��दए�जाय�गे��जनक��संयु��जमीन/�वा�म�व�दो�हे�टे यर�तक�है।�यह�रा�श�तीन�बराबर��क�त��म��लाभाथ�

IASbaba
Web: http://ilp.iasbaba.com/ Score:
Email: ilp@iasbaba.com 0.00 / 198
Page 105
2019 - Test 32 -
Exam Title :
Government...
Email :
Contact :
के�खाते�म����य��लाभ�अंतरण�(DBT) योजना�के�अंतग�त�भेजी�जाएगी।���य��लाभ�अंतरण����या�म��पारद�श�ता�सु�न��त�करेगा�और
�कसान��हेतु�समय�बचाएगा।�

यह�योजना�छोटे �एवं�सीमा�त��कसान��(SMFs) क��आय�को�बढाने�का�ल�य�रखती�है।�यह���येक�SMFs को�समु�चत�फसल��वा��य


सु�न��त�करने�हेतु��व�भ��आगत���ा�त�करके�(जो���येक�फसल�च��के�अंत�म��अनुमा�नत�कृ�ष�आय�के�बराबर�हो) अपनी��व�ीय�ज�रत��पूरा
करने�म��मदद�करती�है।�यह�योजना��कसान��को�ऐसे��य��क��पूत��हेतु�सा�कार��के�चंगुल�म��फंसने�से�भी�बचाएगी�तथा�उनक��कृ�ष
ग�त�व�धय��का�जारी�रहना�सु�न��त�करेगी।�

SMFs भू�मधारक��कसान�प�रवार�को�अ��ल�खत�ढं ग�से�प�रभा�षत��कया�गया�है�“ एक�ऐसा�प�रवार��जसम��प�त, प�नी�और�अ��क�ब�चे


ह��जो�संयु���प�से�संबं�धत�रा�य/संघ�शा�सत��दे श�के�भू-लेख��के�अनुसार�2 हे�टे यर�तक�क��भू�म�का��वा�म�व�रखते�ह�।�

QUESTION 40.
�मशन�र�ा��ान�श���के�बारे�म���न�न�ल�खत�कथन��पर��वचार�क��जए।�

1. इसका�गठन�र�ा�उ�पादन��वभाग��ारा��कया�गया�है।�

2. इसका�उ�े �य��वदे शी�र�ा�उ�ोग�म��IPR सं�कृ�त�को�बढ़ावा�दे ना�है।�

3. डायरे�टरेट�जनरल�ऑफ़��वा�लट��ए�योर�स�(DGQA) को�इस�काय��म�के�सम�वय�और�काया��वयन�क���ज�मेदारी�द��गई�है।�

सही�कूट�चु�नए-

a) केवल�1 और�2
b) केवल�2 और�3
c) केवल�1 और�3
d) उपयु���सभी�
Correct Answer: D
Your Answer:
Explanation

Solution (d)

क���य�र�ा�मं�ी��नम�ला�सीतारमण�ने�27 नवंबर�2018 को�नई��द�ली�म��औपचा�रक��प�से��मशन�र�ा��ान�श���शु���कया।�

�मशन�र�ा��ान�श���को�र�ा�उ�पादन��वभाग��ारा�र�ा��े��म���व-�नभ�रता�बढाने�क��वत�मान�पहल��के�एक�भाग�के��प�म��शु���कया�गया
है।�

�मशन�का�उ�े �य��वदे शी�र�ा�उ�ोग�म��IPR सं�कृ�त�को�बढ़ावा�दे ना�है।�

डायरे�टरेट�जनरल�ऑफ़��वा�लट��ए�योर�स�(DGQA) को�इस�काय��म�के�सम�वय�और�काया��वयन�क���ज�मेदारी�द��गई�है।�

का��कम�का�अं�तम�उ�े �य�भारतीय�र�ा��व�नमा�ण�पा�रतं��म��IP सं�कृ�त�को�आ�मसात�करवाना�है।�

QUESTION 41.
मानव�संसाधन��वकास�मं�ालय�ने�‘सम���श�ा’ नामक�एक�योजना�आरंभ�क��है।�इसके�बारे�म���न�न�ल�खत�म��से�कौन�से�कथन�सही�ह��?

1. इसने�सव���श�ा�अ�भयान�(SSA), रा�ीय�मा�य�मक��श�ा�अ�भयान�(RMSA) तथा�ट�चस��एजुकेशन�(TE) नामक�तीन�योजना��को


अपने�म��समा�हत�कर��लया�है।�

2. यह��कूली��श�ा�को�सम��तथा��ी-�कूल�से�क�ा�12 तक��नरंतरता�के��प�म��लेती�है।�

3. अ�धगम�प�रणाम�के�सुधार�पर�जोर�होगा।�

�न�न�ल�खत�म��से�कूट�चु�नए-

IASbaba
Web: http://ilp.iasbaba.com/ Score:
Email: ilp@iasbaba.com 0.00 / 198
Page 106
2019 - Test 32 -
Exam Title :
Government...
Email :
Contact :

a) केवल�1 और�2
b) केवल�2 और�3
c) केवल�1 और�3
d) उपयु���सभी�
Correct Answer: D
Your Answer:
Explanation

Solution (d)

सम���श�ा�

के���य�बजट�2018-19 ने��ी-नस�री�से�क�ा�12 तक��बना��वभाजन��कये��कूली��श�ा�को�सम��ढं ग�से�दे खने�का���ताव��कया�है।�सम�


�श�ा�- �ी-�कूल�से�क�ा�12 तक��कूली��श�ा��े��म��एक�मह�वपूण��काय��म�है��जसे��कू�ल�ग�के�समान�अवसर��एवं�समतापरक�अ�धगम
प�रणाम��के��प�म��आंक�लत��कूल��भा�वत�को�सुधारने�के�वृहत�ल�य�के�साथ�बनाया�गया�है।�इसने�सव���श�ा�अ�भयान�(SSA), रा�ीय
मा�य�मक��श�ा�अ�भयान�(RMSA) तथा�ट�चस��एजुकेशन�(TE) नामक�तीन�योजना��को�अपने�म��समा�हत�कर��लया�है।�

�श�ा�म��सम�����कोण�

• क�ा�I से�XII तक��कूल��श�ा��े��हेतु�अकेली�योजना�- ह�त�ेप��का�व�र��मा�य�मक�चरण�तक��व�तार�।


• यह��कूली��श�ा�को�सम��तथा��ी-�कूल�से�क�ा�12 तक��नरंतरता�के��प�म��लेती�है।�
• रा�य��क���ी-�ाइमरी��श�ा�पहल��म��समथ�न�करती�है�।
• पहली�बार��कूल��श�ा�के�समथ�न�हेतु�व�र��मा�य�मक��तर��तथा��ी-�ाइमरी��तर��का�समावेशन�।

�शास�नक�सुधार�

• एकल�एवं�एक�कृत��शास�नक�ढांचा�जो�सु�व��थत�काया��वयन�तक�जाता�है�।
• योजना�के�अंतग�त�रा�य��को�अपने�ह�त�ेप��क���ाथ�मकता��नधा��रत�करने�म��लोचशीलता�द��गई�है�।
• एक�एक�कृत��शासन�जो�‘�कूल’ को�एक��नरंतरता�के��प�म��दे खता�है�।

�श�ा�हेतु�वृ��त�कोष�

• बजट�बढ़ा��दया�गया�है�।
• अ�धगम�प�रणाम�एवं�गुणव�ा�सुधार�हेतु�उठाए�गए�कदम�इस�योजना�के�अंतग�त�अनुदान�आवंटन�का�आधार�बन�गे�।

�श�ा�क��गुणव�ा�पर��यान�

• अ�धगम�प�रणाम��के�सुधार�पर�जोर�
• �श�क��का�वृ��त��मता��नमा�ण�
• �श�क��श�ा�सं�थान��जैसे�SCERTs एवं�DIETs को�मजबूत�बनाने�पर�जोर�ता�क��णाली�म��संभावना�संप���श�क��क��गुणव�ा
को�सुधारा�जा�सके�।
• SCERT इन-स�व�स�एवं��ी-स�व�स��श�क���श�ण�हेतु�नोडल�सं�थान�होगा�– यह���श�ण�को�ग�तशील�एवं�आव�यकता-आधा�रत
बनाएगा�।
• गुणव�ा��श�ा�पर��वशेष��यान��जसम��ऑनलाइन�तथा�ऑफलाइन��व�ध�से��श�क��के��मता��नमा�ण�पर�जोर�।�साथ�ही�SCERT/
DIET/BRC/CRC/CTEs/IASEs जैसे��श�क��श�ा�सं�थान��को�मजबूत�बनाना।�
• पु�तकालय��को�मजबूत�बनाने�हेतु���येक��कूल�को�वा�ष�क�अनुदान�।
• लगभग�10 लाख��कूल��को�पु�तकालय�अनुदान��दया�जाना�है�।
• T से�ट�चर�एवं�टे �नोलॉजी�पर��यान�के���त�करने�के�मा�यम�से��श�ा�क��गुणव�ा�सुधारने�पर�वृ��त��यान�।
• संसाधन��का�प�रणाम�उ�मुखी�आवंटन�।

�ड�जटल��श�ा�पर��यान�

• 5 वष��क��समयाव�ध�म��सभी�सेक�डरी��कूल��म��‘ ऑपरेशन��ड�जटल�बोड��’ को�समथ�न�जो��श�ा�म���ां�त�ला�दे गा�– यह�समझने�म�


आसान�होगा�तथा��ौ�ो�गक��आधा�रत�अ�धगम�क�ाएं���लपड�क�ाएं�बन�जाय�गी�।
• �माट� �क�ा�, �ड�जटल�बोड�, एवं�DTH चेनल��के�मा�यम�से��ड�जटल��ौ�ो�गक��का�वृ��त��योग�।

IASbaba
Web: http://ilp.iasbaba.com/ Score:
Email: ilp@iasbaba.com 0.00 / 198
Page 107
2019 - Test 32 -
Exam Title :
Government...
Email :
Contact :
• शाला�कोष, शगुन, शाला�सारथी�जैसी��ड�जटल�पहल��को�मजबूत�बनाया�जायेगा�।
• �कूल��म��अपर��ाइमरी��तर�से�हायर�सेक�डरी��तर�तक�ICT अवसंरचना�को�मजबूत�बनाया�जाएगा�।
• “ द��ा�”, जो��क��श�क��के��लए�एक��ड�जटल�पोट� ल�है, का��श�क��के�कौशल�को�अ�तन�बनाने�हेतु��ापक��योग��कया�जाएगा�

• प�ँच�बढाने�एवं�गुणव�ा��श�ा�के��ावधान�हेतु��ौ�ो�गक��का�वृ��त��योग�– ‘ सबक���श�ा-अ�छ���श�ा�’

�कूल��को�मजबूत�बनाना�

• गुणव�ा�के�सुधार�हेतु��कूल��के�समेकन�पर�जोर�
• �कूल�तक�साव�भौम�प�ँच�हेतु�क�ा�I से�VIII तक�सभी�क�ा��के�ब�च��हेतु�वृ��त�प�रवहन�सु�वधा�।
• �कूल��के�बु�नयाद��ढांचे�क��मजबूती�हेतु�वृ��त�आवंटन�
• संयु���कूल�अनुदान�बढ़��तथा�इसे��कूल�भत��के�आधार�पर�आवं�टत��कया�जाना�है�
• �व�छता�ग�त�व�धय��हेतु��व�श���वाधान�– ‘ �व�छ��व�ालय’ को�समथ�न�
• सरकारी��कूल��म��अवसंरचना�क��गुणव�ा�को�सुधारना�

क�या��श�ा�पर��यान�

• क�या��का�सश��करण�
• KGBVs का�दजा��क�ा�6-8 से�बढ़ाकर�क�ा�6-12 तक�करना�
• अपर��ाइमरी�से�हायर�सेक�डरी��तर�तक�क�या��हेतु�आ�म-र�ा���श�ण�
• CWSN क�या��हेतु��टाईप�ड�क�ा�I से�XII तक��दया�जाएगा�– पहले�इसे�केवल�क�ा�IX से�क�ा�XII तक��दया�जाता�था�।
• ‘ बेट��बचाओ�बेट��बचाओ�के���त�वृ��त���तब�ता�

समावेशन�पर��यान�

• ��त�ब�चा���त�वष��RTE के�अंतग�त�यू�नफाम��हेतु�आवंटन�बढ़ा�।
• RTE के�अंतग�त���त�ब�चा���त�वष��पा�पु�तक�हेतु�आवंटन�बढ़ा�।�उजा�वान�पा�पु�तक��को�लाया�जायेगा।�
• �व�श��आव�यकता�वाले�ब�च��(CwSN) हेतु�आवंटन�बढ़ाकर�3000 �पए�से�3500 �पए���त�बालक���त�वष���कया�गया�। क�ा
1 से�12 क���व�श��आव�यकता�वाली�क�या��हेतु�200 �पये���त�माह��टाईप�ड�।
• ‘ सबको��श�ा�अ�छ���श�ा’ के���त���तब�ता�

कौशल��वकास�पर��यान�

• अपर��ाइमरी��तर�पर�वोकेशनल���क�स�क��ओर�अनावरण�को��व�ता�रत��कया�जाएगा�।
• सेक�डरी��तर�पर�वोकेशनल��श�ा�को�क�रकुलम�के�एक�अखंड�भाग�के��प�म��मजबूत��कया�जाएगा�।
• वोकेशनल��श�ा�जो��क�क�ा�9-12 तक�सी�मत�थी, को�क�ा�6 से�क�रकुलम�के�एक�अखंड�भाग�के��प�म��शु���कया�जाएगा�तथा
इसे�अ�धक��ायो�गक�एवं�उ�ोग�मुखी�बनाया�जाएगा�।
• ‘ कौशल��वकास�’ पर�पुन: �यान�

खेल�एवं�शारी�रक��श�ा�पर��यान�

• इस�अवयव�के�अंतग�त�सभी��कूल��को�खेल�उपकरण��दान��कया�जाय�गे�।
• खेल��श�ा�को�पा��म�का�एक�अ�भ��भाग�बनाया�जाएगा�।
• सभी��कूल��को�योजना�के�तहत�खेल�उपकरण��मल�गे�ता�क��कूल�पा��म�म��खेल��क���ासं�गकता�को�आ�मसात�व�उस�पर�जोर
�दया�जा�सके�।
• ‘ खेलो�इं�डया�’ को�समथ�न�

�े�ीय�संतुलन�पर��यान�

• संतु�लत��श�ा��वकास�को��ो�साहन�
• शै��णक��प�से��पछड़े��लॉ�स�(EBBs), LWEs, �पेशल�फोकस��ड����स�(SFDs), सीमा��े���तथा�नी�त�आयोग��ारा�पहचाने
गए�115 आकां�ी��जल��पर��यान�
• ‘ सबका�साथ�सबका��वकास�’ तथा�सबको��श�ा�अ�छ���श�ा�

QUESTION 42.

IASbaba
Web: http://ilp.iasbaba.com/ Score:
Email: ilp@iasbaba.com 0.00 / 198
Page 108
2019 - Test 32 -
Exam Title :
Government...
Email :
Contact :

�क�म�टू ��ोवाइड��वा�लट��एजुकेशन�इन�मदरसा�(SPQEM) के�बारे�म���न�न�ल�खत�कथन��पर��वचार�क��जए।�:

1. इसे�अ�पसं�यक�मामले�मं�ालय�ने�आरंभ��कया�है।�

2. बढे ��ए��श�क�मानदे य�के�भुगतान�के�मा�यम�से��व�ान, ग�णत, भाषा, सामा�जक�अ�ययन�आ�द�औपचा�रक�क�रकुलम��वषय��के


अ�यापन�हेतु�मदरस��म���मता��को�मजबूत�करना।�

3. मदरस��के�सभी��तर��पर�पु�तकालय�/पु�तक�ब�क��को�मजबूत�करना�एवं��श�क�अ�धगम�साम��यां�उपल�ध�करवाना।�

योजना�के�बारे�म��उपयु���म��से�कौन�से�कथन�स�य�ह��?

a) केवल�1 और�2
b) केवल�2 और�3
c) केवल�1 और�3
d) उपयु���सभी�
Correct Answer: B
Your Answer:
Explanation

Solution (b)

�क�म�टू ��ोवाइड��वा�लट��एजुकेशन�इन�मदरसा�(SPQEM)
SPQEM का�उ�े �य�मदरस��म��गुणव�ा�सुधार�लाना�है�ता�क�मु��लम�ब�चे�रा�ीय��श�ा��णाली�के�औपचा�रक��श�ा��वषय��म��मानक��को
�ा�त�कर�सक��। SPQEM योजना�क��मु�य��वशेषताएं�ह��:

• बढे ��ए��श�क�मानदे य�के�भुगतान�के�मा�यम�से��व�ान, ग�णत, भाषा, सामा�जक�अ�ययन�आ�द�औपचा�रक�क�रकुलम��वषय��के


अ�यापन�हेतु�मदरस��म���मता��को�मजबूत�करना।�
• नए��श�ण�अ�यास��म����येक�दो�वष��पर�ऐसे��श�क��का���श�ण।�
• मा�य�मक�एवं�उ�चतर�मा�य�मक��तर�मदरस��म��वा�ष�क�रख-रखाव�लागत�स�हत�साइंस�लैब, कं�यूटर�लैब��दान�करना।�
• �ाथ�मक/उ�च��ाथ�मक��तर�के�मदरस��म���व�ान/ग�णत��कट��का��ावधान।�
• मदरस��के�सभी��तर��पर�पु�तकालय�/पु�तक�ब�क��को�मजबूत�करना�एवं��श�क�अ�धगम�साम��यां�उपल�ध�करवाना।�
• इस��पांत�रत�योजना�क���व�श���वशेषता�यह�है��क�यह�मदरस��का�नेशनल�इं��ट�ूट��फॉर�ओपन��कू�ल�ग�(NIOS) (औपचा�रक
�श�ा��दान�करने�हेतु��ामा�णक�के����के�तौर�पर) के�साथ�स�पक��को��ो�सा�हत�करती�है�जो�मदरस��म��पढ़�रहे�ऐसे�ब�च��को�क�ा�
5, 8, 10 एवं�12 हेतु��माणप��पाने�म��स�म�बनाएगी।�यह�उ�ह��उ�चतर��श�ा�क��ओर�गमन�करने�म��स�म�बनाएगी�तथा�सु�न��त
करेगी��क�उ�ह��रा�ीय��श�ा��णाली�के�स�श�गुणव�ा�मानक��मल�।�NIOS को�द��जाने�वाली�पंजीकरण�एवं�परी�ा�फ�स�के�साथ-
साथ��योग�क��जाने�वाली��श�ण�अ�धगम�साम��यां�इस�योजना�के�अंतग�त�कवर�क��जाएंगी।�
• योजना�के�तहत�NIOS स�पक��को�मदरस��के�मा�य�मक�एवं�उ�चतर�मा�य�मक��तर�पर�वोकेशनल��श�ा�तक��व�ता�रत��कया
जायेगा।�
• योजना�क���नगरानी�एवं�उसे�लोक��य�बनाने�हेतु�यह�रा�य�मदरसा�बोड��को��व�पो�षत��कया�जायेगा।�भारत�सरकार��वयं�आव�धक
मू�यांकन�करेगी��जसम���थम�दो�वष��के�भीतर��कया�जाएगा।�

नोट�: योजना�को�MHRD �ारा�शु���कया�गया�है�न��क�अ�पसं�यक�मामले�मं�ालय��ारा।�

QUESTION 43.
‘सा�र�भारत’ के�बारे�म���न�न�ल�खत�म��से�कौन�से�कथन�सही�ह��?

1. इस�का��कम�का�उ�े �य�रा�ीय��तर�पर�80% सा�रता��तर��ा�त�करना�है।�

2. इस�काय��म�का��धान�ल�य�7 करोड़��नर�र�वय�क��म��(�जनक��आयु�15 वष��या�अ�धक�है) काय�शील�सा�रता�लाना�है�।

3. यह�काय��म�केवल��ामीण��े���के�पा���जल��म��कवरेज��दान�करता�है।�

IASbaba
Web: http://ilp.iasbaba.com/ Score:
Email: ilp@iasbaba.com 0.00 / 198
Page 109
2019 - Test 32 -
Exam Title :
Government...
Email :
Contact :

�न�न�ल�खत�म��से�कूट�का�चयन�क��जए�:

a) केवल�1 और�2
b) केवल�2 और�3
c) केवल�1 और�3
d) उपयु���सभी�
Correct Answer: D
Your Answer:
Explanation

Solution (d)

सा�र�भारत�

सा�र�भारत�काय��म�‘3’ R’ ( अथा�त�री�ड�ग�, राइ�ट�ग�एवं�अथ�मे�टक�) से�आगे�जाती�है��य��क�यह�सुधार�तथा�सामा�य�क�याण�हेतु


सामा�जक�असमानता��एवं�एक�����क��साधन�वहीनता�पर�जाग�कता�भी�पैदा�करती�है।�यह�काय��म�2009 म����क�म�हला�सा�रता
पर��यान�क���त�कर�रा�ीय��तर�पर�80% सा�रता�हा�सल�करने�के��लए�बनाया�गया�था�ता�क�पु�ष�और�म�हला�सा�रता�के�अंतर�को�10 �
�तशत��ब���से�अ�धक�तक�कम��कया�जा�सके।�इसके�चार�वृहत�उ�े �य�भी�ह�, गैर-सा�र��म��काय�शील�सा�रता�और�अंक�य�समझ�पैदा�करना�;
औपचा�रक��श�ा��णाली�म��समानता�हा�सल�करना�; �ासं�गक�कौशल��वकास�काय��म�लाना�तथा��नरंतर��श�ा�हेतु�अवसर��दान�कर�एक
अ�धगम�समाज�को��ो�सा�हत�करना।�काय��म�का��दान�ल�य�7 करोड़��नर�र�वय�क��म��(�जनक��आयु�15 वष��या�अ�धक�है) काय�शील
सा�रता�लाना�है।�इसके�अंतग�त�14 �म�लयन�अनुसू�चत�जा�तय��(SCs), 8 �म�लयन�अनुसू�चत�जनजा�तय��(STs), 12 �म�लयन
अ�पसं�यको�तथा�36 �म�लयन�अ�यो�को�कवर�करना�शा�मल�है।�म�हला��क��सम��कवरेज�का�ल�य�6 करोड़�है।�दे श�के�27 रा�य��/UTs
के�410 �जल��को�सा�र�भारत�के�तहत�कवर�करने�के��लए�पहचाना�गया�था।�

सा�र�भारत�के�अंतग�त�कवरेज�हेतु�पा�ता�- एक��जला�(�जसम��पूव�वत���जले�से�अलग�कर�बनाया�गया�नया��जला�शा�मल�है) जहाँ�वय�क


म�हला�सा�रता�दर�50 ��तशत�या�उससे�कम�है�( 2001 क��जनगणना�के�अनुसार) को�सा�र�भारत�काय��म�के�अंतग�त�कवरेज�हेतु�पा�
समझा�गया�था।�इसके�अ�त�र��, सभी�वाम-उ�वाद��भा�वत��जले�(भले�उनक��सा�रता�दर�कुछ�भी�हो) भी�इस�काय��म�के�तहत�कवरेज�के
पा��थे।�दे श�म��कुल�365 �जले�थे��जनक��वय�क�म�हला�सा�रता�दर�50 ��तशत�या�उससे�कम�थी।�गृह�मं�ालय�ने�35 �जल��को�वाम-
उ�वाद��भा�वत�घो�षत��कया�था।�हालां�क�, वाम-उ�वाद��भा�वत�30 �जल��क��वय�क�म�हला�सा�रता�दर�50 ��तशत�या�उससे�कम�थी।
आरंभ�म��370 �जल��क��वय�क�म�हला�सा�रता�दर�50 ��तशत�या�उससे�कम�( 2001 क��जनगणना�के�अनुसार) थी, जो�इस�काय��म�के
अंतग�त�करवेज�के�पा��थे।�2001 से�, अनेक�पा���जल��को�दो�अथवा�तीन��ह�स��म��बांटा�जा�चुका�है।�इसने�पा���जल��क��सं�या�बढ़ा�( 35
वाम-उ�वाद��भा�वत��जल��स�हत) द��है।�यह�काय��म�केवल��ामीण��े���के�पा���जल��म��कवरेज��दान�करता�है।�

QUESTION 44.
�टे ट��रसोस��स�टस��(SRCs) को�वय�क��को�शै��क�और�तकनीक��संसाधन�समथ�न��दान�करने�एवं�साम�ी�व���श�ण�मो�ू�स�के��वकास
तथा�उ�पादन�के�मा�यम�से��श�ा�जारी�रखने�का�अ�धदे श��ा�त�है।��न�न�ल�खत�म��से�कौन�इसके�काय��े��म��आता�है�?

1. सा�रता�पदा�धका�रय��का���श�ण�

2. सा�रता�प�रयोजना��का�मू�यांकन�एवं��नगरानी�

3. उ�च��ौ�ो�गक�य�शोध�हेतु�अवसंरचना��दान�करना�

�न�न�ल�खत�म��से�कूट�का�चयन�क��जए�:

a) केवल�1 और�2
b) केवल�2 और�3
c) केवल�1 और�3
d) उपयु���सभी�
Correct Answer: A
Your Answer:
Explanation

IASbaba
Web: http://ilp.iasbaba.com/ Score:
Email: ilp@iasbaba.com 0.00 / 198
Page 110
2019 - Test 32 -
Exam Title :
Government...
Email :
Contact :

Solution (a)

�टे ट��रसोस��स�टस��(SRCs) को�वय�क��को�शै��क�और�तकनीक��संसाधन�समथ�न��दान�करने�एवं�साम�ी�व���श�ण�मो�ू�स�के��वकास


तथा�उ�पादन�के�मा�यम�से��श�ा�जारी�रखने�का�अ�धदे श��ा�त�है।�इसके�अलावा�SRC को��ेरणादायक�एवं�पया�वरण��नमा�ण, काय��शोध�एवं
मू�यांकन�तथा��नगरानी�करने�ह�गे�।

काय���े��
• वय�क��श�ा�काय��म�हेतु��श�ण, अ�धगम�एवं���श�ण�साम�ी�तैयार�करना।�
• सा�रता�पदा�धका�रय��का���श�ण�
• काय��शोध�
• सा�रता�प�रयोजना��का�मू�यांकन�एवं��नगरानी�
• वय�क��श�ा�काय��म�क��भावी�आव�यकता��क��पहचान�हेतु�नवाचारी�प�रयोजना��को�हाथ�म��लेना�
• अ�य�कोई�काय��जो�NLM के�कुल�उ�े �य��क��पूत��हेतु�आव�यक�हो�अथवा��जसे�भारत�सरकार�के�मानव�संसाधन�मं�ालय��ारा
�वशेष��प�से�स�पा�गया�हो।�

QUESTION 45.
सामा�जक��याय�एवं�अ�धका�रता�मं�ालय�ने�द�नदयाल��डसेब�ड��रहै�ब�लटे शन��क�म�(DDRS) शु��क��है।�योजना�के�बारे�म���न�न�ल�खत
म��से�कौन�से�कथन�सही�ह��?

1. योजना�भारत�के�सभी�रा�य��तथा�संघ�शा�सत��े���म��लागू�क��जाएगी।�

2. योजना�के�अंतग�त�NGOs को��वकलांग����य��के�पुनवा�स�हेतु�प�रयोजनाएं�(जैसे��पेशल��कूल, �ी-�कूल�एवं�आरं�भक�ह�त�ेप,


हाफवे�हो�स�एवं�समुदाय�आधा�रत�पुनवा�स�आ�द) चलाने�हेतु��व�ीय�सहायता��दान�क��जाती�है।�

�न�न�ल�खत�म��से�कूट�का�चयन�क��जए�:

a) केवल�1
b) केवल�2
c) 1 और�2 दोन��
d) न�तो�1 न�ही�2
Correct Answer: B
Your Answer:
Explanation

Solution (b)

द�नदयाल��डसेब�ड��रहै�ब�लटे शन��क�म�(DDRS)

DDRS को�सामा�जक��याय�एवं�अ�धका�रता�मं�ालय�के��डपाट� म�ट�ऑफ़�ए�पावरम�ट�ऑफ़�पस��स��वद��डसे�ब�लट�ज़�(DEPwD) �ारा


संग�ठत��कया�जा�रहा�है�। यह�पूव��एवं�उ�रपूव���े��के�13 रा�य��को�कवर�करेगा�जो�ह��प. बंगाल�, झारखंड�, ओ�डशा�, छ�ीसगढ़�, �बहा
र�, असम�, अ�णाचल��दे श�, म�णपुर�, मेघालय�, �मजोरम�, नागाल�ड�, �स��कम�एवं���पुरा�।��हतधारक�अथा�त�NGOs जो�DDRS के
अंतग�त�अनुदान��ा�त�कर�रहे�ह��, रा�य�सरकार�अ�धकारी�एवं��जला��तर�के�अ�धकारी�भागीदारी�कर�गे�।

योजना�के��ावधान�

• ��येक�वष��600 से�अ�धक�NGOs को�पस��स��वद��डसे�ब�लट��के�पुनवा�स�हेतु�अपनी�प�रयोजनाएं�(जैसे��पेशल��कूल, �ी-�कूल


एवं�आरं�भक�ह�त�ेप, हाफवे�हो�स�एवं�समुदाय�आधा�रत�पुनवा�स�आ�द) चलाने�के��लए��व�ीय�सहायता��दान�क��जाती�है�।
• रा�श��ा�त�करने�वाले�NGOs ��येक�वष��35000 से�40000 लाभा�थ�य��हेतु�पुनवा�स�सेवा��का�संचालन�कर�रहे�ह��।

त�य�एवं�आंकड़े�

• जनगणना�2011 के�अनुसार�, भारत�म��लगभग�2.68 अ�म�����ह��जो�दे श�क��कुल�जनसँ�या�का� 2.21% ह��।

IASbaba
Web: http://ilp.iasbaba.com/ Score:
Email: ilp@iasbaba.com 0.00 / 198
Page 111
2019 - Test 32 -
Exam Title :
Government...
Email :
Contact :
• भारत�के�सं�वधान�का�अ�धदे श�समानता, �वतं�ता, �याय�एवं�सभी����य��क��ग�रमा�सु�न��त�करने�हेतु�है��जसका�अथ��है�सभी
(�वशेषकर�कमजोर�लोग��के��लए) के��लए�एक�समावेशी�समाज�।
• भाग�IV का�अनु�छे द�41 ( रा�य�नी�त�के��नदे शक�त�व) – �वशेष��प�से��द�ांगजन��के��लए�है�।
• 2015 म��, सरकार�ने��द�ांगजन��के�कौशल��वकास�हेतु�रा�ीय�काय��योजना�(NAP) शु��क��। NAP �व�भ���वसाय��म�
�द�ांगजन��हेतु�कौशल�क��बात�करता�है�ता�क�उनक��रोजगारपरकता�को�सरकारी�एवं��नजी�दोन���े���म��बढ़ाया�जा�सके�तथा�उनके
�व-रोजगार�क��संभावना��को�भी�बढ़ाया�जा�सके�।

QUESTION 46.
साव�ज�नक��थल��पर�म�हला��को�सुर�ा��दान�करने�हेतु�सरकार�ने�सेफ��सट���ोजे�ट�के�काया��वयन�हेतु�आठ�शहर��को��च��त��कया�है।
सेफ��सट���ोजे�ट�क���न�न�ल�खत�म��से�कौन�सी��वशेषताएं�ह��?

1. ��येक�शहर�म��संवेदनशील�हॉट��पॉट् स�क��पहचान�करना।�

2. अ�यंत�संवेदनशील��े���म��ऑटोमेटेड�नंबर��लेट�को�पढने�वाली�मशीन��क��तैनाती��कया�जाना।�

3. ���ट�लाइ�ट�ग�एवं�म�हला��हेतु�प��लक�टॉयलेट�सु�वधा��को�सुधारना।�

�न�न�ल�खत�म��से�कूट�का�चयन�क��जए�:

a) केवल�1 और�2
b) केवल�2 और�3
c) केवल�1 और�3
d) उपयु���सभी�
Correct Answer: D
Your Answer:
Explanation

Solution (d)

सेफ��सट���ोजे�ट�के�अंतग�त�SCIM पोट� ल�: साव�ज�नक��थल��पर�म�हला��को�सुर�ा��दान�करने�हेतु�सरकार�ने�सेफ��सट���ोजे�ट�के


काया��वयन�हेतु�आठ�शहर��को��च��त��कया�है।�चुने�गए�8 �मुख�शहर�ह��- अहमदाबाद�, ब�गलु��, चे�ई�, �द�ली�, हैदराबाद�, कोलकाता�, ल
खनऊ�तथा�मुंबई�। प�रयोजना��के�अंतग�त�जमीनी�प�रस�प�तय��व�संसाधन��का�सृजन�एवं�म�हला��क�����कोण�सुर�ा�शा�मल�है�।

सेफ��सट���ोजे�ट�क���मुख��वशेषताएं�ह��:

• ��येक�शहर�म��संवेदनशील�हॉट��पॉट् स�क��पहचान�करना।�
• सभी�हॉट��पॉट् स�को�कवर�करने�वाली�CCTV �नगरानी�क���थापना�।
• अ�यंत�संवेदनशील��े���म��ऑटोमेटेड�नंबर��लेट�को�पढने�वाली�मशीन��क��तैनाती��कया�जाना।�
• पहचाने�गए�हॉट��पॉट् स�के�आगे�के�सुभे���े���म��गहन�पे�ो�ल�ग�।
• ���ट�लाइ�ट�ग�एवं�म�हला��हेतु�प��लक�टॉयलेट�सु�वधा��को�सुधारना।�
• पु�लश��टे शन��म��म�हला�हे�प�डे�क�क���थापना, म�हला�समथ�न�क����को�बढ़ावा�दे ने�जैसे�अ�य�कदम�।

QUESTION 47.
�म�एवं�रोजगार�मं�ालय�ने��वज़न�जीरो�कां��स�आर�भ�क��है।�यह�संबं�धत�है-

a) �बना�बीमे�के�शू�य��म�बल�से�
b) कामगार��को�असंग�ठत��े��से�संग�ठत��े��म��लाने�से�
c) पेशेवर�सुर�ा�एवं��वा��य�से�
d) MSMEs �ारा�उ�पा�दत�व�तु��म��शू�य��ु�टय��से�
Correct Answer: C
Your Answer:
Explanation

IASbaba
Web: http://ilp.iasbaba.com/ Score:
Email: ilp@iasbaba.com 0.00 / 198
Page 112
2019 - Test 32 -
Exam Title :
Government...
Email :
Contact :

Solution (c)

�म�एवं�रोजगार�मं�ालय�ने��वज़न�जीरो�कां��स�आर�भ�क��है�तथा�यह�पेशेवर�सुर�ा�एवं��वा��य�हेतु��ासं�गक�है।�

उ�े �य�

• ‘ �वज़न�जीरो�’ क��अवधारणा�तेजी�से�वै��क��वीकाय�ता��ा�त�कर�रही�है�और�इसके�भारत�सरकार�के�दे श�म��पेशेवर�सुर�ा�एवं


�वा��य�मानक��के��लए��योग��कये�जाने�क��संभावना�है�ता�क�पेशेवर�सुर�ा�एवं��वा��य�हालात��को�सुधार�जा�सके�।
• ‘ �वज़न�जीरो�’ क��अवधारणा�चार�मौ�लक��स�ांत��पर�आधा�रत�है�अथा�त�
◦ जीवन�पर�समझौता�नह��,
◦ मानव���ारा�चूक�संभव�है�,
◦ सहन�सीमा��को�मानव�भौ�तक�अवरोध��ारा�प�रभा�षत��कया�जाता�है�तथा�
◦ लोग��को�सुर��त�प�रवहन�एवं�सुर��त�काय��थल��का�अ�धकार�है�।

यह��वज़न�जो�खम��के��नयं�ण, मशीन�, उपकरण��तथा�काय��थल��के�मामले�म��सुर�ा�व��वा��य�सु�न��त�करने�एवं�काय�बल�के�कौशल�को


अ�तन�बनाने�के��स�ांत��पर�आधा�रत�है�।

QUESTION 48.
म�हला�एवं�बाल��वकास�मं�ालय�ने�सभी�रा�य��एवं�संघ�शा�सत��े���को�शा�मल�करते��ए�पोषण�अ�भयान�पर�पहली�रा�ीय�काय�शाला�का
संचालन��कया।�पोषण�योजना�के�बारे�म���न�न�ल�खत�म��से�कौन�से�कथन�सही�ह�� ?

1. अ�भयान�का�ल�य�युवा�बालक�, म�हला��एवं��कशो�रय��म���टं �ट�ग, अ�प-पोषण, एनी�मया�को�एवं�ज�म�के�समय�कम�वजन�को���त�वष�


�मश: 50%, 50%, 80% तथा�50% तक�कम�करना�है�।

2. �मशन�का�ल�य�0-6 आयु�के�ब�च��म���टं �ट�ग�को�2022 तक�38.4% से�25% तक�कम�करना�है� ।

�न�न�ल�खत�म��से�कूट�का�चयन�क��जए�:

a) केवल�1
b) केवल�2
c) 1 और�2 दोन��
d) न�तो�1 न�ही�2
Correct Answer: B
Your Answer:
Explanation

Solution (b)

पोषण�अ�भयान�8 माच��2018 को��धानमं�ी��ारा�झुंझुनू�म��शु���कया�गया�था।�अ�भयान�का�ल�य�युवा�बालक�, म�हला��एवं��कशो�रय��म�


�टं �ट�ग, अ�प-पोषण, एनी�मया�को�एवं�ज�म�के�समय�कम�वजन�को���त�वष���मश: 2%, 2%, 3 % तथा�2% तक�कम�करना�है�। �मशन
का�ल�य�0-6 आयु�के�ब�च��म���टं �ट�ग�को�2022 तक�38.4% से�25% तक�कम�करना�है� ।

पोषण�अ�भयान�

• पोषण�(PM’s Overarching Scheme for Holistic Nourishment) अ�भयान�का�ल�य�2022 तक


कुपोषण�मु��भारत�को�सु�न��त�करना�है�।
• यह�काय��म�अ�प-पोषण, एनी�मया�एवं�ज�म�के�समय�कम�वजन�को��माण�आधा�रत�पोषण�ह�त�ेप��का�अ�भसरण�सु�न��त�कर
एवं�भारत�म��भोजन�पोषण�हेतु�जन�आ�दोलन�उ�प��कर�घटाना�चाहता�है�
• इन�ल�य��क��पूत��हेतु�एक�मह�वपूण��ह�त�ेप�है- पोषण�समृ��एवं��थानीय��प�से�संधाराणीय�बना�दै �नक�आहार�क��गुणव�ा
सुधारना�।
• शै��क�सं�थान��को�वै�ा�नक���व�धयां�बनाने�एवं��प���प�से�प�रभा�षत�संकेतक��के�साथ��ा��तयो�य�काय��योजनाय��तैयार�करने�हेतु
भी��ो�सा�हत��कया�गया�था�।

IASbaba
Web: http://ilp.iasbaba.com/ Score:
Email: ilp@iasbaba.com 0.00 / 198
Page 113
2019 - Test 32 -
Exam Title :
Government...
Email :
Contact :
QUESTION 49.
ऑपरेशन��ड�जटल�बोड��(ODB) का��न�न�ल�खत�म��से�कौन�सा�उ�े �य�है�?

a) ODB �न�वदा�आवेदन��को�भरने�तथा����या�को�पारदश��रखने�हेतु�ऑनलाइन��लेटफाम���दान�करने�का�ल�य�रखता�है।�
b) यह�PSUs म��बोड��बैठक��हेतु�ऑनलाइन�इंटरफेस��दान�करने�का�ल�य�रखता�है।�
c) ODB छा���को��कसी�भी�समय�तथा��कसी�भी��थान�पर�ई-�रसोस�ज�उपल�ध�करवाने�के�अ�त�र��एक��लास��म�को��ड�जटल��लास
�म�म��बदलने�का�ल�य�रखता�है।�
d) उपरो��म��से�कोई�नह��
Correct Answer: C
Your Answer:
Explanation

Solution (c)

ऑपरेशन��ड�जटल�बोड��(ODB)

ऑपरेशन��ड�जटल�बोड��दे श�म��गुणव�ा��श�ा�को�बढ़ावा�दे ने�हेतु��ौ�ो�गक��के�इ�तेमाल�हेतु�है�।�ऑपरेशन��ड�जटल�बोड��एक��ां�तकारी


कदम�है�जो�अ�धगम�के�साथ-साथ��श�ण����या�को�अंत:��या�मक�बनाएगा�तथा���लपड�ल�न�ग�को�एक�अ�यापन�उपागम�के��प�म�
लोक��य�करेगा�।

उ�े �य�

• छा���हेतु��कसी�भी�समय��कसी�भी��थान�पर�ई-संसाधन��क��उपल�धता�के�अलावा�ODB का�ल�य�एक��लास��म�को��ड�जटल
�लास��प�म��बदलना�है�।
• यह�उभरती��ौ�ो�गक�य��जैसे�मशीन�ल�न�ग, आ�ट�फ��सयल�इंटे�लज�स�एवं�डेटा�एना�ल�सस�का�दोहन�कर����परक�अनुकूल
अ�धगम�के�साथ-साथ�इंटेलीज�ट��ूट�र�ग�के��ावधान�म��भी�मदद�करेगा�।
• एक��वशेष��स�म�त�ने�ODB के�अंतग�त��ड�जटल�क�ा��के�इ�तम�कॉ��फ़गरेशन�पर�काय���कया�है�।

उ�चतर��श�ा�सं�थान��(HEIs) म��काया��वयन�

• �व��व�ालय�अनुदान�आयोग�HEIs म��ODB के�हेतु�काया��वयन�एज�सी�होगी�।


• यह�अनुमान�लगाया�गया�है��क�सं�थान��म��ऐसे�5 लाख��लास�म�ह�गे��ज�ह��क���एवं�रा�य�सरकार���ारा�सहायता�द��जा�रही�है�। इन
म��से�, तैयारी�के�आधार�पर�, UGC 10,000 महा�व�ालय��को�पहले�चरण�म��कवर�करने�का���ताव�करता�है��जसम��2 लाख
�लास��स�ह�गे�।
• 2 लाख��लास��स�के��लए�लागत�के�2000 करोड़�होने�का�अनुमान�है�।�इसे�HEFA से�एक�ऋण�के��प�म��एक�क���य�योजना�के
तौर�पर�काया���वत��कया�जा�सकता�है।�

�कूल��म��काया��वयन�

• सभी�सरकारी�एवं�सरकारी�सहायता��ा�त��कूल��(जहाँ�मा�य�मक�तथा�व�र��मा�य�मक�क�ाएं�ह�) म���ड�जटल�/ �माट� �बोड���दान


�कये�जाय�गे�।
• लगभग�1.5 लाख�मा�य�मक/व�र��मा�य�मक��कूल��को�रा�य�एवं�UTs के�सहयोग�से�इस�योजना�के�अंतग�त�कवर��कया�जाएगा�।

QUESTION 50.
�धानमं�ी�आदश���ाम�योजना�उन��ाम��को�ल��त�करती�है�जहाँ��न�न�ल�खत�का�जनसं�या�ब�मत�हो-

a) अनुसू�चत�जा�तय��का�
b) अनुसू�चत�जनजा�तय��का�
c) अ�पसं�यक��का�
d) गरीबी�रेखा�से�नीचे�के�लोग��का�
Correct Answer: A
Your Answer:
Explanation

IASbaba
Web: http://ilp.iasbaba.com/ Score:
Email: ilp@iasbaba.com 0.00 / 198
Page 114
2019 - Test 32 -
Exam Title :
Government...
Email :
Contact :

Solution (a)

�धानमं�ी�आदश���ाम�योजना�(PMAGY) �व��मं�ी�के�बजट�भाषण�म��क��गई�घोषणा�के�प�ात�2009-10 म��पायलट�आधार�पर�शु�


�कया�गया�था�।�योजना�का�ल�य�उन��ाम��का�एक�कृत��वकास�करना�है�जहाँ�अनुसू�चत�जा�तय��क��जनसँ�या�50% से�अ�धक�है।
त�मलनाडु �(225), राज�थान�(225), �बहार�(225), �हमाचल��दे श�(225) एवं�असम�(100) के�कुल�1000 �ाम��को�पायलट�चरण
हेतु�चुना�गया�था।�

QUESTION 51.
म�हला�एवं�बाल��वकास�मं�ालय�ने��धानमं�ी�मातृ�वंदना�योजना�(PMMVY) आरंभ�क�।�इस�योजना�के�बारे�म���न�न�ल�खत�म��से�कौन�से
कथन�सही�ह��?

1. यह�एक�मातृ�व�लाभ�काय��म�है�जहाँ�क���एवं�रा�य�संगठन��म��काय�रत�सभी�म�हला�क�म�य��को�8 महीन��का�सवैत�नक�अवकाश��दया
जाता�है।�

2. योजना�क��दशा��को�पूरा�करने�पर�गभ�वती�एवं��ध��पलाने�वाली�AWWs/ AWHs/ ASHA भी�PMMVY के�अंतग�त�लाभ�उठा


सकती�ह�।�

�न�न�ल�खत�म��से�कूट�का�चयन�क��जए�:

a) केवल�1
b) केवल�2
c) 1 और�2 दोन��
d) न�तो�1 न�ही�2
Correct Answer: B
Your Answer:
Explanation

Solution (b)

�धानमं�ी�मातृ�वंदना�योजना�(PMMVY) एक�मातृ�व�लाभ�काय��म�है�जो�दे श�के�सभी��जल��म��रा�ीय�खा��सुर�ा�क़ानून�, 2013 के


अनुसार�काया���वत��कया�जाता�है।�

उ�े �य�

1. नकद��ो�साहन�के��प�म��मज�री�हा�न�क��आं�शक���तपू�त��करना�ता�क�म�हला�पहले�ब�चे�के�ज�म�से�पूव��और�प�ात�पया��त�आराम
कर�सके�।
2. �दए�गए�नकद��ो�साहन�का�प�रणाम�गभ�वती�एवं��ध��पलाने�वाली�माता��(PW& LM) के�सुधरे��ए��वा��य�के��प�म��सामने
आएगा�।

ल��त�लाभाथ��

1. सभी�गभ�वती�एवं��ध��पलाने�वाली�माताएं�( PW&LM के�अ�त�र�) जो�क���सरकार�अथवा�रा�य�सरकार��अथवा�PSUs म�


�नय�मत�रोजगार��ा�त�ह��अथवा�वे�जो�उस�समय��भावी��कसी�क़ानून�के�अंतग�त�समान�लाभ�ले�रही�ह��
2. वे�सभी�गभ�वती�एवं��ध��पलाने�वाली�माताएं�जो�प�रवार�के�पहले�ब�चे�को�लेकर�01.01.2017 को�या�उसके�बाद�गभ�वती�ह��।
3. लाभाथ��हेतु�गभा�व�था�क���त�थ�एवं�अव�था�MCP काड��पर��लखे�अनुसार�LMP �त�थ�के�मामले�म���गनी�जायेगी�।
4. गभ�पात�/ ��टल�बथ��का�मामला�:
◦ लाभाथ��योजना�के�अंतग�त�केवल�एक�बार�लाभ�लेने�क��पा��है�।
◦ गभ�पात�/ ��टल�बथ��के�मामले�म��, लाभाथ��भ�व�य�म��गभ�वती�होने�क��दशा�म��बची��ई��क�त��लेने�क��पा��होगी�।
◦ इस��कार�, पहली��क�त��ा�त�करने�के�प�ात�, य�द�लाभाथ��का�गभ�पात�होता�है�, उस�दशा�म��भावी�गभ�धारण�क��अव�था
म��वह�केवल��सरी�एवं�तीसरी��क�त�लेने�क��पा��होगी�(बशत��उसने�योजना�क��पा�ता�शत��पूरी�क��ह�) । साथ�ही�, य�द
पहली�व��सरी��क�त��ा�त�करने�के�बाद�लाभाथ��का�गभ�पात�अथवा���टल�बथ��होता�है, वह�भावी�गभा�व�था�के�मामले�म�
केवल�तीसरी��क�त�लेने�क��पा��होगी�(बशत��उसने�योजना�क��पा�ता�शत��पूरी�क��ह�) ।

IASbaba
Web: http://ilp.iasbaba.com/ Score:
Email: ilp@iasbaba.com 0.00 / 198
Page 115
2019 - Test 32 -
Exam Title :
Government...
Email :
Contact :
4. �शशु�मृ�यु�का�मामला�: लाभाथ��योजना�के�अंतग�त�केवल�एक�बार�लाभ�लेने�क��पा��है�। अथा�त�, �शशु�मृ�यु�क��दशा�म��, वह�योजना
के�अंतग�त�लाभ�का�दावा�करने�क��पा��नह��होगी, य�द�उसने�पूव��म��PMMVY के�अंतग�त�मातृ�व�लाभ�क��सभी��क�त���ा�त�कर�ली
ह��।
5. योजना�क��दशा��को�पूरा�करने�पर�गभ�वती�एवं��ध��पलाने�वाली�AWWs/ AWHs/ ASHA भी�PMMVY के�अंतग�त�लाभ
उठा�सकती�ह�।�

PMMVY के�अंतग�त��द��लाभ�

• तीन��क�त��म��5000 �पये�का�नकद��ो�साहन�अथा�त�आंगनबाड़ी�क���(AWC) अथवा�अनुमो�दत��वा��य�क���म��शु�आती�दौर�म�


पंजीकरण�पर�1000 �पए�, गभ�धारण�के�छ: माह�प�ात�कम�से�कम�एक��सव�पूव��जांच�(ANC) पर�2000 �पये�क���सरी
�क�त�तथा�ब�चे�के�ज�म�के�पंजीकरण�तथा�उसके�BCG, OPV, DPT एवं�हेपेटाइ�टस�बी�अथवा�इसक��समक��खुराक��ा�त�कर
लेने�पर�2000 क��तीसरी��क�त�।
• पा��लाभाथ��सं�थागत��सव�पर�जननी�सुर�ा�योजना�(JSY) के�अंतग�त��ो�साहन��ा�त�कर�गे�तथा�JSY के�अंतग�त��ा�त��ो�साहन
को�मातृ�व�लाभ�के�सापे���गना�जाएगा�ता�क�औसतन�एक�म�हला�को�6000 �पये��ा�त�ह��।

QUESTION 52.
‘युवा��वा�भमान�योजना’ के�बारे�म���न�न�ल�खत�कथन��पर��वचार�क��जए�:

1. यह�योजना�उ�र��दे श�सरकार��ारा�आ�थ�क��प�से�कमजोर�तबक��हेतु�शु��क��गई�है।�

2. यह�योजना�आ�थ�क��प�से�कमजोर�तबक��के�युवा��को���तवष��100 �दन��के�रोजगार�क��गारंट��दे गी।�

3. मनरेगा�के�अंतग�त��नयो�जत�युवा�इसके�अंतग�त�पा��नह��ह�गे।�

उपयु���म��से�कौन�से�कथन�स�य�ह��?

a) केवल�1
b) केवल�2 और�3
c) केवल�1 और�3
d) उपयु���सभी�
Correct Answer: B
Your Answer:
Explanation

Solution (b)

म�य��दे श�क��सरकार�ने�एक�योजना�शु��करने�क��घोषणा�क��है�ता�क�शहरी��े���म��आ�थ�क��प�से�कमजोर�तबक��(EWS) के�युवा��को�


अ�थायी�रोजगार�सु�न��त��कया�जा�सके�।

युवा��वा�भमान�योजना�

1. यह�योजना�आ�थ�क��प�से�कमजोर�तबक��के�युवा��को���तवष��100 �दन��के�रोजगार�क��गारंट��दे गी।�


2. �नयोजन�के�दौरान�युवा��को�उनक��पसंद�का�कौशल���श�ण��दया�जाएगा�ता�क�वे�उपल�ध�नौकरी�अवसर��का�लाभ�ले�सक��।
3. जहाँ��ामीण��े���के�लोग��को�महा�मा�गाँधी�रा�ीय��ामीण�रोजगार�गारंट��कानून�(MNREGA) के�अंतग�त�रोजगार��मलता�है�वह�
शहरी�गरीब�इससे�वं�चत�रह�जाते�ह��।
4. यह�योजना�उ�ह���भावी��प�से�आ�छा�दत�करेगी�।

QUESTION 53.
�धानमं�ी�ने�हाल�ही�म���वासी�तीथ��दश�न�योजना�शु��क��है।�इस�योजना�के�बारे�म���न�न�ल�खत�म��से�कौन�से�कथन�सही�ह��?

1. भारतीय�डाय�पोरा�के�एक�समूह�को�वष��म��दो�बार�सरकारी��य�पर�धा�म�क��थल��क��या�ा�पर�ले�जाया�जाएगा।�

2. सरकार�उनके��नवास�वाले�दे श�से�वायुया�ा�भाड़े�स�हत�सभी��य��को�वहन�करेगी।�

IASbaba
Web: http://ilp.iasbaba.com/ Score:
Email: ilp@iasbaba.com 0.00 / 198
Page 116
2019 - Test 32 -
Exam Title :
Government...
Email :
Contact :

3. भारतीय�मूल�के�45 वष��से�65 वष��आयुवग��के�सभी�����इस�हेतु�आवेदन�कर�सकते�ह��तथा�उन�लोग��म��से�ही�समूह�का�चयन��कया


जाएगा��जसम��‘�गर�म�टया�दे श�’ को�वरीयता�द��जायेगी।�

�न�न�ल�खत�म��से�कूट�का�चयन�क��जए�:

a) 1 और�2
b) 2 और�3
c) 1 और�3
d) उपयु���सभी�
Correct Answer: D
Your Answer:
Explanation

Solution (d)

�वासी�तीथ��दश�न�योजना�

1. माननीय��धानमं�ी�ने�यह�योजना�शु��क��है��जसके�अंतग�त�भारतीय�डाय�पोरा�के�एक�समूह�को�वष��म��दो�बार�सरकारी��य�पर�भारत
के�धा�म�क��थल��क��या�ा�पर�ले�जाया�जाएगा।�
2. भारतीय�मूल�के�40 लोग��का�पहला�ज�था��वासी�भारतीय��दवस�पर�अपनी�या�ा�शु��करेगा�।
3. उ�ह��भारत�के�सभी��मुख�धम��के�धम���थल��पर�ले�जाया�जाएगा�तथा�सरकार�उनके��नवास�वाले�दे श�से�वायुया�ा�भाड़े�स�हत�सभी
�य��को�वहन�करेगी।�
4. भारतीय�मूल�के�45 वष��से�65 वष��आयुवग��के�सभी�����इस�हेतु�आवेदन�कर�सकते�ह��तथा�उन�लोग��म��से�ही�समूह�का�चयन��कया
जाएगा��जसम��‘�गर�म�टया�दे श�’ (मॉ�रशस, �फजी, सूरीनाम, गुयाना, ���नदाद�व�टोबेगो, जमैका�आ�द) को�वरीयता�द��जायेगी।�

�गर�म�टया�

1. �गर�म�टया�या�जहाजी�उन�अनुबं�धत�भारतीय�मज�र��के�वंशज�ह���ज�ह���फजी, मॉ�रशस, द. अ��का, पूव��अ��का, मलय��ाय��प,


कै�र�बयाई���प�एवं�द��ण�अमे�रका�( सूरीनाम, गुयाना, ���नदाद�व�टोबेगो�) लाया�गया�था�।
2. उ�ह��यूरोपीय�लोग��क��समृ���तथा��फजी�लोग��को�इन�खेत��म��काम�करने�से�बचाने�तथा�इस��कार�उनक��सं�कृ�त�क��र�ा�हेतु�ग�े�के
खेत��म��काम�करने�के��लए��कराए�पर�लाया�गया�था�।
3. “ ए�ीम�ट�” वह�श�द�है��जसे����टश�सरकार�के�भारतीय�मज�र��के�साथ��ए�अनुबंध�को�संद�भ�त�करने�हेतु�“ �गर�मट�” कहा�जाने
लगा�।
4. इसे��फजी�व�के�र�बयाई�म���कने�क��अव�ध�के��प�म��संद�भ�त��कया�जाता�था��जसके�प�ात�उ�ह��भारत�वापस�जाने�क��अनुम�त�द�
जानी�थी�।

QUESTION 54.
अटल�सौर�कृ�ष�पंप�योजना�के�अंतग�त�, सरकार�सौर�कृ�ष�पंपसेट्स�पर�95% स��सडी�दे �रही�है।��न�न�ल�खत�म��से��कस�रा�य�ने�यह�योजना
शु��क��है�?

a) म�य��दे श�
b) महारा��
c) ओ�डशा�
d) असम�
Correct Answer: B
Your Answer:
Explanation

Solution (b)

अटल�सौर�कृ�ष�पंप�(ASKP) योजना�

1. महारा��सरकार�ने��कसान��को�सौर�कृ�ष�पंपसेट्स�पर�95% स��सडी�दे �ने�हेतु�ASKP योजना�शु��क��है�।

IASbaba
Web: http://ilp.iasbaba.com/ Score:
Email: ilp@iasbaba.com 0.00 / 198
Page 117
2019 - Test 32 -
Exam Title :
Government...
Email :
Contact :
2. 5 एकड़�से�कम�भू�म�वाले��कसान��को�केवल�5% अथा�त�12,000 �पये�भुगतान�करना�होगा�और�उ�ह��तीन�अ��श���के�पंप��मल
जाय�गे�।
3. 5 एकड़�से�अ�धक�भू�म�वाले��कसान��को�केवल�30,000 �पये�भुगतान�करना�होगा�और�उ�ह��पाँच�अ��श���के�पंप��मल�जाय�गे�।

QUESTION 55.
�वा��य�एवं�प�रवार�क�याण�मं�ालय��ारा�मे��ट�अल�हाइजीन��क�म�शु��क��गई�है।�इस�योजना�के�बारे�म���न�न�ल�खत�म��से�कौन�से�कथन
सही�ह��?

1. इसका�ल�य��कशो�रय��के�बीच�मे��ट�अल�हाइजीन�पर�जाग�कता�बढ़ाना�है।�

2. केवल�शहरी��े���म���कशो�रय��तक�उ�च�गुणव�ा�वाले�सेनेटरी�नैप�क�स�क��प�ँच�और��योग�बढ़ाना।�

3. पया�वरण��हतैषी�ढं ग�से�सेनेटरी�नैप�क�स�के�सुर��त��न�तारण�को�सु�न��त�करना।�

�न�न�ल�खत�म��से�कूट�का�चयन�क��जए�:

a) केवल�1
b) 2 और�3
c) 1 और�3
d) उपयु���सभी�
Correct Answer: C
Your Answer:
Explanation

Solution (c)

मे��ट�अल�हाइजीन��क�म�

मु�य��प�से��ामीण��े���क���कशो�रय��के�बीच�मे��ट�अल�हाइजीन�क��ज�रत�को�संबो�धत�करने�हेतु�भारत�सरकार�मे��ट�अल�हाइजीन��क�म
का�समथ�न�कर�रही�है��जसके�तहत�रा�य�/ UTs को�सेनेटरी�नैप�क�स�पै�स�के��वक���कृत��य�हेतु�रा�ीय��वा��य��मशन�के�मा�यम�से�कोष
उपल�ध�करवाए�जाय�गे�। इ�ह���मुखत: रा�य��और�UTs के�अपने�काय��म�काया��वयन�योजना��से��ा�त���ताव��के�अनुसार�स��सडी�दर�
पर��ामीण��कशो�रय��के��लए��दया�जाएगा�।

योजना�के�अंतग�त��न�न�ल�खत�आता�है�:

• �कशो�रय��के�बीच�मे��ट�अल�हाइजीन�पर�जाग�कता�बढ़ाना।�

• �ामीण��े���म���कशो�रय��तक�उ�च�गुणव�ा�वाले�सेनेटरी�नैप�क�स�क��प�ँच�और��योग�बढ़ाना।�

• पया�वरण��हतैषी�ढं ग�से�सेनेटरी�नैप�क�स�के�सुर��त��न�तारण�को�सु�न��त�करना।�

• आशा�काय�कता���को�धनराशी�का��ावधान�ता�क�वे��कशो�रय��के�साथ�मे��ट�अल�हाइजीन�पर�चचा��हेतु�मा�सक�बैठक�कर�सक��।

मे��ट�अल�हाइजीन��क�म�के�इद� �गद� �IEC साम�ी�क��एक��ृंखला��वक�सत�कर�द��गई�है��जसम��360 �ड�ी�उपागम�का��योग��कशो�रय��के


बीच�सुर��त�एवं��व�छ�मे��ट�अल��वा��य�अ�यास��के�बारे�म��जाग�कता�पैदा�करने�के��लए��कया�गया�है�। इसम���कशो�रय��के��लए�ऑ�डयो,
�व�डयो�एवं�री�ड�ग�साम�ी�शा�मल�है�तथा�आशा�काय�कता���तथा�फ��ड��तर�के�अ�य�काय�कता���के��लए��कशो�रय��से�संवाद�हेतु�जॉब-एड् स
शा�मल�है�।

दे श�भर�क��आशा�काय�कता���को���श��त��कया�जाता�है�तथा�वे�सेनेटरी�नैप�क�स�के��योग�को�बढ़ावा�दे ने�व�उसके��वतरण�म��एक�मह�वपूण�
भू�मका�अदा�करती�ह��।

�वा��य�मं�ालय�के�अधीन��वा��य�शोध��वभाग�को�सभी�नए, वैक��पक�पया�वरण��हतैषी�मे��ट�अल�हाइजीन�उ�पाद��के�आंकलन�म��शा�मल
�कए�गया�है�ता�क�वह�उसक��सुर�ा�एवं��वीकाय�ता��वशेषता��क��पड़ताल�कर�सके�।

QUESTION 56.

IASbaba
Web: http://ilp.iasbaba.com/ Score:
Email: ilp@iasbaba.com 0.00 / 198
Page 118
2019 - Test 32 -
Exam Title :
Government...
Email :
Contact :

कं�यूटर�श���संप��दे श��के��व��तरीय�समूह�म���वेश�पाने�हेतु�भारत�ने�अपना�रा�ीय�सुपरकं�यू�ट�ग��मशन�(NSM) आरंभ��कया।�NSM के


बारे�म���न�न�ल�खत�म��से�कौन�से�कथन�सही�ह��?

1. यह�योजना�संयु���प�से��व�ान�एवं��ौ�ो�गक���वभाग�(DST) तथा�इले��ॉ�न�स�एवं�सूचना��ौ�ो�गक���वभाग�(DeitY) �ारा


काया���वत�व�संचा�लत�क��जाती�है।�

2. NSM को�दो��मुख��ै �स�म���वभा�जत��कया�जाता�है- �ब�ड�एंड�बाय�, �ज�ह���मश: C-DAC तथा�ब�गलु��के�इं�डयन�इं��ट�ूट�ऑफ़


साइंस��ारा�आगे�बढ़ाया�जा�रहा�है।�

�न�न�ल�खत�म��से�कूट�का�चयन�क��जए�:

a) केवल�1
b) केवल�2
c) 1 और�2 दोन��
d) न�तो�1 न�ही�2
Correct Answer: C
Your Answer:
Explanation

Solution (c)

रा�ीय�सुपरकं�यू�ट�ग��मशन�(NSM): �मता�एवं�यो�यता��नमा�ण�

यह�योजना�संयु���प�से��व�ान�एवं��ौ�ो�गक���वभाग�(DST) तथा�इले��ॉ�न�स�एवं�सूचना��ौ�ो�गक���वभाग�(DeitY) �ारा�4500 क


रोड़�क��अनुमा�नत�लागत�से�सात�वष��क��अव�ध�हेतु�काया���वत�व�संचा�लत�क��जाएगी।�( नोट�: �मशन�2015 म��शु���कया�गया�था।�यह
सु�ख़�य��म��था��य��क�यह�भारत�के�दो�सबसे�तेज�मौसमी�सुपरकं�यूटर��वक�सत�करने�म��स�म�रहा�है।�)

�मशन�दे श�भर�म��फैले�हमारे�रा�ीय��श�ण�तथा�अनुस�धान�एवं��वकास�सं�थान��को�70 से�अ�धक�उ�च-�न�पादन�कं�यू�ट�ग�सु�वधा��वाली


�व�तृत�सुपरकं�यू�ट�ग���ड�क���थापना�के�मा�यम�से�सश��बनाने�का����कोण�रखता�है।�इन�सुपरकं�यूटर��को�रा�ीय��ान�नेटवक��(NKN) प
र�लगी�नेशनल�सुपरकं�यू�ट�ग���ड�पर�नेटवकर्�ड��कया�जाएगा।�NKN सरकार�का�एक�अ�य�काय��म�है�जो�शै��णक�सं�थान��एवं�R&D लै
�स�को�एक�उ�च�ग�त�नेटवक��से�जोड़ता�है।�शै��णक�एवं�R&D सं�थान�तथा��मुख��यो�ा��वभाग/मं�ालय�इन�सु�वधा��का��योग�कर
भागीदारी�कर�गे�और�रा�ीय��ासं�गकता�के�अनु�योग��वक�सत�कर�गे।��मशन�के�अंतग�त�इन�अनु�योग��के��वकास�से�जुडी�चुनौ�तय��से��नपटने
हेतु�अ�त�पेशेवर�उ�च��दश�न�कं�यू�ट�ग�(HPC) सजग�मानव�संसाधन�का��वकास�शा�मल�है।�

NSM को�दो��मुख��ै �स�म���वभा�जत��कया�जाता�है- �ब�ड�एंड�बाय�, �ज�ह���मश: C-DAC तथा�ब�गलु��के�इं�डयन�इं��ट�ूट�ऑफ़


साइंस��ारा�आगे�बढ़ाया�जा�रहा�है।�

जहाँ�चरण�I म��सुपरकं�यूटस��क��अ�से���लंग�शा�मल�है�वह��चरण�II म��चय�नत�कॉ�पोने�ट�जैसे�मदरबोड��को��थानीय��प�से�बनाया�जाएगा�,


तथा�तीसरे�चरण�म��, सुपरकं�यूटर�को�भारत�म��C-DAC �ारा��डज़ाइन��कया�जाएगा।�

QUESTION 57.
रा�ीय�सहकारी��वकास��नगम�(NCDC) ने�‘ सहकार�22’ नामक�एक��मशन�शु���कया�है।��मशन�का�उ�े �य�है-

a) 2022 तक��कसान��क��आय�दोगुनी�करना�
b) 2022 तक��ामीण��े���म���टाट� �अ�स�क��सं�या�दोगुनी�करना�
c) 2022 तक�सहकारी�सं�था��क��सं�या�दोगुनी�करना�
d) �ामीण��े���म��SHGs क��सं�या�दोगुनी�करना�
Correct Answer: A
Your Answer:
Explanation

Solution (a)

IASbaba
Web: http://ilp.iasbaba.com/ Score:
Email: ilp@iasbaba.com 0.00 / 198
Page 119
2019 - Test 32 -
Exam Title :
Government...
Email :
Contact :

रा�ीय�सहकारी��वकास��नगम�(NCDC) युवा��को�सहकारी��वसाय�उ�म��क��ओर�आक�ष�त�करने�हेतु�एक�युवा��हतैषी�योजना�‘ युवा


सहकार-कोआपरे�टव�एंटर�ाइज�सपोट� �एंड�इनोवेशन��क�म�” लेकर�आया�है।�

योजना�को�NCDC �ारा��न�म�त�1000 करोड़�के�‘ कोआपरे�टव��टाट� -अप�एंड�इनोवेशन�फ�ड�(CSIF)’ से��ल�क��कया�जाएगा।�इसम�


पूव��र��े��क��सहकारी�सं�था�, आकां�ी��जल��तथा�म�हला��अथवा�SC अथवा�ST अथवा�PwD सद�य��वाली�सहकारी�सं�था��हेतु
अ�धक��ो�साहन�होगा।�इन��वशेष��े�णय��हेतु�प�रयोजना�का��व�यन�प�रयोजना�लागत�का�80% तक�होगा�जब�क�अ�य��के��लए�यह�70%
तक�होगा।�इस�योजना�म��3 करोड़��पए�तक�क��प�रयोजना�हेतु��लए�गए�टम��लोन�के��लए�2% कम��याज�दर�क��बात�है�तथा�मूलधन�के
भुगतान�पर�2 वष��के��थगन�का��ावधान�है।�कम�से�कम�एक�वष��तक�संचालनरत�सभी�सहकारी�सं�थाएं�इसक��पा��है।�

सहकार�22

NCDC जो��क�सहकारी�सं�था��क����नया�म��सवा��धक�वरीयता�वाला��व�ीय�सं�थान�है, ने�सहकार�22 शु���कया�है�जो��क�2022 तक


�कसान��क��आय�दोगुनी�करने�संबंधी��मशन�है।�

QUESTION 58.
�वदे श�दश�न�योजना�दे श�म���वषय�आधा�रत�स�क�ट्स�के��वकास�हेतु�पय�टन�मं�ालय�क���लैग�शप�योजना��म��से�एक�है।��न�न�ल�खत�म��से
�कस�रा�य�म���ाइबल�टू �र�म�स�क�ट��वक�सत��कया�जा�रहा�है�?

a) असम�
b) छ�ीसगढ़�
c) उ�राखंड�
d) राज�थान�
Correct Answer: B
Your Answer:
Explanation

Solution (b)

�वदे श�दश�न�योजना, दे श�म��योजनाब��एवं��ाथ�मकतापरक�ढं ग�से��वषय�आधा�रत�स�क�ट्स�के��वकास�हेतु�पय�टन�मं�ालय�क���लैग�शप


योजना��म��से�एक�है।�योजना�2014 -15 म��शु��क��गई�थी�और�आज�तक�मं�ालय�ने�31 रा�य��एवं�UTs म��5997.47 करोड़��पये
क��लागत�क��74 प�रयोजना��को�मंजूरी�द��है।�इन�प�रयोजना��क��30 प�रयोजना��/ �मुख�अवयव��के�इस�वष��पूरा�हो�जाने�क�
संभावना�है।�

जनजा�तय��एवं�जनजातीय�सं�कृ�त�का��वकास�पय�टन�मं�ालय�के��यानके��्ण�के�मु�य��े��म��से�एक�है।�मं�ालय�जनजातीय��े��म��पय�टन�के
�वकास�और��ो�साहन�हेतु�ग�त�व�धय��क��एक��ृंखला�अंजाम�दे �रहा�है।�मं�ालय�अपनी��वदे श�दश�न�योजना�के�तहत�इस��े��म��पय�टन
अवसंरचना��वक�सत�कर�रहा�है।�योजना�के��ाइबल�स�क�ट�थीम�के�अंतग�त�मं�ालय�ने�नागाल�ड, तेलंगाना�और�छ�ीसगढ़�म��381.37 करोड़
�पये�क��4 प�रयोजना��को�मंजूरी�द��है।�

पहला��ाइबल�स�क�ट�छ�ीसगढ़�म���वक�सत��कया�जा�रहा�है।�प�रयोजना�छ�ीसगढ़�म��तेरह��थल��नामत: जशपुर, कुनकुरी, मैनपट,


कमलेशपुर, महेशपुर, कुरदार, सरोदादार, गंगरेल, क�डागाँव, न�थया�नवागाँव, जगदलपुर, �च�कूट, तीथ�गढ़�को�कवर�करती�है।�

QUESTION 59.
सरकार�ने�‘भारत�के�वीर’ को�एक���ट�का�दजा���दान��कया�है।�‘भारत�के�वीर’ के�बारे�म���न�न�ल�खत�म��से�कौन�से�कथन�सही�ह��?

a) यह�भारतीय�अ�� सै�नक�बल��और�सेना�के�उन�शहीद��क��भू�मका�को�उभारने�वाली�एक�ऑनलाइन�पहल�है��ज�ह�ने�कत���के�पथ�पर
अपने�जीवन�का�ब�लदान�कर��दया।�
b) यह�अ�� सै�नक�बल��हेतु�एक��ववाद�समाधान�पोट� ल�है�जहाँ�जवान�अपने�मु�े�उठा�सकते�ह�।�
c) यह�एक�पोट� ल�है�जहाँ�लोग�उन�जवान��के�प�रवार��को�योगदान�दे �सकते�ह���ज�ह�ने�कत���के�पथ�पर�अपने�जीवन�का�ब�लदान�कर
�दया।�
d) यह�एक�पोट� ल�है�जहाँ�जाग�कता�पैदा�करने�हेतु�भारतीय�सै�नक�क��जीवन�शैली�और��े �न�ग�क��चचा��क��जाती�है।�
Correct Answer: C
Your Answer:

IASbaba
Web: http://ilp.iasbaba.com/ Score:
Email: ilp@iasbaba.com 0.00 / 198
Page 120
2019 - Test 32 -
Exam Title :
Government...
Email :
Contact :
Explanation

Solution (c)

Bharat ke Veer

1. सरकार�ने�‘भारत�के�वीर’ (जो��क�काय�वाही�के�दौरान�मारे�गए�अ�� सै�नक�बल��के�जवान��के�प�रवार��को�मदद�दे ने�हेतु�क��गई�एक


�नजी�पहल�है) को�एक���ट�का�दजा���दान��कया�है।�
2. भारत�के�वीर�को��दया�गया�योगदान�आयकर�से�मु��है�।
3. अ�य�कुमार�एवं�पूव��रा�ीय�बैड�म�टन�च��पयन�पुलेला�गोपीचंद�को��या�सय��म��शा�मल��कया�गया�है�।
4. इस�पहल�को�अब�औपचा�रक��प�से�एक�पंजीकृत��यास�क��श�ल�दे �द��गई�है�ता�क�शहीद�जवान��के�प�रवार��को�योगदान�दे ने�एवं
सहयोग��दान�करने�हेतु�नाग�रक��को�एक��लेटफाम���दान��कया�जा�सके�।
5. जनता�भारत�के�वीर�ए�प�तथा�वेबसाइट��व�जट�कर�सकती�है�और�कत���के�पथ�पर�अपने�जीवन�का�ब�लदान�कर�ने�वाले�जवान��के
प�रवार��को�योगदान�एवं�समथ�न�दे �सकती�है�।

QUESTION 60.
भारी�उ�ोग�मं�ालय�ने�FAME इं�डया�योजना�शु��क��है।�योजना�का��न�न�ल�खत�म��से��या�उ�े �य�है�?

a) भारतीय�भारी�उ�ोग��म��काब�न�फुट���ग�को��यूनतम�करने�हेतु�संधारणीय�अ�यास��को�बढ़ावा�दे ना।�
b) इले���क�एवं�हाइ��ड�वाहन��ौ�ो�गक��के��व�नमा�ण�को�बढ़ावा�दे ना।�
c) भारतीय�औ�ो�गक�उ�पाद��को�बढ़ावा�दे ने�और�भारतीय��नया�त��को�बढाने�हेतु�अंतरा��ीय��भाव�वाले�लोग��का��योग�करना।�
d) उपयु���सभी�
Correct Answer: B
Your Answer:
Explanation

Solution (b)

इले���क�एवं�हाइ��ड�वाहन��ौ�ो�गक��के��व�नमा�ण�को�बढ़ावा�दे ने�एवं�उसका�सतत��वकास�सु�न��त�करने�हेतु, भारी�उ�ोग��वभाग�FAME


इं�डया�योजना�लागू�कर�रहा�है।�

योजना�इले���क�वाहन��(EVs) के�साव�ज�नक�प�रवहन�म���योग�को�जोर�दे ने�का���ताव�करती�है�एवं�बाजार�सृजन�तथा�मांग�समेकन��ारा�E


Vs के�अनुकूलन�क��बात�करती�है।�मसौदा�योजना�EV उ�ोग�के�सम���वकास�का����कोण�रखती�है�, �जसम��चा�ज�ग�अवसंरचना��दान
करना, EV �ौ�ो�गक��का�शोध�एवं��वकास�तथा��ापक��वदे शीकरण�क��ओर��ो�साहन�शा�मल�है।�

इस�भावना�के�अंतग�त�, साव�ज�नक�प�रवहन�के��ब�ु�तकरण�को��ो�साहन�दे ने�हेतु�पूण��इले���क�बस��के�समथ�न�को�इस�योजना�म��जोड़��लया


गया�है।�

QUESTION 61.
रा�ीय�सेवा�योजना�(NSS) के�बारे�म���न�न�ल�खत�कथन��पर��वचार�क��जए�:

1. यह��कूल�एवं��व��व�ालय�के�छा���को�सरकार�के�नेतृ�व�वाली�अनेक�सामुदा�यक�सेवा�ग�त�व�धय��व�काय��म��म��भाग�लेने�के�अवसर
�दान�करती�है।�

2. NSS का�एकमा��उ�े �य�युवा�छा���को�सामुदा�यक�सेवा��दान�करने�का���य��अनुभव��दान�करना�है।�

3. ‘मुझे�नह��तुझे’ रा�ीय�सेवा�योजना�का��स�ांत�वा�य�है।�

उपयु���म��से�कौन�सा�कथन�सही�है?

a) 1 और�2
b) 2 और�3
c) 1 और�3

IASbaba
Web: http://ilp.iasbaba.com/ Score:
Email: ilp@iasbaba.com 0.00 / 198
Page 121
2019 - Test 32 -
Exam Title :
Government...
Email :
Contact :

d) उपयु���सभी�
Correct Answer: D
Your Answer:
Explanation

Solution (d)

रा�ीय�सेवा�योजना�(NSS)

रा�ीय�सेवा�योजना�(NSS) भारत�सरकार�के�खेल�एवं�युवा�मामले�मं�ालय�क��एक�क���य��े�क�क��योजना�है�।�यह�+2 बोड���तर�के�11 व�


तथा�12 व��के�युवा�छा���एवं�भारत�के�तकनीक��सं�थान�, महा�व�ालय�तथा��व��व�ालय��तर�पर��नातक�एवं�परा�नातक�के�युवा�छा���को
अवसर��दान�करती�है��क�व��सरकार�के�नेतृ�व�वाली�अनेक�सामुदा�यक�सेवा�ग�त�व�धय��व�काय��म��म��भाग�ले�सक�।�NSS का�एकमा��उ�े श्
य�युवा�छा���को�सामुदा�यक�सेवा��दान�करने�का���य��अनुभव��दान�करना�है।�1969 म��NSS के�गठन�के�बाद�से�छा���क��सं�या�40,0
00 से�बढ़कर�माच��2018 तक�3.8 �म�लयन�तक�हो�चुक��है�। �व�भ���व��व�ालय�, महा�व�ालय��तथा�उ�च�अ�धगम�के�सं�थान��के�छा��
ने��व�भ��सामुदा�यक�सेवा�काय��म��म��भाग�लेने�हेतु�कदम�बढाया�है�।

· वे�सभी�युवा��वयंसेवक�जो�NSS के�नेतृ�व�वाली�सामुदा�यक�सेवा�के�मा�यम�से�दे श�क��सेवा�करने�का��वक�प�अपनाते�ह�, ज�रतमंद�क�


मदद�के���त�गव��एवं�उ�रदा�य�व�क��भावना�के�साथ�NSS बैज�पहनते�ह��।

· NSS बैज�म��मौजूद�कोणाक���हील��जसम��8 छड़��लगी�ह�, �दन�के�24 घंटे�दशा�ता�है�तथा�इसे�पहनने�वाले�को�याद��दलाता�है��क�उसे�24 घं


टे �दे श�सेवा�के��लए�तैयार�रहना�है�।

· बैज�म���यु��लाल�रंग�NSS �वयंसेवक���ारा�दशा�ई�जाने�वाली�उजा��एवं�भावना�का��तीक�है�।

· नीला�रंग���ांड�का��तीक�है��जसका�एक�छोटा�सा�भाग�NSS मानवता�के���त�अपने��ह�से�का�योगदान�दे ने�हेतु�त�पर�है�।

‘ मुझे�नह��तुझे�’ रा�ीय�सेवा�योजना�का��स�ांत�वा�य�है।�

QUESTION 62.
खेल�एवं�युवा�मामले�मं�ालय�ने��न�न�ल�खत�म��से��कस�खेल�को�लोक��य�बनाने�हेतु�एक��मशन�XI �म�लयन�शु���कया�है�?

a) ��केट�
b) खो�- खो�
c) कु�ती�
d) फुटबॉल�
Correct Answer: D
Your Answer:

IASbaba
Web: http://ilp.iasbaba.com/ Score:
Email: ilp@iasbaba.com 0.00 / 198
Page 122
2019 - Test 32 -
Exam Title :
Government...
Email :
Contact :
Explanation

Solution (d)

�मशन�XI �म�लयन�दे श�भर�म��फुटबॉल�को�लोक��य�बनाने�हेतु�खेल�एवं�युवा�मामले�मं�ालय, आल�इं�डया�फुटबॉल�फेडरेशन�(AIFF) तथा


फेडरेशन�इंटरनेशनल�डी�फुटबॉल�एसो�सएशन�(FIFA) का�संयु��काय��म�है।�यह�काय��म�फुटबॉल�को�बढ़ावा�दे ने�हेतु�दे श�भर�के�11 �म�ल
यन�लोग��तक�प�ँचने�का�ल�य�रखता�है।�

भारत�सरकार�ने�इस�काय��म�के��लए�12.55 करोड़��पये�आवं�टत��कए�ह��तथा�इतनी�ही�रा�श�AIFF/ FIFA �ारा�खच��क��जायेगी।


लगभग�6 �म�लयन�ब�च��को�पहले�ही�इस�काय��म�के�तहत�आ�छा�दत��कया�जा�चुका�है।��मशन�XI �म�लयन�के�भाग�के��प�म��दे श�भर�म�
फुटबॉल�को�लोक��य�बनाने�हेतु�आवं�टत�संसाधन�11 �म�लयन�छा��, अ�भभावक��तथा�कोच�को�कवर�करने�हेतु�पया��त�ह�।�

QUESTION 63.
‘ �द�ांग�सारथी�’ ए�प�के�बारे�म���न�न�ल�खत�कथन��पर��वचार�क��जए।�:

1. इसका�ल�य��डपाट� म�ट�ऑफ़�ए�पावरम�ट�ऑफ़�पस��स��वद��डसे�ब�लट�ज�(DEPwD) के�बारे�म��सभी��ासं�गक�सूचनाएं��दान�करना�है।�

2. इसका��लप�री�ड�ग�सॉ�टवेर�इसक���व�श���वशेषता�है�जो�कैमरा�के�मा�यम�से�श�द�पहचान�सकता�है�और�इसे�श�द�म��प�रव�त�त�कर�सकता
है।�

3. मोबाइल�ए�लीकेशन, UN क��क�व�शन�ओन�द�राइट् स�ऑफ़�पस��स��वद��डसे�ब�लट�ज�(UNCRPD) क��अनुपालना�म��है।�

उपयु���म��से�कौन�सा�कथन�सही�है?

a) 1 और�2
b) 2 और�3
c) 1 और�3
d) उपयु���सभी�
Correct Answer: C
Your Answer:
Explanation

Solution (c)

‘ �द�ांग�सारथी�’ — यह�‘�द�ांगजन�’ तक�सूचना�के�सरल��सरण�हेतु�प�ँच�यो�य�एवं��ापक�मोबाइल�ए�लीकेशन�है।�इस�मोबाइल


ए�लीकेशन�का�ल�य��डपाट� म�ट�ऑफ़�ए�पावरम�ट�ऑफ़�पस��स��वद��डसे�ब�लट�ज�(DEPwD) के�बारे�म��सभी��ासं�गक�सूचनाएं��दान�करना
है��जसम��प�ँच�यो�य��ा�प�म���व�भ��कलाएं, �नयम�एवं��व�नयम, �दशा�नद� श, योजनाएं, भागीदार�सं�थान, रोजगार�अवसर�तथा��डसे�ब�लट�
बाजार�शा�मल�है।�

ऑ�डयो�नोट् स�(टे ��ट�से�आवाज�म��प�रवत�न�वाला�सॉ�टवेयर) ‘ �द�ांग�सारथी�’ मोबाइल�ए�लीकेशन�क���व�श���वशेषता�है�जो�ए�लीकेशन


म��लगे�ह��तथा��ल�खत�सूचना�को�ऑ�डयो�फाइल�एवं�समायोजनयो�य�फॉ�ट�साइज़�म��बदल�दे ते�ह���जसे��यो�ा�क��आव�यकता�के�अनुसार
बदला�जा�सकता�है।�इस�मोबाइल�ए�लीकेशन�क��प�ँच��न��त��प�से��ापक�होगी��य��क�इसे���भाषीय��प�से��वक�सत��कया�गया�है
अथा�त�सूचना��ह�द��के�साथ-साथ�अं�ेजी�म��उपल�ध�है।�ए�लीकेशन�को�इस��कार��वक�सत��कया�गया�है��क�एक�बार�डाउनलोड�करने�पर
एं�ाइड��माट� फोन��यो�ा�(�जसके�पास�इ�टरनेट�प�ँच�हो�अथवा�न�हो) �ारा�इस�तक�प�ँच�बनाई�जा�सकती�है।�मोबाइल�ए�प�गूगल��ले�टोर
पर�डाउनलोड�हेतु�उपल�ध�है।�

QUESTION 64.
‘ द�नदयाल�अं�योदय�योजना�– रा�ीय�शहरी�आजी�वका��मशन�(DAY – NULM) के�बारे�म���न�न�ल�खत�म��से�कौन�से�कथन�सही�ह��?

1. इसे�आवास�एवं�शहरी�गरीबी�उ�मूलन�मं�ालय�(MHUPA) �ारा�शु���कया�गया�था।�

2. NULM का��ाथ�मक�ल�य�शहरी�गरीब�है��जसम��शहर��के�आवास�वहीन�लोग�शा�मल�ह�।�

3. रा�श�का�बंटवारा�क���और�रा�य��के�बीच�75:25 के�अनुपात�( पूव��र�रा�य��के�अलावा�) म��होगा।�

IASbaba
Web: http://ilp.iasbaba.com/ Score:
Email: ilp@iasbaba.com 0.00 / 198
Page 123
2019 - Test 32 -
Exam Title :
Government...
Email :
Contact :

�न�न�ल�खत�म��से�कूट�का�चयन�क��जए�:

a) 1 और�2
b) 2 और�3
c) 1 और�3
d) उपयु���सभी�
Correct Answer: D
Your Answer:
Explanation

Solution (d)

रा�ीय�शहरी�आजी�वका��मशन�(NULM) को�23 �सतंबर�2013 को�भारत�सरकार�के�आवास�एवं�शहरी�गरीबी�उ�मूलन�मं�ालय�(M


HUPA) �ारा�शु���कया�गया�था।�इसने��व�मान��वण��जयंती�शहरी�रोजगार�योजना�(SJSRY) का��थान��लया�था।�NULM शहरी�गरीब�
को�उनके�मजबूत�जमीनी��तर�के�सं�थान��पर�संग�ठत�करने, कौशल��वकास�हेतु�अवसर��को�पैदा�करने�पर��यान�क���त�करता�है��जससे�उ�ह�
बाजार-आधा�रत�रोजगार��ा�त�हो�सक��तथा�साख�तक�उनक��आसान�प�ँच�सु�न��त�कर�उ�ह���व-रोजगार�उ�म��था�पत�करने�म��मदद�क��जा
सके।��मशन�का�ल�य�चरणब��ढं ग�से�शहरी�आवास�वहीन�लोग��को�आव�यक�सेवा��से�सुस��जत�आ�य��दान�करना�है।�इसके�अ�त�र��,
�मशन�शहरी����ट�व�डरो�क��आजी�वका�संबंधी��च�ता��का�भी�समाधान�करेगा।�

NULM

क��मु�य��वशेषताएं�ह��:

कवरेज�: 12 व��पंचवष�य�योजना�म��, NULM को�सभी��जला�मु�यालय�क�ब��( भले�ही�उनक��जनसँ�या��कतनी�भी�हो�) तथा�जनगणना


2011 के�अनुसार�एक�लाख�अथवा�अ�धक�क��जनसँ�या�वाले�क�ब��म��लागू��कया�जाएगा।�वत�मान�म��790 शहर�NULM के�अंतग�त�ह�।�
हालां�क�, रा�य���ारा��नवेदन��कए�जाने�पर�अपवाद�पेण�अ�य�क�ब��को�भी�शा�मल�होने�क��अनुम�त�द��जा�सकती�है।�

ल��त�जनसँ�या�: NULM का�मु�य�ल�य�शहरी�गरीबा�ह�, �जनम��शहरी�आवास�वहीन�शा�मल�ह�।�

कोष��का�बंटवारा�: धनराशी�का�बंटवारा�क���एवं�रा�य��के�बीच�75:25 के�अनुपात�म���कया�जाएगा।�पूव��र�एवं��वशेष�दजा���ा�त�रा�य��( अ


�णाचल��दे श�, असम�, म�णपुर�, मेघालय�, �मजोरम�, नागाल�ड�, �स��कम�, ��पुरा�, ज�मू�व�क�मीर�, �हमाचल��दे श�तथा�उ�राखंड�) म�
यह�अनुपात�90:10 होगा।�

QUESTION 65.
‘ ��कल�फॉर�लाइफ�, सेव�ए�लाइफ�’ पहल�के�बारे�म���न�न�ल�खत�कथन��पर��वचार�क��जए�:

1. इसे�कौशल��वकास�एवं�उ��मता�मं�ालय��ारा�शु���कया�गया�है।�

2. इसका�ल�य�हे�थकेयर��णाली�म����श��त�पेशेवर��क��मा�ा�एवं�गुणव�ा�को�बढ़ाना�है।�

इस�पहल�के�बारे�म��उपयु���म��से�कौन�सा/से�कथन�सही�है/ह��?

a) केवल�1
b) केवल�2

IASbaba
Web: http://ilp.iasbaba.com/ Score:
Email: ilp@iasbaba.com 0.00 / 198
Page 124
2019 - Test 32 -
Exam Title :
Government...
Email :
Contact :

c) 1 और�2 दोन��
d) न�तो�1 न�ही�2
Correct Answer: B
Your Answer:
Explanation

Solution (b)

‘ ��कल�फॉर�लाइफ�, सेव�ए�लाइफ�’ पहल�का�ल�य�हे�थकेयर��णाली�म����श��त�पेशेवर��क��मा�ा�एवं�गुणव�ा�को�बढ़ाना�है।�इस�पहल


के�अंतग�त�हे�थकेयर�पेशेवर��के�साथ-साथ�आम�जनता�क���व�श��कुशलता��को�ल��त�करते��ए��व�भ��कोस�ज�क��शु�आत��कए�जाने�क�
योजना�है।�

इस�पहल�को��वा��य�एवं�प�रवार�क�याण�मं�ालय��ारा�शु���कया�गया�है।�

यह�यू�नवस�ल�हे�थ�कवरेज�को�सु�न��त�करने�वाली�एक�बड़ी�योजना�एवं�काय��म�का�भाग�है।�यह�काय��म�समुदाय�को�शा�मल�करने�हेतु
आधार�को��ापक�बनाने��ारा���श��त�एवं�कुशल�लोग�उपल�ध�करवाएगा।�ऐसे�काय��म��के�मा�यम�से�, हम�‘�थम���तपु��कता��’ का
एक�समूह�तैयार�करते�ह��जो�पया��त���श��त�पेशेवर��क���र���को�भरने�हेतु��वशेष���क����तपू�त��करते�ह�।�

QUESTION 66.
‘ वा�स�य�– मातृ�अमृत�कोष�’, जो��क�एक�नेशनल���मन��म�क�ब�क�एवं�लै�टे शन�काउंस�ल�ग�स�टर�है, का�लेडी�हा�ड�ग�मे�डकल�कॉलेज�(L
HMC) नई��द�ली�म��उदघाटन��कया�गया�है।�इसके�बारे�म���न�न�ल�खत�म��से�कौन�सा/से�कथन�सही�है/ह��?

1. इसे�डेनमाक��सरकार�के�सहयोग�से��था�पत��कया�गया�है।�

2. यह��ध��पलाने�वाली�माता���ारा�दान��दया�गया��ध�एक��करेगा, उसे�क�टाणु�र�हत�बनाएगा�तथा�उसे�सुर��त��प�से�सं��हत�करेगा�एवं
इसे�ज�रतमंद��शशु��हेतु�उपल�ध�करवाएगा।�

3. यह�क���उन�अ�य��म�क�ब��स�हेतु�ट��च�ग, �े �न�ग�एवं��दश�न��थल�के��प�म��काय��करेगा��ज�ह���वा��य�एवं�प�रवार�क�याण�मं�ालय�के
अंतग�त��था�पत��कया�जाएगा।�

�न�न�ल�खत�म��से�कूट�का�चयन�क��जए�:

a) 1 और�2
b) 2 और�3
c) 1 और�3
d) उपयु���सभी�
Correct Answer: B
Your Answer:
Explanation

Solution (b)

‘ वा�स�य�– मातृ�अमृत�कोष�’ , एक�नेशनल���मन��म�क�ब�क�एवं�लै�टे शन�काउंस�ल�ग�स�टर�है, �जसका�लेडी�हा�ड�ग�मे�डकल�कॉलेज�(LH


MC) नई��द�ली�म��उदघाटन��कया�गया�है।�यह�क���उ�र�भारत�म��उपल�ध�साव�ज�नक��े��का�सबसे�बड़ा���मन��म�क�ब�क�एवं�लै�टे शन
काउंस�ल�ग�स�टर�होगा�।

• �मुख��ब��: ‘ वा�स�य�– मातृ�अमृत�कोष�’ को�नोव��सरकार, ओ�लो��व��व�ालय�एवं�नॉव��इं�डया�पाट� नर�शप�इ�न�शए�टव�(NIPI)


के�सहयोग�से��था�पत��कया�गया�है�।
• ‘ वा�स�य�– मातृ�अमृत�कोष�’ �ध��पलाने�वाली�माता���ारा�दान��दया�गया��ध�एक��करेगा, उसे�क�टाणु�र�हत�बनाएगा�तथा�उसे
सुर��त��प�से�सं��हत�करेगा�एवं�इसे�ज�रतमंद��शशु��हेतु�उपल�ध�करवाएगा।�इस�डोनर���मन��म�क�ब�क�क���थापना�के�साथ,
�द�ली�और�उसके�आसपास�के�सभी�नवजात��क��जीवन�र�क���मन��म�क�तक�प�ँच�होगी�(भले�ही�ज�म�के�समय�उनक�
प�र��थ�तयां�कुछ�भी�ह�)।�
• यह�क���सम�प�त��तनपान�परामश�दाता��क��सहायता�से�सभी�माता��को��तनपान�समथ�न�दे कर��तनपान�को�सुर��त, �ो�सा�हत�एवं
समथ�न�दे गा�।

IASbaba
Web: http://ilp.iasbaba.com/ Score:
Email: ilp@iasbaba.com 0.00 / 198
Page 125
2019 - Test 32 -
Exam Title :
Government...
Email :
Contact :
• इस�संबंध�म��, ब�चे�क��रोग-��तरोधक��मता�बढाने�हेतु�माता��के�बीच��तनपान�को�लेकर�जाग�कता�बढाने�के��लए��वा��य�एवं
प�रवार�क�याण�मं�ालय�ने�मदस��अ�सोलुट�अफे�शन�(MAA) काय��म�भी�शु���कया�है�।
• यह�सु�वधा��तनपान�को�समथ�न�दे ने�तथा��शशु�सुर�ा�को�सुधारने�के�समथ�न�हेतु�एक�सम�प�त�क���के��प�म��काय��करेगा�। यह�क���उन
अ�य��म�क�ब��स�हेतु�ट��च�ग, �े �न�ग�एवं��दश�न��थल�के��प�म��काय��करेगा��ज�ह���वा��य�एवं�प�रवार�क�याण�मं�ालय�के�अंतग�त
�था�पत��कया�जाएगा।�

QUESTION 67.
�मशन�‘ प�रवार��वकास�’ का��न�न�ल�खत�म��से�कौन�सा�उ�े �य�है�?

a) पारसी�समुदाय�क��जनसं�या�के�दोगुनी�होने�क��दर�को�बढ़ाना।�
b) �थानीय�ब�च��को��न:शु�क��वा��य�एवं��श�ा�जैसे��ो�साहन�दे कर�पूव��र�रा�य��म��कुल��जनन�दर�(TFR) को�बढ़ाना।�
c) �नरोधक��एवं�प�रवार��नयोजन�तकनीक��के�बारे�म��जाग�कता��दान�कर�उ�च�जनसँ�या��जल��म��TFR को�घटाना।�
d) शहरी��व�तार��म��गरीबी�रेखा�से�नीचे�रहने�वाले�प�रवार��के�जीवन��तर�को�बढ़ाना।�
Correct Answer: C
Your Answer:
Explanation

Solution (c)

�वा��य�एवं�प�रवार�क�याण�मं�ालय�ने�दे श�के�145 उन�उ�च�फोकस��जल��म��“ �मशन�प�रवार��वकास�” शु���कया�है�जहाँ�कुल��जनन


दर�सवा��धक�है�। ये�145 �जले�सात�उ�च�फोकस, उ�च�TFR रा�य��म��ह��जो�ह�- उ�र��दे श, �बहार, राज�थान, म�य��दे श, छ�ीसगढ़,
झारखंड�और�असम�।�ये�दे श�क��जनसँ�या�का�44% ह��। ‘ �मशन�प�रवार��वकास�’ का�मु�य�उ�े �य�अ�धकार-आधा�रत��ेमवक��के�भीतर
सूचना, �व�नीय�सेवा��तथा�आपू�त�य��पर�आधा�रत�उ�च�गुणव�ा�प�रवार��नयोजन�चयन��तक�प�ँच�म��तेजी�लाना�है�।

इन�145 �जल��को�कुल��जनन�दर�और�सेवा��दायगी�(PPIUCD एवं�बं��करण��न�पादन�) के�आधार�पर�2025 तक�2.1 के���त�थापन


�तर��जनन�ल�य��क���ा��त�हेतु��वशेष�एवं�ती���यास��के��लए��च��त��कया�गया�है�।�हा�लया�आंकड़े�बताते�ह���क�इन�145 �जल��क��TFR
3.0 (7 HFS के�261 �जल��का�56%) से�अ�धक�अथवा�बराबर�है�तथा�यहाँ�भारत�क��जनसँ�या�का�28% �नवास�करता�है�( लगभग�3
3 करोड़�) । हालां�क�, भारत�के�सुर��त�युगल��का�केवल�22% तथा�अधूरी�ज�रत�वाले�युगल��का�40% इन��जल��म���नवास�करता�है�। इ
न��जल��म��मातृ�व�एवं��शशु��वा��य�संकेतक��पर�भी��वशाल��भाव��दखता�है��य��क�मातृ�व�मृ�यु��का�लगभग�25-30% तथा��शशु�मृ�यु
का�50% इन��जल��म��घ�टत�होता�है�।�साथ�ही�, इन��जल��म��से�115 �जल��(79%) म���कशोर�माता��का�उ�च���तशत�है�।

इस�पहल�का��मुख�साम�रक�फोकस�सु�न��त�सेवा��दायगी, नई��ो�साहन�योजना��के�साथ�तालमेल, व�तु�सुर�ा�सु�न��त�करके, �मता


�नमा�ण�(सेवा��दाता), �नकट��नगरानी�व�काया��वयन�के�साथ�स�म�बनाने�वाले�वातावरण�का��नमा�ण�करने�के�मा�यम�से��नरोधक��तक�प�ँच
सुधारने�पर�होगा�।

�मशन�सभी�145 �जल��म��एक�साथ�लागू��कया�जाएगा�।

QUESTION 68.
क���य��वा��य�एवं�प�रवार�क�याण�मं�ालय�ने��कस�बीमारी�से�जूझ�रहे�लोग��हेतु��मशन�संपक��आरंभ��कया-

a) एचआईवी�
b) �य�रोग�
c) वे�टर�बोन���डजीज�
d) डाय�रया�
Correct Answer: A
Your Answer:
Explanation

Solution (a)

क���य��वा��य�एवं�प�रवार�क�याण�मं�ालय�ने�2030 तक�HIV/AIDS को�समा�त�करने�के�ल�य�के�साथ�रा�ीय�साम�रक�योजना�2017-


24 शु���कया�है।�

IASbaba
Web: http://ilp.iasbaba.com/ Score:
Email: ilp@iasbaba.com 0.00 / 198
Page 126
2019 - Test 32 -
Exam Title :
Government...
Email :
Contact :

�मशन�संपक��को�उन�लोग��का�पता�लगाने��ज�ह��फॉलो-अप�करना�है�तथा�एंट�रे�ोवायरल�थेरेपी�(ART) सेवा��के�दायरे�म��लाया�जाना�है, के
�लए�भी�शु���कया�गया�था।�

90:90:90 रणनी�त�:

यह�UNAIDS काय��म�का�HIV उपचार�हेतु�एक�नया��ववरण�है��जसने��न�न�ल�खत�ल�य��नधा��रत��कए�ह�-

· 90% HIV ��त�लोग��को�अपने�HIV �टे टस�का�पता�लगे�(90% �नदान�),

· HIV ��त�लोग��म��90% �नरंतर�एंट�रे�ोवायरल�थेरेपी��ा�त�कर�गे�(90% का�HIV उपचार�) तथा�

· एंट�रे�ोवायरल�थेरेपी��ा�त�कर�रहे�90% लोग��के��वषाणु�का�दमन�होगा�(90% �वषाणु�दमन�) ।

QUESTION 69.
�याय��वभाग�ने�‘�ो�बोनो�लीगल�स�व�सेज’ पहल�शु��क��है।�इस�पहल�के�बारे�म���न�न�ल�खत�म��से�कौन�से�कथन�सही�ह��?

1. यह�एक�वेब�आधा�रत��लेटफाम��है��जसके�मा�यम�से�इ�छु क�वक�ल�गरीब�वा�दय��के��लए�(जो�मुकदमे�का�खच��वहन�करने�म��अ�म�ह�)
�वयं�को��ो�बोनो�स�व�सेज�हेतु�पंजीकृत�करा�सकते�ह�।�

2. यह�सभी�वा�दय��को�कानूनी�मदद�एवं��ो�बोनो�लॉयस��से�परामश��हेतु�आवेदन�करने�का�एक�मंच�भी��दान�करेगा।�

�न�न�ल�खत�म��से�कूट�का�चयन�क��जए�:

a) केवल�1
b) केवल�2
c) 1 और�2 दोन��
d) न�तो�1 न�ही�2
Correct Answer: A
Your Answer:
Explanation

Solution (a)

�ो�बोनो�लीगल�स�व�सेज�

‘�ो�बोनो�लीगल�स�व�सेज’ पहल�एक�वेब�आधा�रत��लेटफाम��है��जसके�मा�यम�से�इ�छु क�वक�ल�गरीब�वा�दय��के��लए�(जो�मुकदमे�का�खच�


वहन�करने�म��अ�म�ह�) �वयं�को��ो�बोनो�स�व�सेज�हेतु�पंजीकृत�करा�सकते�ह�।��याय��वभाग�ने�इस�पहल�हेतु�अपनी�वेबसाइट�doj.gov.in प
र�ऑनलाइन�ए�लीकेशन�शु��क��है।�इस�ऑनलाइन�पोट� ल�के�मा�यम�से�, सीमा�त�समुदाय��से�वाद��( अनुसू�चत�जा�तय��व�अनुसू�चत
जनजा�तय�, म�हला�, ब�च�, व�र��नाग�रक�, कम�आय�वाले�लोग��तथा�अ�म�लोग��स�हत) �ो�बोनो�वक�ल��से�कानूनी�सहायता�तथा
परामश��हेतु�आवेदन�कर�सकते�ह�।�

QUESTION 70.
क�मट��ो�ाम�के�बारे�म���न�न�ल�खत�कथन��पर��वचार�क��जए�:

1. यह�रा�य�सरकार�के�कम�चा�रय��हेतु�एक�इंड�शन��े �न�ग�काय��म�है।�

2. यह�काय��म�साव�ज�नक�सेवा��दायगी�को�सुधारने�एवं�उन�अ�धका�रय��क���मता��नमा�ण�के�मा�यम�से�नाग�रक�के���त��शासन��दान
करने�का�ल�य�रखता�है�जो�दै �नक�आधार�पर�जनता�से�अंत:��या�करते�ह�।�

3. इस�का��कम�का���या�वयन�रा�य��शास�नक���श�ण�सं�थान�(ATIs) के�मा�यम�से��कया�जाएगा।�

उपयु���म��से�कौन�सा�कथन�सही�है?

a) 1 और�2

IASbaba
Web: http://ilp.iasbaba.com/ Score:
Email: ilp@iasbaba.com 0.00 / 198
Page 127
2019 - Test 32 -
Exam Title :
Government...
Email :
Contact :

b) 2 और�3
c) 1 और�3
d) उपयु���सभी�
Correct Answer: D
Your Answer:
Explanation

Solution (d)

का�म�क, लोक�प�रवाद�एवं�प�शन�मं�ालय�ने�रा�य�सरकार�के�अ�धका�रय��हेतु�एक��े �नग�काय��म�को��ेह��सवे�ऑनलाइन�मो�डफाइड


मोडयु�स�ओन�इंड�शन��े �न�ग�(COMMIT) शु���कया�है।�यह��े �न�ग�काय��म�साव�ज�नक�सेवा��दायगी�त���को�सुधारने�एवं�उन
अ�धका�रय��क���मता��नमा�ण�के�मा�यम�से�नाग�रक�के���त��शासन��दान�करने�का�ल�य�रखता�है�जो�दै �नक�आधार�पर�जनता�से�अंत:��या
करते�ह�।�

यह�काय��म�पूव��के���श�ण�काय��म�का�पूरक�बनेगा�और�COMMIT का�लाभ�यह�है��क�यह�एक�लागत��भावी�काय��म�है�और�इसम�
�व�मान�12- �दनी�इंड�शन��े �न�ग�काय��म�(ITP) के�मुकाबले���तवष��3.3 अ�धका�रय��को�कवर�करने�क��संभावना�है।�

COMMIT काय��म�का��वकास�DoPT �ारा�सं. रा. �वकास�काय��म�(UNDP) के�सहयोग�से��कया�गया�है।�

यह�काय��म��व�ीय�वष��2017-18 म��लगभग�74,000 रा�य�सकरार�के�अ�धका�रय��को�आ�छा�दत�करेगा।�इसक��अव�ध�28 घंटे�क�


होगी��जसम��20 घंटे�का�ई-मो�ूल�तथा�8 घंटे�क��फेस-टू -फेस��े �न�ग�शा�मल�होगी।�20 घंटे�क��ई-�े �न�ग��व�श���प�से��वक�सत��कये�गए�1
2 जेने�रक�एवं�3 डोमेन��व�श��ई-मोडयु�स�के�मा�यम�से�द��जाएगी।�सॉ�ट���क�स�पर�मो�ूल�को�ई-मोडयु�स�एवं�फेस-टू -फेस��े �न�ग�के
मा�यम�से��दया�जाएगा�तथा�डोमेन�मोडयु�स�को�केवल�ई-मोडयु�स�के�मा�यम�से�कवर��कया�जाएगा।�इस�का��कम�का���या�वयन�रा�य
�शास�नक���श�ण�सं�थान�(ATIs) के�मा�यम�से��कया�जाएगा।�

QUESTION 71.
‘ द�नदयाल�उपा�याय��ाम��यो�त�योजना�(DDUGJY)’ के�बारे�म���न�न�ल�खत�म��से�कौन�सा�कथन�सही�है? ?

1. यह��ामीण��े���म��LED आधा�रत����ट�लाइ�ट�ग�उपल�ध�करवाने�का�ल�य�रखता�है।�

2. यह�सब-�ांस�मशन�एवं��वतरण�नेटवक��को�मजबूत�बनाने�का�ल�य�रखता�है�ता�क�पॉवर�लोस�को�रोका�जा�सके।�

3. यह��ामीण�घर��एवं�कृ�ष�उ�े �य��के��लए�फ�डर�पृथ�करण�पर��यान�के���त�करता�है।�

�न�न�ल�खत�म��से�कूट�का�चयन�क��जए�:

a) 1 और�2
b) 2 और�3
c) 1 और�3
d) उपयु���सभी�
Correct Answer: B
Your Answer:
Explanation

Solution (b)

द�नदयाल�उपा�याय��ाम��यो�त�योजना�

भारत�सरकार�के��बजली�मं�ालय�ने��न�न�ल�खत�उ�े �य��के�साथ��ामीण��े���म��द�नदयाल�उपा�याय��ाम��यो�त�योजना�शु��क��है�:

· सभी��ाम��म���बजली�प�ंचाना�

· �कसान��को�पया��त��बजली�एवं�अ�य�उपभो�ा��को��नय�मत�आपू�त��सु�न��त�करने�के��लए�फ�डर�पृथ�करण�

IASbaba
Web: http://ilp.iasbaba.com/ Score:
Email: ilp@iasbaba.com 0.00 / 198
Page 128
2019 - Test 32 -
Exam Title :
Government...
Email :
Contact :

· आपू�त��क��गुणव�ा�एवं��व�सनीयता�सुधारने�हेतु�सब-�ांस�मशन�एवं��वतरण�नेटवक��का�सुधार�

· हा�नय��को�कम�करने�हेतु�मीट�र�ग�

QUESTION 72.
�ामीण��वकास�मं�ालय�ने�‘ आजी�वका��ामीण�ए�स�ेस�योजना�’ शु��क��है।�इस�योजना�का�ल�य�है-

a) भारत�भर�के�सभी��ामीण��े���को�ए�स�ेस��े न��के�मा�यम�से�माल�ढु लाई�प�रवहन�को�आसान�बनाने�हेतु�शहरी��े���से�जोड़ना।�


b) �पछड़े��ामीण��े���म��साव�ज�नक�प�रवहन�सेवा��के�संचालन�म��मदद�कर��व-सहायता�समूह��(SHGs) के�सद�य��को�आजी�वका�का
एक�वैक��पक�साधन��दान�करना।�
c) �ामीण�गरीब��को�ए�स�ेस��े न��क���लीपर��ेणी�म��कम�दर��पर��टकेट��दान�करना।�
d) नाग�रक�सु�वधा��का�सृजन�कर��ामीण��े���म��रेलवे��टे शन�के��नकट�के��े���को��वक�सत�करना।�
Correct Answer: B
Your Answer:
Explanation

Solution (b)

भारत�सरकार�ने�द�नदयाल�अं�योदय�योजना�– रा�ीय��ामीण�आजी�वका��मशन�(DAY – NRLM) के�एक�भाग�के��प�म��एक�उप-योजना


�जसका�नाम�“ आजी�वका��ामीण�ए�स�ेस�योजना�(AGEY)” शु��क��है।�DAY-NRLM के�अंतग�त��व-सहायता�समूह��पछड़े�इलाक��म�
सड़क�प�रवहन�सेवा�को�संचा�लत�कर�गे।�यह��पछड़े��ामीण��े���के�सम��आ�थ�क��वकास�हेतु�सु�र��ाम��को��मुख�सेवा��एवं�सु�वधा�
(जैसे�बाजार, �श�ा�एवं��वा��य�तक�प�ँच) से�जोड़ने�के��लए�सुर��त, वहनीय�एवं�समुदाय�क���नगरानी�वाली��ामीण�प�रवहन�सेवाएं��दान
करेगा।�यह�SHGs हेतु�आजी�वका�के�अ�त�र��साधन�भी��दान�करेगा।�

समुदाय�आधा�रत�संगठन�(CBOs) को�DAY-NRLM के�अंतग�त��द��समुदाय��नवेश�कोष�(CIF) इस�नई�आजी�वका�पहल�म��SHG स


द�य��को�समथ�न�दे ने�हेतु��योग��कए�जाय�गे।�लाभाथ��SHG सद�य��को�CBO �ारा�समुदाय��नवेश�कोष�से�वाहन��य�हेतु�6.50 लाख
�पये�तक�का��याज�मु��ऋण��दान��कया�जाएगा।�वैक��पक��प�से�CBO वाहन�का��वामी�होगा�तथा�इसे�चलाने�हेतु�SHG सद�य�को
लीज�पर�दे गा�जो�CBO को�लीज��कराए�का�भुगतान�करेगा।�

QUESTION 73.
‘ पहाड़ी��े���वकास�काय��म�’ को��कन�पहाड़ी��े���के��वकास�हेतु�शु���कया�गया�है-

a) उ�राखंड�
b) प��मी�घाट�
c) पूव��र�रा�य�
d) म�य��दे श�एवं�छ�ीसगढ़�
Correct Answer: C
Your Answer:
Explanation

Soution (c)

पूव��र��े���वकास�मं�ालय�ने�पहाड़ी��े���वकास�काय��म�शु���कया�है।�

म�णपुर, ��पुरा�एवं�असम�के�पहाड़ी��े���क��एक�अलग�भू-भौ�तक�पहचान�है�तथा�वे�सामा�जक-आ�थ�क��वकास�म���पछड़�रहे�ह�।��व�च�
भूसंरचना�के�प�रणाम�व�प�अवसंरचना, सडक��क��गुणव�ा, �वा��य�एवं��श�ा�आ�द�के�मामले�म��पहाड़ी�एवं�घाट���जल��के�बीच��व�तृत
अंतराल�है।�पहाड़ी��े���वकास�काय��म�इन�सभी�कारक��के�गंभीर�शोध�एवं��वचार-�वमश��से��े�रत�है।�

काय��म�को�म�णपुर�के�पहाड़ी��जल��म��पायलट�आधार�पर�शु���कया�गया�था।�

सरकार�एक�दोतरफा����कोण�अपनाएगी�:

IASbaba
Web: http://ilp.iasbaba.com/ Score:
Email: ilp@iasbaba.com 0.00 / 198
Page 129
2019 - Test 32 -
Exam Title :
Government...
Email :
Contact :

· पूव��र��े��के���येक��े�, ��येक�तबके�एवं�यहाँ��नवास�कर�रही���येक�जनजा�त�का�समान��वकास�सु�न��त�करना।�

· पूव��र��े��के�आठ�रा�य��को�भारत�के�अ�य��वक�सत�रा�य��के�समक��लाना।�

QUESTION 74.
�व�ान�एवं��ौ�ो�गक��मं�ालय�ने�इनोवेट�इन�इं�डया�(i3) �मशन�शु���कया�है।��मशन�के�बारे�म���न�न�ल�खत�म��से�कौन�सा/से�कथन�सही�है/
ह�?

1. यह�र�ा, रोबो�ट�स, नैनो��व�ान�, बायोफामा��तथा�आईट��व�क�यु�नकेशन�क��उ�त��ौ�ो�गक��म��शोध�को��ो�सा�हत�करने�का�ल�य


रखता�है।�

2. यह�भारत�म��बायोफामा��यु�टकल��वकास�को�ग�त�दे ने�हेतु�अपने��क�म�का�पहला�इंड���-एकेडे�मया��मशन�है।�

�न�न�ल�खत�म��से�कूट�का�चयन�क��जए:

a) केवल�1
b) केवल�2
c) 1 और�2 दोन��
d) न�तो�1 न�ही�2
Correct Answer: B
Your Answer:
Explanation

Solution (b)

इनोवेट�इन�इं�डया�(i3) अथा�त�रा�ीय�बायोफामा���मशन�

एक�ऐसी�वै��क��प�से���त�पधा��मक�बायोफामा��यू�टकल�इंड����सृ�जत�करने�हेतु�जो�दे श�क��वहनीय��वा��य�दे खभाल�के�इद� �गद� �क���मुख


बाधा��का�समाधान�कर�सके, इनोवेट�इन�इं�डया�(i3) काय��म�शु���कया�गया�था�।

$250 �म�लयन�धनराशी�वाला��व�ान�एवं��ौ�ो�गक��मं�ालय�के��डपाट� म�ट�ऑफ़�बायोटे �नोलॉजी�(DBT) का�यह�काय��म�अपनी��कार


का�पहल��मशन�है�जो�बायोफामा��यु�टकल��े��म��उ��मता�एवं��वदे शी��व�नमा�ण�को��ो�सा�हत�करने�हेतु�इंड����एवं�एकेडे�मया�को�एक�साथ
लाता�है।��व��ब�क�के�सहयोग�से�भारत�सरकार�का�यह��लैग�शप�काय��म�बायोटे �नोलॉजी�इंड�����रसच��अ�स�ट� स�क��सल�(BIRAC) �ारा
लागू��कया�जाएगा�जो��क�DBT �ारा��था�पत�साव�ज�नक��े��का�उप�म�है।�

QUESTION 75.
‘ पया�वरण�एवं��वकास�पर��रओ�उ�ोषणा�’ के��वषय�म���न�न�ल�खत�कथन��पर��वचार�क��जए।�

1. यह�घो�षत�करती�है��क�दे शज�लोग��क��पया�वरण��बंधन�म��मह�वपूण��भू�मका�है।�

2. इसम��पूव���या�मक��स�ांत�का��नमा�ण�एवं�पो�युटर�पेज��स�ांत�शा�मल�है।�

सही�कथन�चु�नए�

a) केवल�1
b) केवल�2
c) 1 और�2 दोन��
d) न�तो�1 न�ही�2
Correct Answer: C
Your Answer:
Explanation

IASbaba
Web: http://ilp.iasbaba.com/ Score:
Email: ilp@iasbaba.com 0.00 / 198
Page 130
2019 - Test 32 -
Exam Title :
Government...
Email :
Contact :

Solution (c)

दोन��कथन�सही�ह�।�

�रयो��ड�लेरेशन�ऑन�एनवायरनम�ट�एंड�डेवलपम�ट�, �जसे�अ�सर��रयो��ड�लेरेशन�से�छोटा��कया�जाता�था�, 1992 के�संयु��रा��"पया�वरण


और��वकास�स�मेलन" ( UNCED) म���न�म�त�एक�छोटा�द�तावेज�था�, �जसे�अनौपचा�रक��प�से�पृ�वी��शखर�स�मेलन�के��प�म��जाना
जाता�है।��रयो�घोषणा�म��27 �स�ांत��का�समावेश�था�, जो�भ�व�य�म��सतत��वकास�म��दे श��का�माग�दश�न�करने�के��लए�थे।�इस�पर�170 से
अ�धक�दे श��ने�ह�ता�र��कए।�

�वदे शी�लोग��और�उनके�समुदाय��, और�अ�य��थानीय�समुदाय��, उनके��ान�और�पारंप�रक��था��के�कारण�पया�वरण��बंधन�और��वकास�म�


एक�मह�वपूण��भू�मका�है।�रा�य��को�अपनी�पहचान�, सं�कृ�त�और��हत��को�पहचानना�और�उनका��व�धवत�समथ�न�करना�चा�हए�और�सतत
�वकास�क��उपल��ध�म��उनक���भावी�भागीदारी�को�स�म�बनाना�चा�हए।�

पया�वरण�क��सुर�ा�के��लए�, एह�तयाती����कोण�को�रा�य���ारा�उनक���मता��के�अनुसार��ापक��प�से�लागू��कया�जाएगा।�जहां�गंभीर
या�अप�रवत�नीय���त�के�खतरे�ह��, पया�वरणीय��गरावट�को�रोकने�के��लए�लागत��भावी�उपाय��को��थ�गत�करने�के�कारण�के��प�म��पूण�
वै�ा�नक��न��तता�क��कमी�का�उपयोग�नह���कया�जाएगा।�

रा�ीय�अ�धका�रय��को�पया�वरण�लागत��के�आंत�रककरण�और�आ�थ�क�साधन��के�उपयोग�को�बढ़ावा�दे ने�का��यास�करना�चा�हए�, इस
���कोण�को��यान�म��रखते��ए��क���षण�को�साव�ज�नक��हत�के��बना�और�अंतरा��ीय��ापार�को��वकृत��कए��बना���षण�क��लागत�को
सहन�करना�चा�हए।�

QUESTION 76.
भारत��न�न�ल�खत�म��से��कसका�प�कार/ह�ता�रकता��है?

1. मानव�अ�धकार��पर�वै��क�उ�ोषणा�(UDHR)

2. दे शज�लोग��के�अ�धकार��पर�उ�ोषणा�(UNDRIP)

3. �वदे शी�एवं�जनजातीय�लोग��पर�अ�भसमय, 1989

सही�कूट�चु�नए:

a) 1 और�2
b) 2 और�3
c) 1 और�3
d) उपयु���सभी�
Correct Answer: A
Your Answer:
Explanation

Solution (a)

भारत�'�वदे शी�और�जनजातीय�पीपु�स�क�व�शन, 1989' का�ह�ता�रकता��नह��है, ले�कन��वदे शी�और�जनजातीय�आबाद��क�व�शन, 1957


का�ह�ता�रकता��है।�

भारत��वदे शी�लोग��के�अ�धकार��(UNDRIP या�DOTROIP) और�मानव�अ�धकार��क��साव�भौ�मक�घोषणा�(UDHR) पर�संयु��रा�


घोषणा�के��लए�एक�ह�ता�रकता��है।�

IASbaba
Web: http://ilp.iasbaba.com/ Score:
Email: ilp@iasbaba.com 0.00 / 198
Page 131
2019 - Test 32 -
Exam Title :
Government...
Email :
Contact :

QUESTION 77.
‘ �जनेवा�क�व�शन�’ के�बारे�म���न�न�ल�खत�कथन��पर��वचार�क��जए।�

1. यु�बं�दय��के�साथ��वहार�का�मामला�तीसरी�क�व�शन�अथवा�सं�ध�के�अंतग�त�आता�है।�

2. पहली�क�व�शन�यु�काल�के�दौरान�नाग�रक��के�साथ��वाहर�एवं�उनक��सुर�ा�को�दे खती�है।�

सही�कथन�चु�नए�

IASbaba
Web: http://ilp.iasbaba.com/ Score:
Email: ilp@iasbaba.com 0.00 / 198
Page 132
2019 - Test 32 -
Exam Title :
Government...
Email :
Contact :

a) केवल�1
b) केवल�2
c) 1 और�2 दोन��
d) न�तो�1 न�ही�2
Correct Answer: A
Your Answer:
Explanation

Solution (a)

�जनेवा�क�व�शन�

· वे�यु�बं�दय��( POWs) क��र�ा�करने�वाले��नयम�ह�।�

· वे�पहले�1929 के�जेनेवा�क�व�शन�म���व�तृत�थे�और�बाद�म����तीय��व��यु��के�सबक�के�बाद�तीसरे�1949 के�जेनेवा�क�व�शन�म��संशोधन


�कया�गया।�

· चार��जनेवा�अ�भसमय�ह�।�

· तीसरा��जनेवा�क�व�शन�प�रभा�षत�करता�है�और��ववरण��दया�जाता�है��क��कसे�POW माना�जा�सकता�है�और�उसे��कस�तरह�से��वहार
�कया�जाना�चा�हए।�

तीसरा��जनेवा�अ�भसमय�

· तीसरा��जनेवा�क�व�शन�यु��के�कै�दय��के��लए�सुर�ा�क��एक��व�तृत��ृंखला��दान�करता�है।�यह�उनके�अ�धकार��को�प�रभा�षत�करता�है
और�उनके�उपचार�और�अं�तम��रलीज�के��लए��व�तृत��नयम��नधा��रत�करता�है।�POW क����थ�त�केवल�अंतरा��ीय�सश���संघष��म��लागू�होती
है।�

· POW आमतौर�पर��कसी�एक�प��के�सश���बल��के�एक�संघष��के�सद�य�होते�ह��जो���तकूल�पाट��के�हाथ��म��आते�ह�।�तीसरे�1949 के
जेनेवा�क�व�शन�म��उन����य��क��अ�य��े�णय��को�भी�वग�कृत��कया�गया�है��जनके�पास�POW का�अ�धकार�है�या�उ�ह��POW के��प�म�
माना�जा�सकता�है।�

· श�ुता�म����य��भाग�लेने�पर�POW का�मुकदमा�नह��चलाया�जा�सकता।�उनक��नजरबंद��सजा�का�एक��प�नह��है�, ब��क�केवल�संघष��म�


आगे�क��भागीदारी�को�रोकने�के��लए�है।�श�ुता�समा�त�होने�के�बाद�उ�ह���बना�दे री��कए��रहा��कया�जाना�चा�हए।��नरोध�श���संभव�यु�
अपराध��के��लए�उन�पर�मुकदमा�चला�सकती�है�, ले�कन��ह�सा�के�कृ�य��के��लए�नह��जो�अंतररा�ीय�मानवीय�कानून�के�तहत�वैध�ह�।�

· "POWs को�सभी�प�र��थ�तय��म��मानवीय��प�से��वहार��कया�जाना�चा�हए।�वे��ह�सा�के��कसी�भी�काय��के�साथ-साथ�धमक��, अपमान


और�साव�ज�नक��ज�ासा�के��खलाफ�संर��त�ह�।"

उ�च�अनुबंध�वाले�दल��म��से�एक�के��े��म��होने�वाले�एक�अंतररा�ीय�च�र��के�सश���संघष��के�मामले�म��, संघष��के��लए���येक�पाट��एक
�यूनतम�के��प�म��, �न�न�ल�खत��ावधान��को�लागू�करने�के��लए�बा�य�होगी:

( १) श�ुता�म��कोई�स��य��ह�सा�नह��ले�रहा�है�, �जसम��सश���बल��के�सद�य�शा�मल�ह��, �ज�ह�ने�अपने�ह�थयार�डाल��दए�ह��और�उन�लोग�


ने�बीमारी�, घाव�, �नरोध�, या��कसी�अ�य�कारण�से�बचाव��कया�है�, सभी�प�र��थ�तय��म��मानवीय��प�से��वहार��कया�जाएगा�, �बना�जा�त
, रंग�, धम��या�आ�था�, �ल�ग�, ज�म�या�धन�, या��कसी�अ�य�समान�मापदं ड�पर��था�पत�कोई���तकूल�अंतर।�

इसके��लए��न�न�ल�खत�कृ�य��कसी�भी�समय�और��कसी�भी��थान�पर��न�ष��रह�गे�और�जो�भी�उपरो�����य��के�संबंध�म��होगा:

· जीवन�और�����के��लए��ह�सा�, �वशेष��प�से�सभी��कार�क��ह�या�, उ�प�रवत�न�, �ूर�उपचार�और�यातना�;

· बंधक��को�लेना�;

· ���गत�ग�रमा�पर�आघात�, �वशेष��प�से�अपमानजनक�और�अपमानजनक�उपचार�;

IASbaba
Web: http://ilp.iasbaba.com/ Score:
Email: ilp@iasbaba.com 0.00 / 198
Page 133
2019 - Test 32 -
Exam Title :
Government...
Email :
Contact :

· वा�य��को�पा�रत�करना�और��बना��कसी��नय�मत��प�से�ग�ठत�अदालत��ारा�सुनाए�गए��पछले�फैसले�के��बना�सभी��या�यक�गारं�टय��को
खा�रज�करना�, जो�स�य�लोग���ारा�अप�रहाय��के��प�म��पहचाने�जाते�ह�।�

(2) घायल��और�बीमार��क��दे खभाल�क��जाएगी�.

· एक��न�प��मानवीय��नकाय�, जैसे��क�रेड��ॉस�क��अंतरा��ीय�स�म�त�, संघष��के��लए�पा�ट�य��को�अपनी�सेवाएँ��दान�कर�सकती�है।�

· वत�मान�समझौते�के�अ�य��ावधान��के��वशेष�समझौत��के�मा�यम�से�, संघष��के�प��को�और�अ�धक��भावी�बनाने�का��यास�करना�चा�हए।�

· पूव�वत���ावधान��के�आवेदन�संघष��के��लए�पा�ट�य��क��कानूनी���थ�त�को��भा�वत�नह��करेगा।�

�जनेवा�स�मेलन��म��"सुर�ा�श��य�" क���व�था�है�जो�यह�सु�न��त�करते�ह���क�दल��के�संघष��के��ावधान��का��वरोध��कया�जा�रहा�है।
�स�ांत��प�म��, ��येक�प��को�उन�रा�य��को�ना�मत�करना�चा�हए�जो�संघष��के�प��म��नह��ह��, �य��क�उनक��"र�ा�करने�वाली�श��यां" ह�।
�वहार�म��, रेड��ॉस�क��अंतरा��ीय�स�म�त�आमतौर�पर�इस�भू�मका�को��नभाती�है।�

क�व�शन�

· �थम��जनेवा�क�व�शन�"फ��ड�म��सश���बल��म��घायल�और�बीमार�क����थ�त�के�संशोधन�के��लए" (पहली�बार�1864 म��अपनाया�गया�,


1906, 1929 और�अंत�म��1949 म��संशो�धत) ;

· �सरा��जनेवा�क�व�शन�"ज�मी�, बीमार�और�समु��म��सश���बल��के�जहाज��के�संशोधन�के��लए" (पहली�बार�1949 म��अपनाया�गया�, हेग


क�व�शन�(ए�स) 1907 के�उ�रा�धकारी) ;

· तीसरा��जनेवा�क�व�शन�"यु��के�कै�दय��के�उपचार�के�सापे�" (पहली�बार�1929 म��अपनाया�गया�, 1949 म��अं�तम�संशोधन) ;

· चौथा��जनेवा�क�व�शन�"यु��के�समय�म��नाग�रक����य��के�संर�ण�के�सापे�" (पहली�बार�1949 म��अपनाया�गया�, 1899 के�हेग


क�व�शन�(��तीय) और�हेग�क�व�शन�( IV) 1907 के�कुछ��ह�स��पर�आधा�रत)।�

1949 के�स�मेलन��को�तीन�संशोधन��ोटोकॉल�के�साथ�संशो�धत��कया�गया�है�:

· �ोटोकॉल�I (1977) अंतरा��ीय�सश���संघष��के�पी�ड़त��के�संर�ण�से�संबं�धत�है�

· �ोटोकॉल�II (1977) गैर-अंतरा��ीय�सश���संघष��के�पी�ड़त��के�संर�ण�से�संबं�धत�है�

· �ोटोकॉल�III (2005) एक�अ�त�र���व�श���तीक�को�अपनाने�से�संबं�धत�है�

नोट�- भारत�चार�स�मेलन��के��लए�पाट��है�, ले�कन�अ�त�र���ोटोकॉल�नह��

QUESTION 78.
‘ टोच��इ�फे�शंस�’ संबं�धत�ह�-

1. यौन�संचा�रत�रोग��से�

2. �ूण�असामा�यता��से�

सही�कथन�चु�नए�:

a) केवल�1
b) केवल�2
c) 1 और�2 दोन��
d) न�तो�1 न�ही�2
Correct Answer: C
Your Answer:
Explanation

IASbaba
Web: http://ilp.iasbaba.com/ Score:
Email: ilp@iasbaba.com 0.00 / 198
Page 134
2019 - Test 32 -
Exam Title :
Government...
Email :
Contact :

Solution (c)

TORCH Infections

· ट�ओआरएचसी�, �जसम��टो�सो�लाज़मो�सज़�, अ�य�(�सफ�लस�, वै�रकाला-ज़ो�टर�, परोवोवायरस�बी�19), �बेला�, साइटोमेगालोवायरस


(सीएमवी) और�हप�ज़�सं�मण�शा�मल�ह��, ज�मजात��वसंग�तय��से�जुड़े�सबसे�आम�सं�मण�ह�।�

· ट�ओआरएचसी�के�अ�धकांश�सं�मण�ह�के�मातृ���णता�का�कारण�बनते�ह��, ले�कन��ूण�के�गंभीर�प�रणाम�होते�ह��, और�मातृ�सं�मण�के


उपचार�म��अ�सर��ूण�के�प�रणाम�पर�कोई��भाव�नह��पड़ता�है।�

· TORCH सं�मण�को��ूण�क��असामा�यता��का�कारण�माना�जाता�है�, �जसम��नवजात��शशु��म��भी�माइ�ोसेफली�शा�मल�है।�

Note – सीएमवी�, हप�ज�एसट�डी�ह��

QUESTION 79.
‘ नेशनल�फामा��यू�टकल��ाइ�स�ग�अथॉ�रट��(NPPA)’ के�बारे�म���न�न�ल�खत�कथन��पर��वचार�क��जए।�?

1. NPPA उन�दवा��के�मू�य��नयत�करती�है��ज�ह����स�( �ाइसेज�कं�ोल�) ऑड�र�, 2013 क��सूची�-I के�अंतग�त�आव�यक�दवा��क�


रा�ीय�सूची�(NLEM) म��रखा�गया�है।�

2. गैर-सूचीब��दवा��क��क�मत��को���तवष��10% तक�बढाने�क��अनुम�त�है��जसक���नगरानी�NPPA �ारा�क��जाती�है।�

गलत�कथन�चु�नए�:

a) केवल�1
b) केवल�2
c) 1 और�2 दोन��
d) न�तो�1 न�ही�2
Correct Answer: D
Your Answer:
Explanation

Solution (d)

दोन��कथन�सही�ह�।�

समाचार: नेशनल�फामा��यु�टकल��ाइ�स�ग�अथॉ�रट��(एनपीपीए) ने���स�(मू�य��नयं�ण) आदे श�, 2013 के�पैरा�19 के�तहत�साव�ज�नक


�हत�म��असाधारण�श��य��का�आ�ान��कया�है�, �ापार�मा�ज�न�यु��करण�के�मा�यम�से�मू�य��नयं�ण�के�तहत�42 गैर-अनुसू�चत�एंट�-क�सर
दवा��को�लाने�के��लए�, �ापार�कै�प�ग�30% पर�मा�ज�न�, जो�उनक��खुदरा�क�मत��को�85% तक�कम�करेगा।�

एनपीपीए�वत�मान�म��डीपीसीओ�क��अनुसूची�- 1 के�तहत�आव�यक�दवा��क��रा�ीय�सूची�(एनएलईएम) म��रखी�गई�दवा��क��क�मत��को


ठ�क�करता�है�

गैर-अनुसू�चत�दवा��को�हर�साल�क�मत��म��10% तक�क��वृ���क��अनुम�त�है�, �जसक���नगरानी�एनपीपीए��ारा�क��जाती�है।�

QUESTION 80.
‘एंजाइम-�ल��ड�इ�मुनोसोब�ट�अ�से�’ के�बारे�म���न�न�ल�खत�कथन��पर��वचार�क��जए।�

1. यह�सीरम�एंट�बाडी�संक��ण�का��नधा�रण�करने�हेतु�एक�उपकरण�है।�

2. इसे��ूबर�लो�सस�म��माइकोबै�ट��रयम�का�पता�लगाने�हेतु��योग��कया�जाता�है।�

3. इसे�र��नमून��म��HIV एंट�बाडीज�का�पता�लगाने�हेतु��योग��कया�जाता�है।�

IASbaba
Web: http://ilp.iasbaba.com/ Score:
Email: ilp@iasbaba.com 0.00 / 198
Page 135
2019 - Test 32 -
Exam Title :
Government...
Email :
Contact :

सही�कथन�चु�नए�

a) 1 और�3
b) केवल�3
c) 1, 2 और�3
d) 1 और�2
Correct Answer: C
Your Answer:
Explanation

Solution (c)

ELISA

· ए�लसा�(एंजाइम-�ल��ड�इ�यूनोसॉब�ट�एसे) एक��लेट-आधा�रत�टे �ट�तकनीक�है��जसे�पे�टाइड् स�, �ोट�न�, एंट�बॉडी�और�हाम�न�जैसे�पदाथ�


का�पता�लगाने�और�मा�ा�दे ने�के��लए��डज़ाइन��कया�गया�है।�

· एंजाइम�इ�यूनोएसे�(ईआईए) जैसे�अ�य�नाम��का�उपयोग�उसी�तकनीक�का�वण�न�करने�के��लए�भी��कया�जाता�है।�

· ए�लसा�म��, एक�एंट�जन�को�एक�ठोस�सतह�पर���थर��कया�जाना�चा�हए�और��फर�एक�एंट�बॉडी�के�साथ�ज�टल��कया�जाना�चा�हए�जो�एक
एंजाइम�से�जुड़ा��आ�है।�

· एक�औसत�दज��का�उ�पाद�बनाने�के��लए�एक�स�स�े ट�के�साथ�इ��यूबेशन�के�मा�यम�से�संयु��मत�एंजाइम�ग�त�व�ध�का�आकलन�करके�जांच
पूरी�क��जाती�है।�पता�लगाने�क��रणनी�त�का�सबसे�मह�वपूण��त�व�एक�उ�च��व�श��एंट�बॉडी-एंट�जन�इंटरै�शन�है।�

अनु�योग�

· �य��क�ए�लसा�को�एंट�जन�क��उप��थ�त�या�एक�नमूने�म��एंट�बॉडी�क��उप��थ�त�का�मू�यांकन�करने�के��लए��कया�जा�सकता�है�, यह�सीरम
एंट�बॉडी�सां�ता�(जैसे�एचआईवी�परी�ण�या�वे�ट�नील�वायरस) के��नधा�रण�के��लए�एक�उपयोगी�उपकरण�है।�

· इसने�खा��उ�ोग�म��संभा�वत�खा��एलज��, जैसे��ध�, मूंगफली�, अखरोट�, बादाम�, और�अंडे�और�सी�लएक�रोग�के��लए�सीरोलॉ�जकल


र��परी�ण�का�पता�लगाने�म��भी�आवेदन�पाया�है।�

· ए�लसा�का�उपयोग��वष��व�ान�म��दवा��के�कुछ�वग��के��लए�तेजी�से��क��पत����न�के��प�म��भी��कया�जा�सकता�है।�

· डॉ।�डे�नस�ई��बडवेल�और�ए�ल�टर�वोलेर�ने��व�भ���कार�क��बीमा�रय��, जैसे�ड�गू�, मले�रया�, चगास�रोग�, जोहान�क��बीमारी�और�अ�य


का�पता�लगाने�के��लए�ए�लसा�परी�ण�बनाया।�

· ए�लसा�परी�ण�का�उपयोग�मे�डकल��योगशाला��म��इन��व�ो�डाय�नो��ट�स�के��प�म��भी��कया�जाता�है।�

ए�लसा�के�अ�य�उपयोग��म��शा�मल�ह��:

· तपे�दक�म��माइकोबै�ट��रयम�एंट�बॉडी�का�पता�लगाना�

· मल�म��रोटावायरस�का�पता�लगाना�

· सीरम�म��हेपेटाइ�टस�बी�माक�र�का�पता�लगाना�

· मल�म��ई�कोलाई�के�एंटरोटॉ��सन�का�पता�लगाना�

· र��के�नमून��म��एचआईवी�एंट�बॉडी�का�पता�लगाना�

QUESTION 81.
‘ �ेयस�’ योजना�के�बारे�म���न�न�ल�खत�कथन��पर��वचार�क��जए।�

IASbaba
Web: http://ilp.iasbaba.com/ Score:
Email: ilp@iasbaba.com 0.00 / 198
Page 136
2019 - Test 32 -
Exam Title :
Government...
Email :
Contact :

1. यह�उ�चतर��श�ा�म��‘अन���हाइल�यू�लन�’ तं��क���थापना�करता�है।�

2. इसे�से�टर���कल�क��सल�(SSCs) �ारा�लागू��कया�जाएगा।�

3. यह��वशेष��प�से�इंजी�नय�र�ग��नातक��के��लए�है।�

सही�कथन�चु�नए�

a) केवल�1
b) 1 और�2
c) केवल�2
d) 1 और�3
Correct Answer: B
Your Answer:
Explanation

Solution (b)

अपर��टस�शप�और���कल�म��उ�च��श�ा��ा�त�युवा��के��लए�योजना�(SHREYAS)

· यह�रा�ीय�अ���टस�शप��ोमोशनल��क�म�( NAPS) के�मा�यम�से�अ�ैल�2019 म���नकलने�वाले�सामा�य��नातक��को�उ�ोग��श�ुता


अवसर��दान�करने�के��लए�मानव�संसाधन��वकास�मं�ालय��ारा�शु���कया�गया�है।�

· इसका�उ�े �य�भारतीय�युवा��को�रोजगार�के�अवसर��दान�करना�है�, ता�क�वे�' काय��काय��जो�खम�' और�' वजीफे�' क��कमाई�कर�सक�।�

· SHREYAS �ड�ी�पा��म��म��मु�य��प�से�गैर-तकनीक��के�छा���के��लए�एक�काय��म�है�, जो�मु�य��प�से�गैर-तकनीक��है�, जो


अपने��श�ण�म��रोजगारपरक�कौशल�का�प�रचय�दे ते�ह��, �श�ा�के�अ�भ��अंग�के��प�म���श�ुता�को�बढ़ावा�दे ते�ह��और��श�ा��व�था�म�
सरकार�के��यास��को�सु�वधाजनक�बनाने�के��लए�रोजगार�भी�उपल�ध�कराते�ह��ता�क��प��रा�ते�ह��छा���को�उनके��नातक�होने�के�दौरान�और
बाद�म��रोजगार�के�अवसर�उपल�ध�ह�।�

· SHREYAS एक�काय��म�टोकरी�है��जसम��तीन�क���य�मं�ालय��क��पहल�शा�मल�है�, अथा�त्�मानव�संसाधन��वकास�मं�ालय�, कौशल


�वकास�और�उ��मता�मं�ालय�और��म�और�रोजगार�मं�ालय�रा�ीय��श�ुता�संवध�न�योजना�( NAPS), रा�ीय�कै�रयर�सेवा�( NCS) और
उ�च��श�ण�सं�थान��म��बीए�/ बीएससी�/ बीकॉम�(�ावसा�यक) पा��म��क��शु�आत।�

· �श�ुता�अव�ध�के�दौरान�, छा��को�लगभग��।�का�मा�सक�वजीफा��मलेगा।�उ�ोग��ारा���त�माह�6,000, �जनम��से�25% क����तपू�त��N


APS काय��म�के�तहत�क��जाएगी।�अपर��टस�शप�अव�ध�के�अंत�म��, संबं�धत��े��कौशल�प�रषद��ारा�परी�ण��कया�जाएगा�और�सफल
छा���को�उनके��ड�ी��माण�प��के�अलावा�कौशल��माण�प���मलेगा।�

· एनएपीएस�योजना�के�तहत�, क���सरकार��श�ुता�क��अव�ध�के�दौरान���त�माह��टाइप�ड�का�25% अ�धकतम��।�1500 तक�साझा�करती


है।�इसके�अलावा�, मूल���श�ण�लागत�क��ओर���. 7500 तक�क��रा�श�पूरी�क��जाएगी�, जहाँ�ज�रत�है।�

उ�े �य�

· उ�च��श�ा��णाली�क��सीखने�क�����या�म��रोजगार��ासं�गकता�क��शु�आत�करके�छा���क��रोजगार��मता�म��सुधार�करना�

· �थायी�आधार�पर��श�ा�और�उ�ोग�/ सेवा��े���के�बीच�घ�न��काया��मक�संबंध�बनाना�

· छा���को�एक�ग�तशील�तरीके�से�कौशल��दान�करने�क��मांग�क��जाती�है�

· उ�च��श�ा�म��' आप�सीखते�समय�' �णाली��था�पत�करने�के��लए�

· अ�छ��गुणव�ा�क��जनश���हा�सल�करने�म���ापार�/ उ�ोग�क��मदद�करना�

· सरकार�के��यास��को�सु�वधाजनक�बनाने�के�साथ�छा��समुदाय�को�रोजगार�से�जोड़ना�

Operations

IASbaba
Web: http://ilp.iasbaba.com/ Score:
Email: ilp@iasbaba.com 0.00 / 198
Page 137
2019 - Test 32 -
Exam Title :
Government...
Email :
Contact :

· �ाथ�मक�योजना�का�संचालन�रा�ीय���श�ुता�संवध�न�योजना�(एनएपीएस) के�साथ��कया�जाएगा�, जो���येक��वसाय�/ उ�ोग�म��कुल


काय�बल�के�10% तक���श�ु��को�रखने�का��ावधान�करता�है।�

· यह�योजना�शु��म��ब��क�ग�कौशल�बीमा�सेवा�(बीएफएसआई) , खुदरा�, �वा��य�दे खभाल�, �रसंचार�, लॉ�ज��ट�स�, मी�डया�, �बंधन


सेवा��, आईट�ईएस�और�अपैरल��ारा�से�टर�कौशल�प�रषद��(एसएससी) �ारा�काया���वत�क��जाएगी।�

· उभरते��ए��श�ुता�मांग�और�पा��म�समायोजन�के�साथ�समय�के�साथ�अ�धक��े���को�जोड़ा�जाएगा।�

QUESTION 82.
�न�न�ल�खत�म��से�कौन�सी�घटना�पहले��ई�?

a) ऑपरेशन�मेघ�त�
b) �तथवाल�क��लड़ाई�
c) ल�गेवाला�क��लड़ाई�
d) रेजांग�ला�क��लड़ाई�
Correct Answer: B
Your Answer:
Explanation

Solution (b)

�टथवाल�क��लड़ाई�( 1947), रेजांग�ला�( 1962), गंगासागर�( 1971), ल�गेवाला�( 1971), ऑपरेशन��ाइड�ट�( 1971), ऑपरेशन
मेघ�त�( 1987)

QUESTION 83.
‘ फेन�व�यजीव�अ�यारण�’ अव��थत�है-

a) महारा��म��
b) झारख�ड�म��
c) म�य��दे श�म��
d) गुजरात�म��
Correct Answer: C
Your Answer:
Explanation

Solution (c)

फेन�व�यजीव�अभयार�य�

· यह�का�हा�रा�ीय�उ�ान�का�एक�लोक��य�बफर�ज़ोन�है।�

· यह�का�हा�टाइगर��रजव��के�मु�क���वेश��ार�के�पास���थत�है।�

· यह�लोक��य��प�से�फेन�व�यजीव�अभयार�य�के��प�म��जाना�जाता�है�और�म�य��दे श�और�छ�ीसगढ़�रा�य�क��सीमा��के�करीब�का�हा
बाघ�अभयार�य�के�द��णी��े��म����थत�है।�

QUESTION 84.
हाल�ही�म��, ‘ चागोस���पसमूह�’ �कस�संदभ��म��सु�ख़�य��म��था�?

a) मॉ�रशस�तथा�यूनाइटे ड��क�गडम�के�बीच�सं�भुता��ववाद�को�लेकर।�

IASbaba
Web: http://ilp.iasbaba.com/ Score:
Email: ilp@iasbaba.com 0.00 / 198
Page 138
2019 - Test 32 -
Exam Title :
Government...
Email :
Contact :

b) इसने�भारत�के��व���यह�आरोप�लगाते��ए�वाद�दायर��कया��क�भारत�ने�एक�पूव��क���त�थ�पर�परमाणु�ह�थयार��क��दौड़�को�रोकने�तथा
परमाणु��न:श��ीकरण�क��अपनी�बा�यता�को�पूरा�नह���कया�है।�
c) भारत�क��लंबी��री�क���मसाइल�परी�ण�सु�वधा।�
d) �ह�द�महासागर�म��चीन�के�बढ़ते��भाव�को�रोकने�हेतु�सेश�स�म��भारत�का�समु���सै�य�अ�ा।�
Correct Answer: A
Your Answer:
Explanation

Solution (a)

चागोस���पसमूह�

समाचार: ��टे न�दवारा�मॉ�रशस�को�"�जतनी�ज�द��हो�सके�, चौगोस���प�वापस�करना�चा�हए": आईसीजे�

इसके�बारे�म��

· वे�सात�एटोल�का�एक�समूह�है��जसम��मालद�व���पसमूह�से�लगभग�500 �कलोमीटर�द��ण�म���ह�द�महासागर�म��60 से�अ�धक


उ�णक�टबंधीय���प�शा�मल�ह�।�

· ��प��क��यह��ृंखला��ह�द�महासागर�म��लंबी�सबमरीन�पव�त��ृंखला�, चागोस-लैका�डव��रज�का�सबसे�द��णी���पसमूह�है।�

· ��प�, जो�अमे�रक��सै�य�अ�े��डएगो�गा�स�या�के�घर�ह��, 1968 म��डीकोलाइज़ेशन�के�दौरान�मॉ�रशस�के�पूव�����टश��े��से�अलग�हो�गए�थे।

· अंतरा��ीय��यायालय�( ICJ) ने�यह��नण�य��दया�है��क�अ�ध�नयम�अंतरा��ीय�कानून�के�तहत�अवैध�था।�

· आईसीजे��ारा��नण�य�केवल�सलाह�है।�

· अ��का�के�पूव��तट�से�2,000 मील�से�अ�धक��र���थत���प��पर�सं�भुता�रखने�वाले�इस�मामले�पर�अब�संयु��रा��महासभा��ारा�बहस
क��जाएगी�- �जसने�लंदन�के��वरोध�के�बावजूद�मामले�को�ICJ को�संद�भ�त��कया।�

QUESTION 85.
‘ �लोबल��ड�जटल�हे�थ�पाट� नर�शप�(GDHP)’ के��वषय�म���न�न�ल�खत�कथन��पर��वचार�क��जए।�

1. इसे�भारत��ारा�शु���कया�गया�था।�

2. यह�सरकार�, सरकारी�एज��सय��एवं��व���वा��य�संगठन�का�एक�सहयोग�है।�

सही�कथन�चु�नए�:

a) केवल�1
b) केवल�2
c) 1 और�2 दोन��
d) न�तो�1 न�ही�2
Correct Answer: B
Your Answer:
Explanation

Solution (b)

वै��क��ड�जटल��वा��य�भागीदारी�(GDHP)

· समाचार: भारत�ने�‘ 4th �लोबल��ड�जटल�हे�थ�पाट� नर�शप�स�मट’ क��मेजबानी�क��

IASbaba
Web: http://ilp.iasbaba.com/ Score:
Email: ilp@iasbaba.com 0.00 / 198
Page 139
2019 - Test 32 -
Exam Title :
Government...
Email :
Contact :

· यह�सरकार��, सरकारी�एज��सय��और�ब�रा�ीय�संगठन��(ड��यूएचओ) का�एक�अंतररा�ीय�सहयोग�है�, जो�सबूत-आधा�रत��ड�जटल


�ौ�ो�ग�कय��के�सव��म�उपयोग�के�मा�यम�से�अपने�नाग�रक��के��वा��य�और�क�याण�म��सुधार�करने�के��लए�सम�प�त�है।�

· सरकार���ौ�ो�गक��क��श���को�बढ़ावा�दे ने�और�नवाचार�और�साव�ज�नक-�नजी�भागीदारी�को�बढ़ावा�दे ने�के��लए�मह�वपूण���नवेश�कर�रही�ह�


जो�उ�च�गुणव�ा�, �टकाऊ��वा��य�और�सभी�क��दे खभाल�का�समथ�न�करती�ह�।�

· GDHP को�ऑ��े �लयाई��ड�जटल��वा��य�एज�सी��ारा�शु���कया�गया�था�

· जीडीएचपी��ड�जटल��वा��य�सेवा��के�काया��वयन�म��वै��क�सहयोग�और�सहयोग�क��सु�वधा��दान�करता�है।�

· वत�मान�म��जीडीएचपी�म��23 दे श�, �े��और��व���वा��य�संगठन�भाग�ले�रहे�ह�।�

· GDHP म����तभा�गय��म��अज�ट�ना�, ऑ��े �लया�, ऑ���या�, �ाजील�, कनाडा�, ए�टो�नया�, हांगकांग�SAR, भारत�, इंडोने�शया�, इट
ली�, जापान�, सऊद��अरब�, �यूजील�ड�, पोल�ड�, पुत�गाल�, �स�गापुर�, द��ण�के�रा�य�के�व�र���ड�जटल��वा��य�अ�धकारी�शा�मल�ह�।
को�रया�, �वीडन�, यू�ेन�, यूनाइटे ड��क�गडम�, संयु��रा�य�अमे�रका�, उ��वे�और��व���वा��य�संगठन।�

· GDHP स�चवालय�वत�मान�म��ऑ��े �लयाई��ड�जटल��वा��य�एज�सी��ारा�सम�थ�त�है�, �ड�जटल�हे�थकेयर��स�टम�के��वतरण�और


ऑ��े �लया�के��लए�रा�ीय��ड�जटल��वा��य�रणनी�त�के�मा�यम�से�सभी�ऑ��े �लयाई�लोग��के��लए��वा��य�प�रणाम��म��सुधार�के�साथ�काम
करता�है।�

QUESTION 86.
‘ �च�सुलब��ेटर�’ ��थत�है-

a) पेटागो�नया�म��
b) युकाटन��ाय��प�म��
c) मृत�सागर�म��
d) गोलन�हाइट् स�म��
Correct Answer: B
Your Answer:
Explanation

Solution (b)

�च��सुलुब��ेटर�

· यह�मै��सको�म��युकाटन��ाय��प�के�नीचे�दफन�एक�इ�पै�ट��ेटर�है�

· �भाव�क���त�थ��ेटे शस-पैलोजेन�सीमा�( K-Pg सीमा) के�साथ�ठ�क�से�मेल�खाती�है�, जो�66 �म�लयन�साल�पहले�क��तुलना�म��थोड़ा�कम


है�, और�एक��ापक��प�से��वीकार��कए�जाते�ह���स�ांत�यह�है��क�घटना�से���नया�भर�म��जलवायु��वधान��ेटे शस-पैलोजेन��वलु��त�का
कारण�था।�घटना�, एक�बड़े�पैमाने�पर��वलु�त�होने�के��लए��जसम��पृ�वी�पर�75% पौधे�और�पशु��जा�तयां��वलु�त�हो�ग��, �जसम��सभी�गैर-
ए�वयन�डायनासोर�शा�मल�ह�।�

QUESTION 87.
‘ एमने�ट��इंटरनेशनल�’ है-

a) गृह�यु���के�शरणा�थ�य��क��मदद�हेतु�सं. रा��क��एक�एज�सी।�
b) अ�यंत�गरीब�लोग��क��मदद�हेतु�एक�गैर-सरकारी��वयंसेवी�संगठन।�
c) यु��से�तबाह�दे श��म���च�क�सा�आपात��हेतु�एक�अंतर-सरकारी�एज�सी।�
d) एक�वै��क�मानवा�धकार�संगठन।�
Correct Answer: D
Your Answer:
Explanation

IASbaba
Web: http://ilp.iasbaba.com/ Score:
Email: ilp@iasbaba.com 0.00 / 198
Page 140
2019 - Test 32 -
Exam Title :
Government...
Email :
Contact :

Solution (d)

एमने�ट��इंटरनेशनल�लंदन���थत�एक�गैर-सरकारी�संगठन�है�जो�मानव�अ�धकार��पर�क���त�है।�संगठन�का�कहना�है��क���नया�भर�म��इसके
सात��म�लयन�से�अ�धक�सद�य�और�समथ�क�ह�।�

QUESTION 88.
‘FATF �े�सूची�’ म��शा�मल�दे श��से�ता�पय��है-

a) वह�दे श�मनी�लॉ���ग�व�आतंक���व�यन�से�लड़ने�हेतु�पया��त�कदम�नह��उठा�रहा।�
b) उस�दे श�का�बाजार��व��से��ापर�हेतु�पया��त��प�से�खुला�नह��है।�
c) उस�दे श�पर�मानवा�धकार�उ�लंघन�के�गंभीर�आरोप�लगे�ह�।�
d) उस�दे श�को�IMF �ारा��व�ीय�सहायता�क��आव�यकता�है।�
Correct Answer: A
Your Answer:
Explanation

Solution (a)

FATF �े��ल�ट�

· एफएट�एफ�क���े�सूची�म��वे�दे श�शा�मल�ह��जो�मनी�लॉ���ंग�और�आतंकवाद���व�पोषण�से�लड़ने�के��लए�पया��त�नह��ह�।�

· यह�इस�धारणा�को����करता�है��क��कसी�दे श�क���व�ीय��णाली�कमजोर�है�और�आतंकवाद��ग�त�व�धय��को�रोकने�के��लए���तबं�धत
मनी�लॉ���ंग�या��व�पोषण�समूह��को�रोकने�के��लए��भावी�उपाय�नह���कए�जा�रहे�ह�।�

· इन�सू�चय��पर�रखे�गए�दे श��म���वदे शी��नवेश�म��कमी�दे खी�जाती�है�और��वदे शी�कंप�नयां�आतंकवाद��ग�त�व�धय��के�संभा�वत�संबंध��पर


�वचार�करने�म��संकोच�करती�ह�।�

· IMF, व�ड��ब�क�या�ए�शयन�डेवलपम�ट�ब�क�आ�द�के��वदे शी�ऋण��भा�वत�होते�ह�।�यह�अंतररा�ीय�बाजार��से�ऋण�जुटाने�के��लए�भी�क�ठन


सा�बत�हो�सकता�है।�

QUESTION 89.
‘ डोमेन�नेम��स�टम�(DNS)’ के��वषय�म���न�न�ल�खत�कथन��पर��वचार�क��जए।�

1. यह�वह�तरीका�है��क��जससे�इ�टरनेट�डोमेन�ने�स�का�पता�लगाया�जाता�है�और�उ�ह��इ�टरनेट��ोटोकॉल�(IP) पत��म��त�द�ल��कया�जाता�है।

2. इसका�रखरखाव�गूगल�इंक��ारा��कया�जाता�है।�

सही�कथन�चु�नए�:

a) केवल�1
b) केवल�2
c) 1 और�2 दोन��
d) न�तो�1 न�ही�2
Correct Answer: A
Your Answer:
Explanation

Solution (a)

डॉमेन�नेम��स�टम�(DNS)

IASbaba
Web: http://ilp.iasbaba.com/ Score:
Email: ilp@iasbaba.com 0.00 / 198
Page 141
2019 - Test 32 -
Exam Title :
Government...
Email :
Contact :

समाचार: दे श�म��इंटरनेट�उपयोगकता���के��लए�तेजी�से�और�अ�धक�सुर��त��ाउ�ज़�ग�अनुभव��दान�करने�के�उ�े �य�से�, सरकार�ज�द�ही�एक


साव�ज�नक�डीएनएस�शु��करेगी�, यह�सु�न��त�करते��ए��क�नाग�रक��का�डेटा��थानीय��प�से�सं�हीत�हो।�

इस�बारे�म��

· हमारी�अपनी�साव�ज�नक�डीएनएस�लाने�का�मु�य�उ�े �य�उपल�धता�सु�न��त�करना�है�, �वशेष��प�से�छोटे �इ�टरनेट�सेवा��दाता�


(आईएसपी) के��लए��जनके�पास��व�सनीय�डीएनएस�नह��है।�

· रोल-आउट�को�रा�ीय�सूचना��व�ान�क����ारा��न�पा�दत��कया�जाएगा�

· ऐसा�नह��है��क�उपयोगकता���को�अ�नवाय���प�से�भारत�के�साव�ज�नक�डीएनएस�म���श�ट�करने�क��आव�यकता�होगी।�एक�उपयोगकता�
�कसी�भी�DNS को�चुनने�के��लए��वतं��है�

�या�?

· एक�DNS इंटरनेट�के��लए�एक��नद� �शका�क��तरह�है।�

· यह�कं�यूटर�, सेवा��, या�इंटरनेट�या�एक��नजी�नेटवक��से�जुड़े�अ�य�संसाधन��के��लए�एक�पदानु��मत�और��वके���कृत�नामकरण��णाली


है।�

· यह���येक���तभागी�सं�था��को�स�पे�गए�डोमेन�नाम��के�साथ��व�भ��सूचना��को�जोड़ता�है।�

· सबसे��मुख��प�से�, यह�अंत�न��हत�नेटवक���ोटोकॉल�के�साथ�कं�यूटर�सेवा��और�उपकरण��क��पहचान�और�पहचान�के��लए�आव�यक
आईपी�ए�ेस�के��लए�अ�धक�आसानी�से�याद��कए�गए�डोमेन�नाम��का�अनुवाद�करता�है।�

· यह�डोमेन�नाम��को��न�द���करने�और���येक�डोमेन�के��लए�आ�धका�रक�नाम�सव�र��को�ना�मत�करके�इंटरनेट�संसाधन��के��लए�उन�नाम��को
मैप�करने�क���ज�मेदारी�स�पता�है।�

· यह�उन�डोमेन�नाम��को�प�रव�त�त�करने�म��मदद�करता�है�जो�लोग��के��लए�आईपी�पते�म��याद�रखने�म��आसान�होते�ह��, �जनका�उपयोग
कं�यूटर�/ मशीन���ारा�संचार�करने�के��लए��कया�जाता�है।�

· य�द�DNS या�तो�धीमा�है�या�काम�करने�म���वफल�है�, तो�उपयोगकता��वेब�पते�का�पता�नह��लगा�पाएंगे�

QUESTION 90.
‘ टोरेस�जलस��ध�’ �कनके�बीच���थत�है-

a) द. को�रया�एवं�जापान�
b) उ. को�रया�एवं�चीन�
c) ऑ��े �लया�एवं��यूजील�ड�
d) पापुआ��यू��गनी�तथा�ऑ��े �लया�
Correct Answer: D
Your Answer:
Explanation

Solution (d)

टोरेस���े ट�एक���े ट�है�जो�ऑ��े �लया�और�पापुआ��यू��गनी�के�बीच��ह�द�और��शांत�महासागर��को�जोड़ती�है।�

QUESTION 91.
‘ के�’ ( Cay ) के��वषय�म���न�न�ल�खत�कथन��पर��वचार�क��जए।�

1. यह�उस�समय��न�म�त�होता�है�जब�एक��हमनद�बफ��के�पृथ�करण�एवं�आसपास�क��आधार�च�ान�के�घष�ण��ारा�एक�U- आकार�क��घाट�
काट�दे ती�है।�

IASbaba
Web: http://ilp.iasbaba.com/ Score:
Email: ilp@iasbaba.com 0.00 / 198
Page 142
2019 - Test 32 -
Exam Title :
Government...
Email :
Contact :

2. यह�सामा�यत: आक��टक��े��म��पाया�जाता�है।�

सही�कथन�चु�नए�:

a) केवल�1
b) केवल�2
c) 1 और�2 दोन��
d) न�तो�1 न�ही�2
Correct Answer: D
Your Answer:
Explanation

Solution (d)

Cay

· इसके�अलावा��े�या�क��, कोरल�रीफ�क��सतह�पर�एक�छोटा�, कम�ऊंचाई�वाला�, रेतीला���प�है।�

· पूरे��शांत�, अटलां�टक�और�भारतीय�महासागर��म��(कै�र�बयन�म��और��ेट�बै�रयर�रीफ�और�बेलीज�बै�रयर�रीफ�पर) उ�णक�टबंधीय�तट��पर


पाया�जाता�ह�।�

· जब�समु��धाराएं�रीफ़�क��सतह�के�पार�ढ�ली�तलछट�को��डपॉ�ज़टल�नोड�ले�जाती�ह��, जहां�करंट�धीमा�हो�जाता�है�या��सरे�करंट�के�साथ
प�रव�त�त�हो�जाता�है�, तो�इसका�तलछट�भार�मु��हो�जाता�है।�

· धीरे-धीरे�, जमा�तलछट�क��परत��रीफ�सतह�पर�बनती�ह�।�इस�तरह�के�नोड् स�रीफ�के��व�डवड��या�लेवड���े���म��होते�ह��जहां�सतह�कभी-कभी


पुराने�रीफ�या�समु��तट�क��च�ान�के�उ�व�के�आसपास�होती�है।�

· तलछट�संचय�से�उ�प����प�लगभग�पूरी�तरह�से�जैव-रासाय�नक�तलछट�से�बना�है�- पौध��और�जानवर��के�कंकाल�अवशेष�- आसपास�के


रीफ�पा�र��थ�तक�तं��से।�

· य�द�सं�चत�तलछट�मु�य��प�से�रेत�है�, तो���प�को�कै�कहा�जाता�है�; य�द�वे�मु�य��प�से�बजरी�ह��, तो���प�को�मोटू �कहा�जाता�है।�

· केई�तलछट�मु�य��प�से�कै��शयम�काब�नेट�(सीएसीओ�3) से�बना�है�, मु�य��प�से�एरेगनाइट�, के�साइट�और�उ�च�मै�नी�शयम


कैलीसाइट।�

· वे�असं�य�पौध��(जैसे�, कोरलीन�शैवाल�, हरी�शैवाल�हा�लमेडा�क���जा�त) और�जानवर��(जैसे�, मूंगा�, मोल�क�, फोरै�मनीफेरा) �ारा


उ�पा�दत��कए�जाते�ह�।�छोट��मा�ा�म���स�लकेट�तलछट�भी��पंज�और�अ�य��ा�णय���ारा�योगदान��दया�जाता�है।�

· समय�के�साथ�, समु���प�ी�गुआनो�के��न�ेपण�से��म���और�वन�प�त�का��वकास�सतह�पर�हो�सकता�है।�

QUESTION 92.
‘ �म�डल�ई�ट��स�यू�रट��अलाय�स�(MESA)’ के��वषय�म���न�न�ल�खत�कथन��पर��वचार�क��जए।�

1. इसे�ऑग�नाइजेशन�ऑफ़�इ�ला�मक�कोऑपरेशन�(OIC) के��वदे श�मं��य��के�46 व��स��म��लांच��कया�गया�था।�

2. यह��क�गडम�ऑफ़�सऊद��अरब�(KSA) के��दमाग�क��उपज�है।�

सही�कथन�चु�नए�:

a) केवल�1
b) केवल�2
c) 1 और�2 दोन��
d) न�तो�1 न�ही�2
Correct Answer: D

IASbaba
Web: http://ilp.iasbaba.com/ Score:
Email: ilp@iasbaba.com 0.00 / 198
Page 143
2019 - Test 32 -
Exam Title :
Government...
Email :
Contact :
Your Answer:
Explanation

Solution (d)

म�य�पूव��सुर�ा�गठबंधन�( MESA)

· यह�उ�री�अटलां�टक�सं�ध�संगठन�क��तज��पर�अरब�दे श��का�अमे�रका��ारा�एक���ता�वत��ायो�जत�गठबंधन�है।�

· इसे�"अरब�नाटो" कहा�जा�रहा�है।�

· भावी�सद�य�सऊद��अरब�, बहरीन�, कतर�, कुवैत�, संयु��अरब�अमीरात�, ओमान�, जॉड�न�और��म��ह�।�

· �कृ�त�म��र�ा�मक�, गठबंधन�क��संरचना�का�उ�े �य�भाग�लेने�वाले�रा���के�सामा�य�उ�े �य��को�पूरा�करना�है�और��े�ीय���थरता�, सुर�ा


और�समृ���के��लए�खतर��का�पता�लगाना�है।�

QUESTION 93.
‘ मायसेटोमा�’ के��वषय�म���न�न�ल�खत�कथन��पर��वचार�क��जए।�

1. यह�एक��वलनशील�बीमारी�है�जो�केवल�मानव�एवं�पशु��के�पाँव�को��भा�वत�करती�है।�

2. यह�एक��वषाणु�रोग�है।�

3. यह�बीमारी�नंगे�पाँव�रहने�वाली�जनसं�या�म��अमूमन�पाई�जाती�है।�

सही�कथन�चु�नए�

a) 1 और�2
b) केवल�2
c) केवल�3
d) 1 और�3
Correct Answer: C
Your Answer:
Explanation

Solution (c)

मायसीटोमा�

· यह�एक��चरका�लक�, उ�रो�र��वनाशकारी���ण�इ��लैमटोरी�बीमारी�है�जो�आमतौर�पर�पैर�क��होती�है�ले�कन�शरीर�का�कोई�भी��ह�सा
�भा�वत�हो�सकता�है।�

· सं�मण�शायद�सबसे�कवक�या�बै�ट��रया�के��वचा�के�नीचे�के�ऊतक��म���ॉमै�टक�संचारण��ारा�अ�ध��हत��कया�जाता�है�

· मायसीटोमा�का�वण�न�आधु�नक�सा�ह�य�म��1694 म���कया�गया�था�, ले�कन�पहली�बार�19 व��शता�द��के�म�य�म��भारतीय�शहर�म�रा�म�


�रपोट� ��कया�गया�था�, और�इस�लए�शु��म��इसे�म�रा�फुट�कहा�जाता�था�

· माइसेटोमा�आमतौर�पर�युवा�वय�क��को��भा�वत�करता�है�, �वशेष��प�से�20 से�40 वष��क��आयु�के�पु�ष��, �यादातर��वकासशील�दे श�


म�।�

· कम�सामा�जक�आ�थ�क���थ�त�के�लोग�और�मैनुअल���मक�जैसे��क�कृ�ष�वद्�, मज�र�और�चरवाहे�सबसे�अ�धक��भा�वत�होते�ह�।�

· माइकोटोमा�के��ेरक�जीव���नया�भर�म���वत�रत��कए�जाते�ह��, ले�कन�‘माइसेटोमा�बे�ट’ म��उ�णक�टबंधीय�और�उपो�णक�टबंधीय��े���म�


�था�नक�ह��, �जसम��बोलीव�रयन��रप��लक�ऑफ�वेनेजुएला�, चाड�, इ�थयो�पया�, भारत�, मॉ�रटा�नया�, मै��सको�, सेनेगल�, सोमा�लया�, सू
डान�और�यमन�शा�मल�ह�।�

IASbaba
Web: http://ilp.iasbaba.com/ Score:
Email: ilp@iasbaba.com 0.00 / 198
Page 144
2019 - Test 32 -
Exam Title :
Government...
Email :
Contact :

· संचरण�तब�होता�है�जब��ेरक�जीव�मामूली�आघात�या�एक�मम���चोट�के�मा�यम�से�शरीर�म���वेश�करता�है�, आमतौर�पर�कांटा�चुभता�है।�

· माइसेटोमा�और�उन����य��के�बीच�एक��प��संबंध�है�जो�नंगे�पैर�चलते�ह��और�मैनुअल���मक�ह�।�यह�बीमारी�नंगे�पांव�आबाद��के�बीच
आम�है�जो��ामीण��े���म���था�नक��े���म��रहते�है।�

· उपचार�जीवाणु�के��लए��ेरक�जीव��पर��नभ�र�करता�है�; यह�एक�द�घ�का�लक�एंट�बायो�ट�स�संयोजन�है�जब�क�कवक�के��कार�के��लए�यह
एंट�फंगल�दवा��और�सज�री�का�संयु��है।�

QUESTION 94.
‘ टाइड��क�म�’ जुडी�है-

a) टाइड�एनज��से�
b) टे क�उ��मता�से�
c) वोकेशनल��े �न�ग�से�
d) �श�ा�ऋण��से�
Correct Answer: B
Your Answer:
Explanation

Solution (b)

�ौ�ो�गक��इ��यूबेशन�और�उ��मय��का��वकास�( TIDE 2.0) योजना�

· तकनीक�उ��मता�प�र��य�म��ग�त�को�बढ़ावा�दे ने�के��लए�योजना�तैयार�क��गई�है।�

· यह�योजना�उभरती��ौ�ो�ग�कय��जैसे��क�IoT, AI, �लॉक-चेन�, रोबो�ट�स�इ�या�द�का�उपयोग�करके�इन�यूबेटर��को��व�ीय�और�तकनीक�


सहायता��दान�करती�है।�

QUESTION 95.
�न�न�ल�खत�म��से�कौन�सा�दे श�‘ द��ण�ए�शयाई�मु���ापार��े��(SAFTA)’ का�भाग�नह��है�?

a) मालद�व�
b) �ीलंका�
c) �यांमार�
d) पा�क�तान�
Correct Answer: C
Your Answer:
Explanation

Solution (c)

द��ण�ए�शयाई�मु���ापार��े��(SAFTA)

· यह�इ�लामाबाद�, पा�क�तान�म��12 व��साक��स�मेलन�म���कया��आ�समझौता�है।�

· इसने�अफगा�न�तान�, बां�लादे श�, भूटान�, भारत�, मालद�व�, नेपाल�, पा�क�तान�और��ीलंका�म��2.08 �ब�लयन�लोग��का�मु���ापार��े�


बनाया�

�स�ांत�

· सम��पार�प�रकता�और�फायदे �क��पार�प�रकता�ता�क�सभी�अनुबं�धत�रा�य��को�समान��प�से�लाभ�हो�सके�, अपने�आ�थ�क�और


औ�ो�गक��वकास�के��तर�, उनके�बाहरी��ापार�के�पैटन��, और��ापार�और�टै �रफ�नी�तय��और��णा�लय��को��यान�म��रखते��ए�;

IASbaba
Web: http://ilp.iasbaba.com/ Score:
Email: ilp@iasbaba.com 0.00 / 198
Page 145
2019 - Test 32 -
Exam Title :
Government...
Email :
Contact :

· आव�धक�समी�ा��के�मा�यम�से���मक�चरण��म��टै �रफ�सुधार�कदम�क��बातचीत�, सुधार�और��व�ता�रत�;

· अ�प��वक�सत�रा�य��क���वशेष�आव�यकता��क��पहचान�और�उनके�प��म��ठोस�तरजीही�उपाय��पर�सहम�त�;

· अपने�क�चे�, अध�-संसा�धत�और��सं�कृत��प��म��सभी�उ�पाद��, �व�नमा�ण�और�व�तु��को�शा�मल�करना।�

QUESTION 96.
‘ �टाट� अप�र��क�ग��ेमवक��’ के��वषय�म���न�न�ल�खत�कथन��पर��वचार�क��जए।�

1. इसे�नी�त�आयोग��ारा�जारी��कया�जाता�है।�

2. र��क�ग�म��सव��च�तीन�रा�य��को��टाट� -अ�स�को�समथ�न�दे ने�हेतु�100 - 100 करोड़��पये��दए�जाते�ह�।�

सही�कथन�चु�नए�:

a) केवल�1
b) केवल�2
c) 1 और�2 दोन��
d) न�तो�1 न�ही�2
Correct Answer: D
Your Answer:
Explanation

Solution (d)

�टाट� अप�र��क�ग��ेमवक��

· उ�ोग�और�आंत�रक��ापार�संवध�न��वभाग�( DPIIT) ने�2019 के��लए��टाट� अप�र��क�ग�का��सरा�सं�करण�जारी��कया।�

· �टाट� अप�र��क�ग��ेमवक��का�ल�य��टाट� अ�स�को�समथ�न�दे ने�के��लए�एक�मजबूत�पा�र��थ�तक��तं���था�पत�करने�के��लए�रा�य��/ क��शा�सत


�दे श��को�र�क��दान�करना�है।�

· �ेमवक��रा�य��और�क��शा�सत��दे श��को�एक-�सरे�से�अ�छ���था��क��पहचान�करने�, सीखने�और�दोहराने�के��लए��ो�सा�हत�करता�है।�

· र��क�ग��ेमवक��2019 म��7 �तंभ�और�30 ए�शन�पॉइंट�शा�मल�ह�।�

· �तंभ�सं�थागत�सहायता�, �व�नयम��को�सरल�बनाने�, साव�ज�नक�खरीद�को�आसान�बनाने�, इ��यूबेशन�समथ�न�, सीड�फं�ड�ग�सहायता�, उ�


म��न�ध�सहायता�और�जाग�कता�और�आउटरीच�संबंधी�ग�त�व�धय��के��लए�रा�य��के�संघ�शा�सत��दे श��के��यास��का�आकलन�कर�गे।�

· र��क�ग�अ�यास�का�उ�े �य�1 मई�, 2018 से�30 जून�, 2019 तक�मू�यांकन�अव�ध�के�दौरान�रा�य��/ क��शा�सत��दे श���ारा��कए�गए
उपाय��का�मू�यांकन�करना�है।�

QUESTION 97.
‘ �वाय��’ के��वषय�म���न�न�ल�खत�कथन��पर��वचार�क��जए।�

1. यह�भारतीय�उ��मता�पा�रतं��के�भीतर�के��मुख��हतधारक��को�गवन�म�ट�ई-मा�क�ट�लेस�तक�साथ�लाता�है।�

2. यह�म�हला�उ��मता�को��वक�सत�करने�एवं�साव�ज�नक�खरीद��म��MSME �े��एवं��टाट� -अ�स�क��भागीदारी�को��ो�सा�हत�करने�का


ल�य�रखता�है।�

सही�कथन�चु�नए�:

a) केवल�1

IASbaba
Web: http://ilp.iasbaba.com/ Score:
Email: ilp@iasbaba.com 0.00 / 198
Page 146
2019 - Test 32 -
Exam Title :
Government...
Email :
Contact :

b) केवल�2
c) 1 और�2 दोन��
d) न�तो�1 न�ही�2
Correct Answer: C
Your Answer:
Explanation

Solution (c)

SWAYATT

· यह�गवन�म�ट�ई�माक�ट�लेस�( GeM) पर�eTransactions के�मा�यम�से��टाट� -अप�, म�हला�और�युवा�लाभ�को�बढ़ावा�दे ने�के��लए�एक


पहल�है।�

· यह�रा�ीय�उ�म�पोट� ल�सरकार�ई-माक�ट�लेस�के��लए�भारतीय�उ�मशीलता�पा�र��थ�तक��तं��के�भीतर��मुख��हतधारक��को�एक�साथ
लाएगा।�

· यह�आगे��व�ेता��और�सेवा��दाता��क���व�भ���े�णय��को�कैटप��टं ग�करके�समावेशी�को�बढ़ावा�दे ने�क��को�शश�करेगा�, �नमा�ता�


और��व�ेता��क���व�श���ेणी�के���श�ण�और�पंजीकरण�को�सु�वधाजनक�बनाने�के��लए�स��य�कदम�उठाएगा�, म�हला�उ��मता�का
�वकास�करेगा�और�एमएसएमई��े��और��टाट� -अप�क��भागीदारी�को�बढ़ावा�दे गा।�

QUESTION 98.
�न�न�ल�खत�म��से�कौन�सा���प�अंडमान�म����थत�नह��है�?

a) �तबे���प�
b) �म�नकॉय���प�
c) �वराज���प�
d) शहीद���प�
Correct Answer: B
Your Answer:
Explanation

Solution (b)

�म�नकॉय�ल���प�म����थत�है�

QUESTION 99.
‘ ��बल�के�मेलो�मस�’ �कसक���जा�त�है-

a) मधुम�खी�क��
b) रोड�ट्स�क��
c) सी�स�क��
d) पोरपोइस�क��
Correct Answer: B
Your Answer:
Explanation

Solution (b)

��बल�केई�मेलो�मस�

· यह�फॅ�मली�मु�रदाई�और�सबफॅ�मली�मु�रनाई�म��कृंतक�क��एक��वलु�त��जा�त�है।�

IASbaba
Web: http://ilp.iasbaba.com/ Score:
Email: ilp@iasbaba.com 0.00 / 198
Page 147
2019 - Test 32 -
Exam Title :
Government...
Email :
Contact :

· यह��ेट�बै�रयर�रीफ�के�उ�री��सरे�पर���थत�एक�वन�प�त�कोरल�, पृथक��ा�बल�केई�क��एक��था�नक��जा�त�थी।�

· रीफ�के��लए�एकमा���तनपायी��था�नकता�को�दे खते��ए�, यह�जलवायु�प�रवत�न�के�कारण�इस��तनपायी��जा�त��थम��ले�खत��वलु�त


�जा�त�हो�गयी�है।�

IASbaba
Web: http://ilp.iasbaba.com/ Score:
Email: ilp@iasbaba.com 0.00 / 198
Page 148

Você também pode gostar